[obm-l] Re: [obm-l] Questões da ESAEx Por favor!

2003-08-14 Por tôpico Cláudio \(Prática\)



Ficaram faltando estas três:
 
3) O produto das distâncias de um ponto qualquer de 
uma hipérbole de equação
   (x^2/a^2) - (y^2/b^2) = 1 às suas 
assíntotas é ?
 
Assíntotas: 
1)  ay - bx = 
0  
2)  ay + bx = 0.
 
Seja o ponto P = (r,s) pertencente à 
hipérbole.
A distancia de P até a assíntota 1 é (fórmula da 
distância de ponto a reta):
| as - br |/raiz(a^2+b^2)
 
A distancia de P até a assíntota 2 
é:
| as + br |/raiz(a^2+b^2)
 
Portanto, Produto = |a^2s^2 - 
b^2r^2|/(a^2+b^2)
 
Mas P pertence à hipérbole. Logo:
r^2/a^2 - s^2/b^2 = 1 ==>
b^2r^2 - a^2s^2 = a^2b^2 ==>
|a^2s^2 - b^2r^2| = a^2b^2 ==>
 
Produto = a^2b^2/(a^2+b^2) = 
constante.
 
*
 
7) Considerando um sistema linear de 10 equações e 
10 incógnitas, o número de multiplicações e divisões necessárias para resolvê-lo pela regra de Cramer é igual a ?
 
Serão calculados 11 determinantes 10x10 e depois 
serão necessarias 10 divisões (uma pra cada variável).
Cada determinante tem 10! termos e cada termo 
envolve o produto de 10 numeros ==> 
cada determinante envolve 10*10! multiplicações 
==> 
o numero total de multiplicações será 11*10*10! = 
10*11!
 
Assim, serão efetuadas 10*11! multiplicações e 11 
divisões.
 
Moral da história: use eliminação 
gaussiana.
 
*
 
9) A matriz da transformação que passa xy = 1 para 
a forma canônica (x^2) / 2 + (y^2)/2 = 1 é ?
 
xy = 1 é equação de uma hipérbole equilátera tendo 
os eixos coordenados como assíntotas, com semi-eixos medindo raiz(2) e focos 
sobre a reta y = x;
 
x^2/2 + y^2/2 = 1 é equação de uma circunferência 
de raio raiz(2) centrada na origem. 
 
Assim, não existe nenhuma transformação linear que 
faça esta passagem.
 
Por outro lado, se a segunda equação fosse x^2/2 - 
y^2/2 = 1, ela representaria uma hipérbole equilátera cujas assíntotas são as 
retas y = x e y = -x, com semi-eixos medindo raiz(2) e focos sobre o 
eixo-x.
 
Nesse caso, a transformação seria uma rotação de 
-pi/4 (ou 7pi/4) em torno da origem.
Matriz = | cos(-pi/4)   
sen(-pi/4) | = (1/raiz(2)) * |  1    -1  |
 
| -sen(-pi/4)  cos(-pi/4) 
| 
|  -1    1  |
 
 
Um abraço,
Claudio.


Re: [obm-l] Re: [obm-l] Problemas em Aberto - Algarismos

2003-08-14 Por tôpico Cláudio \(Prática\)
Exatamente.

O enunciado da Olimpiada Iraniana de 1999 está aqui:
http://www.geocities.com/CollegePark/Lounge/5284/math/99.html
e não fala nada sobre zeros ou número de algarismos.

Ainda estou tentando...

Um abraço,
Claudio.

- Original Message -
From: <[EMAIL PROTECTED]>
To: <[EMAIL PROTECTED]>
Sent: Tuesday, August 05, 2003 12:45 PM
Subject: [obm-l] Re: [obm-l] Problemas em Aberto - Algarismos


> Esse segundo problema caiu na OBM 2000, numa versão mais fácil.
>  Acho que foi essa versão a que vc resolveu, jah que ele dizia que as duas
> potências têm que ter o mesmo número de algarismos, de modo que os zeros
> não modificavam a quantidade de algarismos.
>  Ateh mais,
>  Yuri
>
> -- Mensagem original --
>
> >Caros colegas:
> >
> >Aqui vao dois problemas que ainda estao em aberto na lista. O primeiro
> foi
> >enviado pelo Duda Stabel. O segundo eh da olimpiada iraniana, se nao me
> >engano.
> >
> >1) Determinar o conjunto de números inteiros positivos que satisfazem à
> duas
> >condições: (i) todo número possui exatamente dois algarismos não-nulos,
> >sendo um deles o três(3), (ii) todo número é quadrado perfeito.
> >
> >2) Prove ou disprove: existe uma potencia de 2 tal que ao se permutar os
> >algarismos de sua representacao decimal obtem-se uma outra potencia de
> 2.
> >
> >Esse segundo tem uma solucao aparentemente simples, mas esta solucao
exclui
> >o caso de potencias de 2 com algarismos "0" internos (ou seja, numeros
> do
> >tipo "abcdefg").
> >
> >Um abraco,
> >Claudio.
> >
> >
> >
>
> []'s, Yuri
> ICQ: 64992515
>
>
> --
> Use o melhor sistema de busca da Internet
> Radar UOL - http://www.radaruol.com.br
>
>
>
> =
> Instruções para entrar na lista, sair da lista e usar a lista em
> http://www.mat.puc-rio.br/~nicolau/olimp/obm-l.html
> =

=
Instruções para entrar na lista, sair da lista e usar a lista em
http://www.mat.puc-rio.br/~nicolau/olimp/obm-l.html
=


Re: [obm-l] PREFERÊNCIA SEXUAL

2003-08-14 Por tôpico joao dias
At 21:17 5/8/2003 -0300, you wrote:
Olá! meus amigos,

Talvez eu não tenha mais vida longa na lista, isto se não for apedrejado por
tocar num assunto tão delicado, mas é que, opção sexual, cada qual administra
da melhor maneira possível. É bom deixar claro que em momento algum
tive intenções de subestimar o prof. Morgado, jamais faria isto, apenas
ratifiquei que o autor Jonofon Sérates participou magistralmente nos dois
programas. Agora, não vamos misturar as coisas, pois quem de nós não tem um
caso lamentável nas nossas famílias. O problema é que a verdade, muitas 
vezes é
desconfortável. Sôbre a ocorrência policial, talvez tenha sido um dos 
primeiros
a saber através da minha querida filha Dândara de 14 anos que adora o seu
método CUCA LEGAL e vinha trocando e-mail's com o autor. Todos ficamos
estarrecidos, mas a vida segue o seu curso e oremos! para que nossos 
filhos não
sejam vítimas deste mundo insãno.


Caro Jorge .

Vc. pegou outra vez a contra-mão.

Pelos adjetivos: magistral,simpático ,estratosféricos e Best-Seller's , O 
citado, parece ser um dos seus ídolos.
A questão não é de preferencia sexual  e sim de escolher entre a matemática 
como ciencia e a matemática circense (a dos Trapalhões).
De ser um bom caráter ou ser  imoral e indecente.
De escrever bons livros para o ensino ou  livros que prometem ensinar  com 
facilidade e que criam a maior confusão na cabeça dos alunos quando eles 
vão aprender a forma correta.
Ser homosexual foi uma escolha que ele fez , não cabe  o julgamento.

O problema é ver o nome  de um ÍCONE do ensino do nosso País, citado no 
meio desta sujeira.
O Mestre Morgado fez bem TODAS as escolhas ,diferentemente do referido senhor.
Seus livros são adotados nas melhores escolas e universidades.
Professor respeitado no meio acadêmico no Brasil e no exterior.
Um Mestre que dedica sua vida ao ensino da matemática aos jovens e 
professores,mesmo nas suas horas de folga está sempre presente na 
lista  com seu brilhantismo e um humor digno de um Veríssimo.
Apesar da vasta cultura é um homem de extrema humildade .Quando erra ,(são 
raros os momentos em que isto acontece) assume prontamente, sempre com um 
comentário humorado do tipo: a minha descendencia luzitana me traiu.

Eu teria grande honra em ter um filho  assistindo  as aulas do Prof. Morgado.

Eu jamais teria um filho no colégio em que o autor do CUCA-LEGAL fosse 
professor.

[]s.
joaodias
=
Instruções para entrar na lista, sair da lista e usar a lista em
http://www.mat.puc-rio.br/~nicolau/olimp/obm-l.html
=


[obm-l] Número de soluções de sistema linear - Correção

2003-08-14 Por tôpico Alexandre Daibert
Desculpem-me pelo meu erro. O problema é determinar o número de soluções 
inteiras não negativas...
Sendo assim como posso resolver? (nível de segundo grau se possível)

Claudio Buffara escreveu:

on 06.08.03 02:15, Alexandre Daibert at [EMAIL PROTECTED] wrote:

 

Gostaria de ajuda para o seguinte problema:

Calcular o número de soluções do sistema:
16a + 8b + 4c + 2d + e = 23
sendo a, b, c, d, e inteiros positivos.
se possível usar somente conhecimentos de ensino médio, se isto não for
possível, pelo tente explicar mais ou menos o q está fazendo para q um
ignorante aluno q ainda não entrou em um curso superior possa entender :-)
Alexandre Daibert - Juiz de Fora - [EMAIL PROTECTED]

=
Instruções para entrar na lista, sair da lista e usar a lista em
http://www.mat.puc-rio.br/~nicolau/olimp/obm-l.html
=
   

Oi, Alexandre:

Se a, b, c, d, e sao inteiros positivos, entao o menor valor possivel para:
16a + 8b + 4c + 2d + e  eh igual a  16*1 + 8*1 + 4*1 + 2*1 + 1 = 31 > 23.
Logo, o sistema dado (composto duma unica equacao) nao tem solucao em
inteiros positivos, ou seja, o numero de solucoes pedido eh zero.
Provavelmente, o enunciado nao eh bem esse. De uma conferida.

Um abraco,
Claudio.
=
Instruções para entrar na lista, sair da lista e usar a lista em
http://www.mat.puc-rio.br/~nicolau/olimp/obm-l.html
=
 



=
Instruções para entrar na lista, sair da lista e usar a lista em
http://www.mat.puc-rio.br/~nicolau/olimp/obm-l.html
=


[obm-l] Neve caindo

2003-08-14 Por tôpico Marcio Afonso A. Cohen
Esse problema ocupou minha cabeca durante boa parte de uma palestra
chata que eu tive que assitir hoje. Consegui fazer no finalzinho da
palestra. Eh um bom treino para o pessoal que vai fazer a 2a fase da obm
agora (em particular aos que vao fazer a 1a fase da obm-u).
"Numa determinada manha, numa determinada rua, comeca a cair neve
numa taxa constante. Ao meio dia, sai do quilometro zero dessa estrada um
carrinho limpador de neve, que tem a propriedade de retirar a neve numa
razao constante. Sabendo-se que ele passou pelo quilometro 2 as 14hs, e pelo
Km 3 as 16hs, determine o horario em que comecou a nevar."

Obs: Estou assumindo que a neve cai de forma igual independente da posicao
que voce se encontra na rua.

Abracos,
Marcio

=
Instruções para entrar na lista, sair da lista e usar a lista em
http://www.mat.puc-rio.br/~nicolau/olimp/obm-l.html
=


[obm-l] EsaEx - Quero Passar !

2003-08-14 Por tôpico João



Guto, desculpa!
Eu me bati e o erro que eu cometia estava em 
fatorar (x^4 - 1) = (x^2 + 1)(x + 1)(x - 1) assim eu sempre fiquei com algo a 
mais!
 
OBRIGADÃO MESMO!
 
 
Só as curtas agora!
 
1) Mostre que a soma de todas as raízes da eq. Z^n 
- 1 = 0, no conjunto dos complexos é zero!
 
2) A tg do ângulo que a reta normal à curva Ax + By 
+ Cx^2 + Dxy + Ey^2 + Fx^3 = na origem, forma com o eixo 0x é?
 
3) Prove que a função algébrica equivalente a 2 
arctgx + arctgy = (PI)/4 é (x^2 + 2x - 1) / (x^2 - 2x - 1)
 
4) As equações das assíntotas da função y = cotg h(x) são as retas...
 
5) A eq. polar do círculo que passa por P( sqrt3, 
75graus ) e tem centro nas retas (teta = 45graus) e (Rô sen teta - sqrt8) 
é:
 
6) Como demonstrar a relação de 
Euler, sendo (i = sqrt -1)  ?
 
7) A reta y = ax + b é perpendicular à reta tg ao 
gráfico da curva y = 1/(sqrt(x^2 +1) no ponto de abscissa x = 1. Nestas 
condições, a + b = ?
 
8) ESSA É BRABÍSSIMA!! De quantas maneiras 
diferentes se pode colocar 3 anéis em 5 dedos?
 
9) Sejam X,Y,Z matrizes de 3a. ordem em que XY = 
Z^(-1) e Y = 3X. Se Det (Z) = 12, qual o valor de Det (X)?
 
10) Considere os lugares geométricos do plano 
cartesiano definido pelas equações: E1: (x - y)^2 + x(1 + 2y) <= 7/8 
e    E2: x - y + m = 0
  Determine, caso 
existam, o valor de m e as cordenadas do ponto P(x,y), de modo que P(x,y) seja a 
única solução para E1 INTERSEÇÂO E2
 
 
 
 

  - Original Message - 
  From: 
  J Augusto 
  Tavares 
  To: [EMAIL PROTECTED] 
  Sent: Thursday, August 14, 2003 7:40 
  AM
  Subject: [obm-l] Re: [obm-l] OUTRAS 
  Questões Esaex
  
  
  numerador--[(y^3 - 1) + (y^4 -1) + (y^6 - 1)]
  denominador--(y^6 - 1)
  vou fatorar 
  (y^3 - 1) <=> (y -1)(y^2 + y + 1)
  (y^4 -1) <=> (y -1)(y^3 +y^2 + y + 1)
  (y^6 - 1) <=>(y^3 + 1)(y^3 -1)<=>(y^3+1)(y -1)(y^2 + y + 1)
  agora:
  numerador( vou colar (y-1) em evidencia):  (y-1)[(y^2 + y + 1) + (y^3 
  +y^2 + y + 1) + (y^3+1)(y^2 + y + 1)]
  denominador: (y^3+1)(y -1)(y^2 + y + 1),
  eliminando y-1 de ambos e substituindo y por 1,
  numerador: 13, denominador:6, ou seja, 13/6 esse limite!
  se eu 
  nao errei nada, eh claro! eheheh 
     
  Abracao 
     
  Guto.
  obs.: 
  eu so nao tinha efetuado as contas, mas daria um resultado sim ...!
   
   
   
   
  

Resposta: 
    Fazendo  (x+1) = y^12 , como 
x->0  ,   y->1.
 (y^3 + y^4 + y^6 - 3)/(y^6 - 1) ,  [(y^3 - 
1) + (y^4 -1) + (y^6 - 1)]/[(y^3 + 1)(y^3 -1)]
 eleminando o fator (y-1), nao existira mais 
 a indeterminacao ! 
 
Esta fatoração vai te levar novamente a 
(y^2 + 1)(y + 1)(y - 1) / (y + 1)(y2 -y + 1)(y - 1)(y^2 + y + 
1)
Faremos um bocado de conta e o 
resultado não bate 
 
 


[obm-l] Olimpiada Estadual de Matematica do Rio de Janeiro

2003-08-14 Por tôpico Olimpiada Brasileira de Matematica
Caros(as) amigos(as) da lista;

Aos interessados ja' esta' no ar o site da Olimpiada
de Matematica do Rio de Janeiro.
Confiram:

http://www.omerj.com.br/

Abracos, Nelly.

=
Instruções para entrar na lista, sair da lista e usar a lista em
http://www.mat.puc-rio.br/~nicolau/olimp/obm-l.html
=


Re: [obm-l] ENQUETE - BELEZA MATEMATICA

2003-08-14 Por tôpico A. C. Morgado
Mais cinco (sem a convicção dos cinco primeiros):
6) A reta de Euler.
7) O círculo dos 9 pontos.
8) Os teoremas belgas a respeito das seções cônicas.
9) Agora, um lema que considero engenhoso e prova muitos teoremas 
interessantes e lindos: se p eh primo, nos inteiros modulo p todo 
elemento nao-nulo eh invertível.
10)  Meu ultimo voto fica  para depois  de ponderar outras opiniões.
Morgado

===


=
Instruções para entrar na lista, sair da lista e usar a lista em
http://www.mat.puc-rio.br/~nicolau/olimp/obm-l.html
=


Re: [obm-l] Trigonometria (ajuda)

2003-08-14 Por tôpico Augusto Cesar de Oliveira Morgado
Lembre-se que voce estah resolvendo um SISTEMA de duas equaçoes; as soluçoes sao z=1, 
y=0  e z=1/2 y=raiz(3)/2.
A primeira da x=2kpi e a segunda x = pi/3 + 2kpi.
Voce estah se confundindo ao achar que z=1/2 eh soluçao e isso nao eh verdade: z=1/2 
eh soluçao desde que y=raiz(3)/2.


Em Sat, 9 Aug 2003 20:41:53 -0300 (ART), Nelson <[EMAIL PROTECTED]> disse:

> Desculpe-me, mas eu não entendi. Vou detalhar um pouco mais.
> De acordo com o livro, resolvendo o sistema, encontraríamos:
> (1) z = 1 => y = 0. Nesse caso, senx = 0 e cosx = 1; logo x= 2kpi
> (2) z = 1/2 => y = raiz3/2. Nesse caso, senx = raiz3/2 e cosx = 1/2; 
> logo x = pi/3 + 2kpi
> 
> Ok, transcrevi a resolução do livro. Mas, analizemos o (2):
> senx = sqrt3/2 => x = pi/3 + 2kpi ou x = (pi - pi/3) + 2kpi = 2pi/3 + 2kpi
> ,e em:
> cosx = 1/2 => x = +ou- pi/3 + 2kpi 
>  
> Pronto... Resolvi separadamente cada equação (senx = sqrt3/2 e cosx = 1/2), e não 
> consigo entender qual foi o critério para a solução ser somente, no caso de (2), 
> pi/3 + 2kpi.
>  
> Obrigado pela atenção.
> Nelson 
>  
> Ariel de Silvio <[EMAIL PROTECTED]> wrote:
> 
> mas e o senx??
> sen(-pi/3) = -sqrt(3)/2
>  o resultado de senx + sqrt(3).cosx seria ZERO
>  
> 
> *** MENSAGEM ORIGINAL ***
> 
> As 17:47 de 9/8/2003 Nelson escreveu:
> Olá a todos, estou com uma dúvida muito fácil, mas que não consigo uma explicação 
> teórica.
> Para resolver equações trigonométricas do tipo a(senx) + b(cosx) = c, onde a, b e c 
> são números conhecidos, existem, basicamente, três métodos para resolve-las.
> Por exemplo:
>  
> Resolver a equação senx + raiz3(cosx) = raiz3 (obs: raiz quadrada)
>  
> Utilizando o método que tenho dúvida, fica assim:
>  
> Fazemos senx = y, cosx = z e resolvemos o sistema:
> y + (raiz3)z = raiz3
> y^2 + z^2 = 1
>  
> (1) z = 1 => y = 0. Nesse caso, senx = 0 e cosx = 1; logo x= 2kpi
> (2) z = 1/2 => y = raiz3/2. Nesse caso, senx = raiz3/2 e cosx = 1/2; 
> logo x = pi/3 + 2kpi
>  
> Esse exemplo foi tirado do livro temas e metas, de Antônio dos Santos Machado. O 
> problema é que temos em cosx = 1/2, x = +ou- pi/3 + 2kpi, então, qual é a explicação 
> para descatarmos o negativo?:
>  
> Desde já, agradeço.
> Nelson 
> 
> 
> 
> -
> Conheça o novo Cadê? - Mais rápido, mais fácil e mais preciso.
> Toda a web, 42 milhões de páginas brasileiras e nova busca por imagens! 
> 
> 
> 
> -
> Conheça o novo Cadê? - Mais rápido, mais fácil e mais preciso.
> Toda a web, 42 milhões de páginas brasileiras e nova busca por imagens!
=
Instruções para entrar na lista, sair da lista e usar a lista em
http://www.mat.puc-rio.br/~nicolau/olimp/obm-l.html
=


[obm-l] Problema 6 do dia 1 da IMC-rascunhos

2003-08-14 Por tôpico Johann Peter Gustav Lejeune Dirichlet
Oi turmaOntem imprimi a prova da decima
IMC.Eu nao consegui fazer muita coisa,afinal
ainda estou nmo ensino me dio.Mas parei pra
pensar no problema seis no onibus enquanto
voltava a casa.Aqui vai um resumo.

"Seja f(x)=soma de 1 ate n de a_n*x^n um
polinomio em R[x].Se f(t)=0 acarreta Re(t)<0
entao a_k*a_(k+3)http://www.cade.com.br
=
Instruções para entrar na lista, sair da lista e usar a lista em
http://www.mat.puc-rio.br/~nicolau/olimp/obm-l.html
=


Re: [obm-l] BELEZA: belgas e pontos.

2003-08-14 Por tôpico Augusto Cesar de Oliveira Morgado
Os teoremas a respeito de as seçoes do cone por planos que nao contem o vertice serem 
elipses, parabolas ou hiperboles foram demonstrados por dois belgas, Quetelet e 
Dandelin, e sao conhecidos por muitos como os teoremas belgas. As demonstraçoes sao 
particularmente elegantes e surpreendentemente simples, principalmente se expostas no 
quadro-negro pelo Wagner.
O problema dos pontos eh o problema do jogo interrompido, no qual o dinheiro das 
apostas deve ser repartido proporcionalmente a probabilidade que cada um dos dois 
jogadores tem de ser o vencedor. Foi resolvido por Pascal e por Fermat e a soluçao de 
Fermat eh, na minha opiniao, sensacional!




Em Sat, 9 Aug 2003 18:55:15 -0300, "Nicolau C. Saldanha" <[EMAIL PROTECTED]> disse:

> On Sat, Aug 09, 2003 at 12:27:25PM -0300, A. C. Morgado wrote:
> > 8) Os teoremas belgas a respeito das seções cônicas.
> 
> Outro que eu não sei o que é.
> 
> []s, N.
> =
> Instruções para entrar na lista, sair da lista e usar a lista em
> http://www.mat.puc-rio.br/~nicolau/olimp/obm-l.html
> =
> 
> 

=
Instruções para entrar na lista, sair da lista e usar a lista em
http://www.mat.puc-rio.br/~nicolau/olimp/obm-l.html
=


Re: [obm-l] EsSA

2003-08-14 Por tôpico Henrique Patrício Sant'Anna Branco
> Numa fábrica, trabalhadores reuniram-se para
> presentear um amigo que iria casar. O presente
> escolhido foi a quantia de 900,00, que seria dividida
> igualmente entre eles. Por razões particulares, dois
> daqueles trabalhadores tiraram seus nomes da lista e,
> por isso, decidiu-se diminuir a quantia para 888,00,
> de modo que na nova divisão coubesse a cada
> participante a mesma cota de antes da saída dos dois
> colegas. Com isso, coube a cada um dos participantes a
> quantia de :

Vamos chamar de x o número de trabalhadores no início da divisão. Assim,
(x-2) é o número final de participantes. Como as cotas tem que ser iguais,
temos:
900/x = 888/(x-2) ==> 12x - 1800 = 0
Resolvendo, achamos x = 150. Agora, 900/150 = 6.
Segunda opção.

Abraços,
Henrique.

=
Instruções para entrar na lista, sair da lista e usar a lista em
http://www.mat.puc-rio.br/~nicolau/olimp/obm-l.html
=


[obm-l] Cochilo na aula de algebra

2003-08-14 Por tôpico Claudio Buffara
Caros colegas:

Aqui vai um bonitinho:

Um estudante acordou no fim de uma aula de algebra a tempo de ouvir o
professor falar: "...e vou dar uma dica pra voces: todas as raizes sao reais
e positivas."

Quando ele olhou pro quadro-negro, viu uma equacao polinomial de 10o. grau
que o professor tinha dado como dever de casa e que ele comecou a copiar
feito um maluco.

Infelizmente, o professor apagou rapidamente o quadro e ele soh teve tempo
de copiar os dois primeiros termos: x^10 - 20*x^9. Ele tambem reparou que o
termo constante era +1024.

Pergunta: Quais as raizes da equacao?

Um abraco,
Claudio.

=
Instruções para entrar na lista, sair da lista e usar a lista em
http://www.mat.puc-rio.br/~nicolau/olimp/obm-l.html
=


Re: [obm-l] Cochilo na aula de algebra

2003-08-14 Por tôpico Claudio Buffara
Title: Re: [obm-l] Cochilo na aula de algebra



Oi, Fael:

Veja a solucao do Yuri.

Um abraco,
Claudio.

on 09.08.03 22:18, [EMAIL PROTECTED] at [EMAIL PROTECTED] wrote:

Pelo teorema das raizes racionais temos as raizes p/q da eq. de modo que a[n] eh divisor de q e a[0] eh divisor de p. 
No problema abaixo temos a[n] = 1 e a[0] = 1024. Com a[n] =1 e as raizes sao reais e positivas, temos que as raizes da equacao abaixo eh o conjunto dos divisores positivos de 1024 (=2^10) que sao obtidas tomando-se o produto 2^10 dois a dois  {2,4,8,16,32,64,128,256,512,1024} 
Se cometi algum deslize, por favor Claudio (ou qualquer outro participante) me corrija. 


Em uma mensagem de 9/8/2003 20:11:17 Hora padrão leste da Am. Sul, [EMAIL PROTECTED] escreveu: 




Caros colegas: 

Aqui vai um bonitinho: 

Um estudante acordou no fim de uma aula de algebra a tempo de ouvir o 
professor falar: "...e vou dar uma dica pra voces: todas as raizes sao reais 
e positivas." 

Quando ele olhou pro quadro-negro, viu uma equacao polinomial de 10o. grau 
que o professor tinha dado como dever de casa e que ele comecou a copiar 
feito um maluco. 

Infelizmente, o professor apagou rapidamente o quadro e ele soh teve tempo 
de copiar os dois primeiros termos: x^10 - 20*x^9. Ele tambem reparou que o 
termo constante era +1024. 

Pergunta: Quais as raizes da equacao? 

Um abraco, 
Claudio.









Re: [obm-l] continuidade

2003-08-14 Por tôpico Claudio Buffara
Oi, Eder:

Por alguma razao, na tela do meu computador aparece 1/x" (aspas ao inves do
expoente numerico). Desculpe a falha. Mas bom saber que o que eu fiz serviu
pra alguma coisa.

Um abraco,
Claudio.



on 11.08.03 00:07, edalbuquerque at [EMAIL PROTECTED] wrote:

> Oi Cláudio,
> 
> Talvez vc naum tenha observado que a função é f(x)=1/x²,não
> f(x)=1/x.De qualquer maneira,a resolução abaixo deuma
> encaminhada boa e acho que consegui terminar o problema.
> 
> Brigadão,
> 
> Eder
> 
>> on 10.08.03 20:58, edalbuquerque at [EMAIL PROTECTED]
> wrote:
>> 
>>> Como eu provo que f(x)=1/x² é contínua?
> Melhor,como determinar
>>> o delta apropriado?
>>> 
>>> Grato por qualquer ajuda.
>>> 
>>> Eder
>>> 
>> Oi, Eder:
>> 
>> Devemos ter cuidado pra definir f, pois seu dominio nao cont
> em x = 0.
>> 
>> Seja  a <> 0. Temos que provar que lim(x -> a) 1/x = 1/a.
>> 
>> 
>> Seja eps > 0.
>> 
>> Como a <> 0, teremos |a| > |a|/2 > 0
>> 
>> Tomemos delta = min( a^2*eps/2, |a|/2 )
>> 
>> |x - a| < delta ==>
>> 
>> a - delta < x < a + delta ==>
>> 
>> a - |a|/2 < x < a + |a|/2 ==>
>> 
>> se a < 0, entao 3a/2 < x < a/2
>> e
>> se a > 0, entao a/2 < x < 3a/2 ==>
>> 
>> de qualquer jeito, |x| > |a|/2 ==> 1/|x| < 1/(|a|/2)
>> 
>> Assim:
>> |1/x - 1/a| = |x - a|/(|a||x|) < delta/
> (|a||a|/2) = 2delta/a^2 <= eps
>> 
>> 
>> Um abraco,
>> Claudio.
>> 
>> 
>> PS: Acabei nao respondendo a sua pergunta. O delta apropriad
> o voce acha
>> resolvendo o problema de tras pra frente, ou seja, fazendo:
>> 
>> |1/x - 1/a| = |x - a|/(|a||x|) < delta/(|a||x|) <= eps ==>
>> 
>> delta <= eps*|a|*|x|
>> 
>> A partir desse ponto, voce soh precisa achar um limitante in
> ferior para |x|
>> (no caso, eu achei |a|/2).
>> 
>> 

=
Instruções para entrar na lista, sair da lista e usar a lista em
http://www.mat.puc-rio.br/~nicolau/olimp/obm-l.html
=


Re: [obm-l] BELEZA: belgas e pontos.

2003-08-14 Por tôpico Nicolau C. Saldanha
On Sun, Aug 10, 2003 at 02:08:56AM -0300, Eduardo Wagner wrote:
> Nao ha duvida sobre o que esta escrito acima. Entretanto, ha um pedaco de
> frase assim: "e ja sabemos o que estas curvas sao".
> Sim! Nos sabemos, mas os alunos provavelmente nao.

Como não? No ensino médio eles sabem. Eu estranho a idéia de estudar
cônicas de forma puramente grega, sem mencionar que estas são as curvas
de grau 2.
 
> Ja que fui citado (ou provocado) em mensagem anterior, quero dizer que
> os alunos podem perfeitamente conhecer as conicas muito antes de estarem
> familiarizados com a geometria analitica no espaco, translacoes e rotacoes.
> E isto eh muito bom. Conhecer desde cedo as curvas e suas diversas formas,
> definidas por um unico numero chamado excentricidade.
> A demonstracao "legal" que usa as esferas eh totalmente elementar e permite
> obter um resultado surpreendente que vai agora para a "beleza matematica"
> da lista.

Só para pacificar um pouco, eu também gosto muito da demonstração com
as esferas, não estou de forma alguma querendo sugerir que a demonstração
que apresentei (de natureza algébrica) deva eliminar a demonstração de
natureza mais geométrica. Mas eu me lembro de, ao descobrir a demonstração
algébrica, ter tido aquela sensação de "isto é tão simples, pq me esconderam
isso durante tanto tempo?"...

[]s, N.
=
Instruções para entrar na lista, sair da lista e usar a lista em
http://www.mat.puc-rio.br/~nicolau/olimp/obm-l.html
=


Re: [obm-l] ENQUETE - BELEZA MATEMATICA

2003-08-14 Por tôpico Johann Peter Gustav Lejeune Dirichlet
A sete e muy legal!!!Veja a Eureka!5(acho) 

--- André Martin Timpanaro
<[EMAIL PROTECTED]> escreveu: > 1- O teorema
do número primo (pela prova
> elementar dada por Erdos)
> 2- O teorema de Pitágoras (a prova usando um
> quadrado dentro de outro 
> quadrado
> é incrível na minha opinião e muitos alunos
> nunca chegam a ver nenhuma prova 
> para esse
> teorema que também é a base da trigonometria)
> 3- Se m e n são naturais não nulos a raiz
> m-ésima de n é natural ou 
> irracional (a prova é
> uma generalização do caso manjado m=2)
> 4- Seja A uma matriz quadrada. Se
> multiplicarmos todos os elementos de uma 
> linha (ou
> coluna) pelo mesmo número e somarmos os
> resultados aos elementos 
> correspondentes
> de outra linha (ou coluna), formando a matriz
> B, então detA=detB (teorema de 
> Jacobi)
> (esse teorema é muito útil para simplificar o
> cálculo de determinantes e 
> pode ser provado
> por indução)
> 5- A fórmula para a soma dos termos de uma PA
> (por mostrar a importância de 
> se encontrar
> padrões para simplificar cálculos extensos)
> 6- Existem infinitos números primos (a prova de
> Euclides é íncrivel, sem 
> comentários)
> 7- A reta de Euler (a prova por geometria
> analítica é chata mas o resultado 
> é surpreendente)
> 8- A soma dos ângulos das faces de um poliedro
> convexo de V vértices é 
> (V-2).(360º)
> (um resultado interessante na minha opinião e
> que fornece algumas 
> informações sobre o
> poliedro)
> 9- O Teorema fundamental da álgebra (já
> apareceu na lista uma prova 
> acessível ao 2º grau)
> 10- O pequeno teorema de Fermat
> 
> 
> André T.
> 
> 
> 
> 
> >From: Claudio Buffara
> <[EMAIL PROTECTED]>
> >Reply-To: [EMAIL PROTECTED]
> >To: Lista OBM <[EMAIL PROTECTED]>
> >CC: Claudio Buffara
> <[EMAIL PROTECTED]>
> >Subject: [obm-l] ENQUETE - BELEZA MATEMATICA
> >Date: Sat, 09 Aug 2003 10:24:26 -0300
> >
> >Caros colegas da lista:
> >
> >Gostaria de contar com sua participacao numa
> enquete sobre "beleza
> >matematica".
> >
> >O que eu precisao eh que cada um de voces me
> envie uma lista contendo algo
> >como 5 a 10 problemas/teoremas que voces
> consideram os mais bonitos e cujas
> >solucoes/demonstracoes sao as mais elegantes
> e/ou inusitadas e/ou
> >engenhosas. Nao precisa incluir a
> solucao/demonstracao, apenas o enunciado.
> >No entanto, se voce tiver em mente uma
> solucao/demonstracao especifica
> >(entre varias existentes) nao deixe de
> mencionar pelo menos o metodo
> >utilizado.
> >
> >A unica restricao eh que estes resultados
> devem ser de um nivel acessivel a
> >um aluno normal de 2o. grau (ou seja, o Ultimo
> Teorema de Fermat e o 
> >Porisma
> >de Poncelet estao fora, mas o caso n = 4 do
> UTF e a versao para triangulos
> >do Porisma poderiam ser incluidos).
> >
> >Importante: os resultados devem ser acessiveis
> a um aluno normal de 2o.
> >grau, mas nao necessariamente fazer parte do
> curriculo normal do 2o. grau.
> >
> >Tambem nao precisa responder hoje ou amanha ou
> mesmo na semana que vem. 
> >Acho
> >que vale a pena pensar por um tempo e
> consultar a literatura - as vezes 
> >pode
> >ter um resultado belissimo do qual voce
> simplesmente se esqueceu por nao
> >encontra-lo ha muito tempo. As Eurekas sao uma
> otima referencia. O "Proofs
> >from the Book" tambem, apesar de nem tudo lah
> ter nivel de 2o. grau.
> >
> >Se houver um numero suficiente de respostas,
> eu me comprometo a publicar 
> >uma
> >compilacao dos problemas e teoremas mais
> votados.
> >
> >Desde jah a gradeco o interesse de quem quiser
> participar.
> >
> >Um abraco,
> >Claudio.
> >
> >
>
>=
> >Instruções para entrar na lista, sair da lista
> e usar a lista em
>
>http://www.mat.puc-rio.br/~nicolau/olimp/obm-l.html
>
>=
> 
>
_
> MSN Hotmail, o maior webmail do Brasil. 
> http://www.hotmail.com
> 
>
=
> Instruções para entrar na lista, sair da lista
> e usar a lista em
>
http://www.mat.puc-rio.br/~nicolau/olimp/obm-l.html
>
= 

___
Conheça o novo Cadê? - Mais rápido, mais fácil e mais preciso.
Toda a web, 42 milhões de páginas brasileiras e nova busca por imagens!
http://www.cade.com.br
=
Instruções para entrar na lista, sair da lista e usar a lista em
http://www.mat.puc-rio.br/~nicolau/olimp/obm-l.html
=


[obm-l] Não é piada!!!

2003-08-14 Por tôpico Frederico Reis Marques de Brito
Outra dia, quer dizer noite, assistindo o prigramo do tal "senhor gordo", em 
que ele lia alguns hilariantes erros dos alunos nas provas de redação, 
comecei a lembrar de algumas pérolas matemáticas dadas pelos alunos das mais 
diversas engenharias e afins nas provas de cálculo. Então só para distrair 
um pouco, listei algumas *( fatos verídicos ocorridos em minhas classes ou 
de amigos professores .) :

(1)   Derivar   sen xusando a  "regra do produto" , algo do tipo:   " 
cos . x  +  sen . 1 "

(2)   Usar que todas as funções matemáticas são lineares. Desta forma:  
sen(x+y) = sen x + sen y ,  e^{x-y} = e^x - e^y ,  1/ (a+b) = 1/a + 1/b  
etc...

(3)  Achar que uma reta não é uma curva...

(4) lim_{  x -> +  infinito } (x^2 + 3x + 1 ) =  infinito^2 + 3infinito + 1 
...

(5)  i = \sqrt{ - 0 } ... ( Observe a necessidade do - . )

(6)  Essa é de uma aluna de Matemática para a professora de Geometria Plana: 
"Ah professora, eu não sei pra quê tudo isto... Eu vou ser professora de 
Matemática não de Geometria.!"

Aproveito para tb puxar a orelha dos autores de livros de Cálculo I, alguns 
perlo menos,  não sei bem com qwual objetivo, talvez a simplificação 
extrema,   eles  consideram que, por exemplo,  não existe
lim_{ x -> 0}   \sqrt(x) . ...

Abraços,
Frederico.
_
MSN Hotmail, o maior webmail do Brasil.  http://www.hotmail.com
=
Instruções para entrar na lista, sair da lista e usar a lista em
http://www.mat.puc-rio.br/~nicolau/olimp/obm-l.html
=


Re: [obm-l] DÚVIDA...correçao

2003-08-14 Por tôpico Augusto Cesar de Oliveira Morgado
Eh, eh, eh! Nossa, fiz 3 ao quadrado igual a 27!
Estah corrigido abaixo.


Em Thu, 14 Aug 2003 20:58:08 -0300, "A. C. Morgado" <[EMAIL PROTECTED]> disse:

Henrique Patrício Sant'Anna Branco wrote:
 
Não tenho muita certeza das minha resoluções, mas já que ninguém respondeu,vai aí 
alguma tentativa...
Num triângulo retângulo, a hipotenusa é o triplo de um dos catetos.
Considerando B  o ângulo oposto ao menor lado, podemos afirmar que 
tgB + secB  é igual a ...

Tomando o comprimento do menor cateto como unidade, a hipotenusa vale 3, o menor 
cateto vale 1 e o outro cateto vale sqrt(8) (Pitagoras!)

 tanB + secB = 1/sqrt(8) + 3/sqrt(8) = 4/sqrt(8) = sqrt(2)
 

=
Instruções para entrar na lista, sair da lista e usar a lista em
http://www.mat.puc-rio.br/~nicolau/olimp/obm-l.html
=


Re: [obm-l] Teorema das raízes racionais

2003-08-14 Por tôpico Faelccmm
Eh isso ai henrique! Eu iria postar a mesma mensagem ainda hoje, se vc nao postasse estaria do mesmo jeito na lista, pois tbem gostaria de obter uma demonstracao algebrica ou de preferencia geometrica (as minhas preferidas!)


Em uma mensagem de 10/8/2003 01:02:43 Hora padrão leste da Am. Sul, [EMAIL PROTECTED] escreveu:


Caros,
Recentemente foi usado na lista o teorema das raízes racionais, que segue:
Se um polinômio f(x) = a_n*x^n + a_(n-1)*x^(n-1) + ... + a_0 tiver raízes
racionais, estas serão da forma p/q com p divisor de a_0 e q divisor de a_n.
Todo mundo aprende isso no ensino médio, mas é raro ver a demonstração.
Pesquisando na Internet, nao achei nada também...
Alguém saberia me dar uma demonstração desse teorema?

Grato,
Henrique.





Re: [obm-l] PROBLEMA INCOMPLETO

2003-08-14 Por tôpico Bernardo Vieira Emerick
Só agora abri o seu e-mail, e, por isso, não pude ainda refletir detidamente 
sobre o problema que foi colocado. A reflexão faz-se necessária uma vez que 
a resposta que primeiro nos vem à cabeça difere daquela que é dada como 
correta. Alerto ainda que nunca estudei nada sobre Teoria das Decisões, e, 
por conseguinte, não li o livro que sustenta a controvérsia.
Na segunda opção do problema, a probabilidade de se ganhar - os valores 
efetivamente não importam, uma vez que são iguais nas duas opções - é de 
1/2.
Consideremos que na primeira opção a probabilidade de se escolher qualquer 
um dos times é de 1/2 - parece-me que só faria sentido termos valores 
diferentes se houvesse algum tipo de conhecimento por parte daquele que 
escolhe. Digamos que o time 1 é melhor do que o time 2, isto é, que a 
probabilidade do time 1 ganhar (p1) é maior do que a do time 2 (p2) - p1 > 
p2. Como p1 + p2 = 1, segue-se que p1>1/2, e, consequentemente, p2<1/2.
A probabilidade de se ganhar será dada por p = 1/2p1 + 1/2p2 = 1/2(p1 + p2) 
= 1/2.
O que o autor do livro pode ter pensado, e isto explicaria a opção 1, é que 
se toma a escolha entre duas probabilidades diferentes - na opção 1 - e 
entre duas probabilidades iguais - na opção 2.


From: [EMAIL PROTECTED]
Reply-To: [EMAIL PROTECTED]
To: [EMAIL PROTECTED]
Subject: [obm-l] PROBLEMA INCOMPLETO
Date: Fri,  8 Aug 2003 20:21:59 -0300
Boa Noite! Pessoal,

Em atenção as opiniões dos colegas: Camilo, Augusto e Okakama, segue na 
íntegra
o enunciado completo do bombástico problema da escolha racional, que se
encontra no livro Teoria da Decisão - HOWARD RAIFFA, cuja resposta correta 
ou
melhor, a resposta do livro é a inacreditável escolha da Opção 1. (PASMEM!)

Suponha que o experimentador pergunte a um indivíduo "quem você acha que
ganhará a Primeira Copa do Mundo de Beisebol, que está para iniciar-se 
dentro
em pouco - o time da Liga Americana ou o time da Liga Nacional". "Conheço 
tão
pouco de beisebol", responde o sujeito, "que hesito em responder. Ignoro 
por
exemplo quais os times e como se houveram na temporada passada". "Está 
ótimo",
exclama o experimentador. "Eu queria escolher uma situação como essa. 
Suponha
que lhe ofereça uma escolha entre as duas seguintes opções:

Opção 1. Selecione uma equipe, Americana ou Nacional, e coloque a sua 
escolha
em um envelope selado. Se a equipe que você selecionar ganhar o jogo a se
realizar, você recebe $ 100,00. Caso contrário, você não ganha nada.

Opção 2. Retire uma bola de uma urna contendo 50 bolas alaranjadas e 50 
azuis.
Você receberá $ 100,00 se retirar uma bola alaranjada e $ 0,00 se retirar 
uma
azul. (Todas as bolas são igualmente prováveis de serem retiradas). A 
retirada
será feita no fim do jogo.

Que opção você prefere?

NOTA: Gostaria de saber qual o peso no conhecimento da modalidade de 
esporte?

Tenham um bom final de semana!


WebMail UNIFOR - http://www.unifor.br
=
Instruções para entrar na lista, sair da lista e usar a lista em
http://www.mat.puc-rio.br/~nicolau/olimp/obm-l.html
=
_
MSN Hotmail, o maior webmail do Brasil.  http://www.hotmail.com
=
Instruções para entrar na lista, sair da lista e usar a lista em
http://www.mat.puc-rio.br/~nicolau/olimp/obm-l.html
=


Re: [obm-l] ENQUETE - BELEZA MATEMATICA

2003-08-14 Por tôpico Henrique Patrício Sant'Anna Branco
> Por mais que eu ache pedante e ridiculo alguem se vangloriar de ter o QI
> mais alto do mundo, nesse caso acho que a Marilyn estah certa. Voce deve
> trocar de porta.
>
> Desculpem a minha ignorancia, mas o que ha de errado com o argumento de 1
> milhao de portas? Me parece que, nesse caso, a probabilidade de voce ter
> escolhido a porta certa de primeira eh apenas de 1/1.000.000. Logo, a
> probabilidade da outra porta ter o premio eh de 999.999/1.000.000. Ou nao?

Cláudio,

No problema original, temos três portas, escolhemos uma e o apresentador
logo em seguida abre outra que, com certeza, não tem o prêmio. Inicialmente,
havia uma chance de 1/3 de uma determinada porta conter o prêmio. Ao ser
aberta uma das portas e mostrar que ela não contém o prêmio, sobram apenas
duas portas: a que você escolheu e uma outra. É como se a probabilidade
tivesse sido "atualizada" pelo fato do apresentador mostrar uma porta que
não contém o prêmio (isso é o Teorema de Bayes se não me engano). Agora que
sobraram apenas duas portas, cada uma delas tem uma em duas chances (1/2) de
ter o prêmio e, portanto, não há justificativa (matematica) para trocar de
porta ou não. O fato do apresentador abrir uma das portas muda a
probabilidade das DUAS portas e não apenas para uma, como a Sra. Marilyn
quer nos fazer crer.

Quanto ao argumento de 1 milhão de portas... Como você disse, a
probabilidade de você ter escolhido a porta certa de primeira é de 1/10^6
que é a mesma probabilidade de cada uma das outras portas individualmente.
Lembre-se que todas as probabilidades devem somar 1 = 10^6/10^6. O caso que
você apontou (999.999/10^6) é a probabilidade combinada de todas as outras
portas (cada uma entre as 10^6 portas têm probabilidade de 1/10^6) que você
não escolheu de terem o prêmio e não de uma única porta das que você não
escolheu. Se você simplesmente muda de porta, a probabilidade continua sendo
a mesma... E, se ele abrir 777.777 portas sem o prêmio, a probabilidade de
TODAS as portas fica em 1/222.223 e, novamente, não faz diferença mudar a
porta...

Espero ter sido claro.
Abraço,
Henrique.

=
Instruções para entrar na lista, sair da lista e usar a lista em
http://www.mat.puc-rio.br/~nicolau/olimp/obm-l.html
=


Re: [obm-l] ENQUETE - BELEZA MATEMATICA

2003-08-14 Por tôpico Camilo Marcantonio Junior
   Oi Henrique e demais colegas que comentaram essa questão,
 
 O Cláudio e a Marilyn estão claramente corretos.  
 Não vou comentar a questão pois o prof Nicolau já o fez no seu  excelente artigo Como Perder Amigos e Enganar Pessoas. Abaixo, a reprodução da resposta do prof. Nicolau.
 
      um abraço,
   Camilo
 
PS: Henrique, preste atenção no erro comum que é citado no texto abaixo.
 
OFF: é interessante como as questões desse artigo geram discussões quase que intermináveis.
 
 
  EXCERTO DO ARTIGO CITADO ACIMA
1. A resposta correta é que, trocando de porta, a probabilidade de ganhar o carro é 2/3, enquanto não trocando a probabilidade é apenas 1/3. Uma forma simples de ver isto é a seguinte: trocando de porta, o convidado ganha, desde que a primeira porta que ele escolher esconda um dos dois bodes, como se pode facilmente perceber. A melhor estratégia para o convidado é, portanto, trocar sempre, e assim sua probabilidade de ganhar fica sendo 2/3.
 
O erro comum aqui é achar que, após a eliminação de uma porta (que foi aberta pelo apresentador, revelando um bode), há uma simetria entre as duas outras portas e a probabilidade de cada uma esconder o carro é 1/2. Não existe, entretanto, tal simetria, pois a porta escolhida pelo convidado não poderia, pelas regras, ser trocada pelo apresentador, enquanto a outra poderia ter sido aberta, mas não foi.
 
Este processo de fato era seguido em um programa nos Estados Unidos. Uma longa e áspera discussão ocorreu na imprensa quanto a qual era o valor correto da probabilidade, e pessoas que deveriam ser capazes de resolver um problema trivial como este passaram pela vergonha de publicar soluções erradas. Julgamos melhor esquecer os detalhes deste episódio deprimente.
Henrique_Patrício_Sant'Anna_Branco <[EMAIL PROTECTED]> wrote:

> Por mais que eu ache pedante e ridiculo alguem se vangloriar de ter o QI> mais alto do mundo, nesse caso acho que a Marilyn estah certa. Voce deve> trocar de porta.>> Desculpem a minha ignorancia, mas o que ha de errado com o argumento de 1> milhao de portas? Me parece que, nesse caso, a probabilidade de voce ter> escolhido a porta certa de primeira eh apenas de 1/1.000.000. Logo, a> probabilidade da outra porta ter o premio eh de 999.999/1.000.000. Ou nao?Cláudio,No problema original, temos três portas, escolhemos uma e o apresentadorlogo em seguida abre outra que, com certeza, não tem o prêmio. Inicialmente,havia uma chance de 1/3 de uma determinada porta conter o prêmio. Ao seraberta uma das portas e mostrar que ela não contém o prêmio, sobram apenasduas portas: a que você escolheu e uma ou!
tra. É
 como se a probabilidadetivesse sido "atualizada" pelo fato do apresentador mostrar uma porta quenão contém o prêmio (isso é o Teorema de Bayes se não me engano). Agora quesobraram apenas duas portas, cada uma delas tem uma em duas chances (1/2) deter o prêmio e, portanto, não há justificativa (matematica) para trocar deporta ou não. O fato do apresentador abrir uma das portas muda aprobabilidade das DUAS portas e não apenas para uma, como a Sra. Marilynquer nos fazer crer.Quanto ao argumento de 1 milhão de portas... Como você disse, aprobabilidade de você ter escolhido a porta certa de primeira é de 1/10^6que é a mesma probabilidade de cada uma das outras portas individualmente.Lembre-se que todas as probabilidades devem somar 1 = 10^6/10^6. O caso quevocê apontou (999.999/10^6) é a probabilidade combinada de todas as outrasportas (cada uma entre as 10^6 portas têm probabilidade de 1/10^6) que vocênão escolheu de ter!
em o
 prêmio e não de uma única porta das que você nãoescolheu. Se você simplesmente muda de porta, a probabilidade continua sendoa mesma... E, se ele abrir 777.777 portas sem o prêmio, a probabilidade deTODAS as portas fica em 1/222.223 e, novamente, não faz diferença mudar aporta...Espero ter sido claro.Abraço,Henrique.=Instruções para entrar na lista, sair da lista e usar a lista emhttp://www.mat.puc-rio.br/~nicolau/olimp/obm-l.html=Conheça o novo Cadê? - Mais rápido, mais fácil e mais preciso.
Toda a web, 42 milhões de páginas brasileiras e nova busca por imagens!

Re: [obm-l] ENQUETE - BELEZA MATEMATICA

2003-08-14 Por tôpico Bernardo Vieira Emerick
Que piada!!! Marylin vos Savant, tida como a pessoa com o maior QI do mundo 
(concordo com o Domingos Jr.: bulsshit!) confundiu tudo. O problema era 
assim: num jogo, a pessoa escolha uma entre três portas. O apresentador, 
então, abra uma das portas. Como ele sabe qual é a porta que contém o 
prêmio, ele abre uma que não o contém - já que o jogo dar-se-ia por 
encerrado. A pergunta é: o jogador deveria trocar de porta?
Segundo Marylin, sim!, porque a probabilidade da opção que ele teria 
continuaria 1/3, enquanto a outra aumentaria para 2/3!!! Qual a razão disso? 
A probabilidade da porta que ele escolheu não poderia subir subitamente para 
1/2, como sugerem os matemáticos. Ora, como então a outra porta pode??? Isso 
ela não explica.
Ela aparentemente desconhece o conceito primeiro de probabilidade, que é a 
chance de se acertar, e por isso está atrelada ao número de possibilidades 
possíveis e o número de possibilidades "requeridas" para se acertar o 
resultado. Então, a probabilidade será dada - como é de conhecimento geral, 
exceto possivelmente de Marylin - pela divisão do número de possibilidades 
"requeridas" pelo número total de possibilidades. Parece-me que ela acredita 
que a única forma de se aumentar a probabilidade é aumentando o número de 
possibilidades "requeridas". Isso justificaria o "we've learned nothing to 
allow us to revise the chances on the shell under your finger" que ela diz. 
O que mudou, e que ela incrivelmente não percebeu, é o número total de 
possibilidades. Simplificando para ela, o numerozinho de baixo diminuiu, 
então o número do outro lado do sinal de igualdade aumentou, já que o 
numerozinho de cima da fração permaneceu constante. Será que assim ela 
entenderia???



From: "Domingos Jr." <[EMAIL PROTECTED]>
Reply-To: [EMAIL PROTECTED]
To: <[EMAIL PROTECTED]>
Subject: Re: [obm-l] ENQUETE - BELEZA MATEMATICA
Date: Mon, 11 Aug 2003 19:03:11 -0300
O Noga Alon conta que fizeram esta pergunta para ele uma vez que ele
começou explicando a prova de Euclides de que há infinitos primos
em um programa de televisão, eu acho:
And today, are there still infinitely many primes?

E sem sair do clima, deem uma olhada em
http://qsilver.queensu.ca/~phil158d/intro/montyh3.htm
Eu deveria ter visto isso antes de escrever o meu artigo da Eureka!

--- x ---
Putz, essa mulher do QI mais alto do mundo (bullshit!) não concorda com o
Princípio da Indução Finita também! hehehe, o pior é que é sério!!!
=
Instruções para entrar na lista, sair da lista e usar a lista em
http://www.mat.puc-rio.br/~nicolau/olimp/obm-l.html
=
_
MSN Hotmail, o maior webmail do Brasil.  http://www.hotmail.com
=
Instruções para entrar na lista, sair da lista e usar a lista em
http://www.mat.puc-rio.br/~nicolau/olimp/obm-l.html
=


Re: [obm-l] ajuda

2003-08-14 Por tôpico Bernardo Vieira Emerick
Aparentemente, há três formas de se verificar se a equação (x^2 
+1)^2+(x^2+3x-17)^2 = 0.
1- x^2 + 1 = 0 e  x^2+3x-17 =0. As raízes que zeram os dois termos serão 
soluções. Evidentemente, isso não ocorre neste caso.
2- (x^2 + 1)^2=-[(x^2+3x-17)^2]. Isso também não pode ocorrer, já que os 
dois termos elevados ao quadrado serão positivos.
3- Cheguei a esta equação (é possível que tenha havido algum erro, uma vez 
que fiz com pressa, e não conferi os resultados): 2x^4 + 6x^3 -23x^2 - 102x 
+ 290 = 0.
Digamos que f(x) = 2x^4 + 6x^3 -23x^2 - 102x + 290. Então, a sua derivada 
será
f´(x) = 8x^3 + 18x^2 - 46x - 102. Nos pontos em que a derivada é zero, 
teremos um ponto de máximo ou de mínimo relativo, conforme o sinal da 
derivada de segunda ordem. Para que se ache as soluções do problema em 
questão, devemos substituir os valores que anulam a derivada de primeira 
ordem na função f, e conferir os seus valores. Se o ponto de máximo relativo 
for positivo e o de mínimo relativo negativo, teremos uma raiz de f. Logo, 
teremos pelo menos duas soluções reais. (Isso é garantido já que f(x) é 
contínua para todo x pertencente aos reais).
Esse exercício não será resolvido. Mais uma questão em aberto!


From: "Daniel Pini" <[EMAIL PROTECTED]>
Reply-To: [EMAIL PROTECTED]
To: <[EMAIL PROTECTED]>
Subject: [obm-l] ajuda
Date: Sat, 9 Aug 2003 18:29:16 -0300
Eu estou com duvida na seguinte questão:
dada  a equação (x²+1)²+(x²+3x-17)²=0, pode-se afirmar que no universo dos 
números reais, o seu conjunto solução:
a) é vazio
b) tem apenas dois elementos
Obs: essa é uma questão da prova do CN e segundo os cursinhos preparatorios 
a resposta correta é a) mas a marinha divulgou com  correta a alternativa 
b)

_
MSN Hotmail, o maior webmail do Brasil.  http://www.hotmail.com
=
Instruções para entrar na lista, sair da lista e usar a lista em
http://www.mat.puc-rio.br/~nicolau/olimp/obm-l.html
=


Re: [obm-l] Ajuda

2003-08-14 Por tôpico Thyago
Olá Cláudio,

Obrigado pelas dicas  :-)

Mas a resolução que eu fiz não foi nada prática não.
Eu já utilizei todas estas propriedades e não consegui chegar em nada.
Bom, só para esclarecer um pouco mais... vou colocar o exercício que gerou
tal questão:


(IME) Sejam 1, X2, X3, ..., Xn as raízes de x^n=1. Calcule: P = (1 -
x2)(1-x3)...(1-xn).

Fazendo uso de Briot-Rufini e fatoração de polinômios, conseguimos chegar
facilmente na resposta P = n.
Mas, utilizando o tratamento vetorial de números complexos com a fórmula
1-cis(a) = -2isen(a/2)cis(a/2) chegamos em

P = 2^(n-1) . S

Onde S = sen(pi/n) . sen(2pi/n) . sen(3pi/n) . ... . sen[(n-1)pi/n]

Daí, utilizando a resposta da primeira resolução com a resposta da segunda
resolução temos que S = n/[2^(n-1) ]
Dá para ver que esta demonstração para S não é nada prática.

Você citou uma "solução padrão" para este tipo de problema. Qual seria?

Aguardo resposta

Atenciosamente
¡Thyago!

- Original Message -
From: Cláudio (Prática) <[EMAIL PROTECTED]>
To: <[EMAIL PROTECTED]>
Sent: Monday, August 11, 2003 2:19 PM
Subject: Re: [obm-l] Ajuda


> Oi, Thyago:
>
> A solução "padrão" pra esse tipo de problema realmente envolve complexos e
> polinômios.
>
> Tentando resolver outros problemas similares, você vai perceber que
> complexos e polinômios são uma forma de resolução bastante natural.
>
> Os resultados básicos são os seguintes:
> 1) Todo número complexo pode ser representado na forma R*(cos(a) +
> i*sen(a)), onde "R" é um real não negativo e "a" é um real qualquer (mas
> normalmente limitado ao intervalo [0, 2pi) ou então (-pi,pi]);
> 2) e^(i*a) = cos(a) + i*sen(a): essa é a definição da função exponencial
> complexa, que permite, por exemplo, que você transforme sequências de
senos
> e cossenos de números reais em PA em sequências de complexos em PG, que as
> vezes são mais fáceis de manipular;
> 3) Um polinômio com coeficientes reais pode ser expresso como o produto de
> binômios da forma (x - b) e/ou trinômios da forma (x^2 - 2*R*cos(a)*x +
> R^2), onde a e b são números reais quaisquer e R é um real positivo.
>
> Um abraço,
> Claudio.
>
>
> - Original Message -
> From: "dex" <[EMAIL PROTECTED]>
> To: <[EMAIL PROTECTED]>
> Sent: Monday, August 11, 2003 11:05 AM
> Subject: [obm-l] Ajuda
>
>
> > Olá pessoal
> >
> > Gostaria de saber uma boa demonstração para o exercício abaixo
> >
> > P = sen(pi/n) . sen(2pi/n) . sen(3pi/n) . ... . sen[(n-1)pi/n]
> > com n Inteiro positivo
> >
> > A resposta é P = n/[2^(n-1)], mas cheguei até este resultado de uma
> maneira
> > muito pouco prática, nada natural para uma questão de matemática (de
> > vestibular). Consegui prová-la utilizando o resultado de uma outra
> questão,
> > que versava sobre polinômios e complexos. Ou seja, se eu não tivesse
visto
> > esta outra questão não conseguiria provar nada!
> >
> > Atneciosamente
> > ¡Thyago!
> >
>
> =
> Instruções para entrar na lista, sair da lista e usar a lista em
> http://www.mat.puc-rio.br/~nicolau/olimp/obm-l.html
> =
>

=
Instruções para entrar na lista, sair da lista e usar a lista em
http://www.mat.puc-rio.br/~nicolau/olimp/obm-l.html
=


Re: [obm-l] ENQUETE - BELEZA MATEMATICA

2003-08-14 Por tôpico Eduardo Casagrande Stabel
Olá!

É minha vez de enviar meus problemas/teoremas bonitos...

1) O teorema, devido a Euler, que diz que quando s > 1 temos ZETA(s) =
SOMA{ 1/n^s, n=1...infinito } = PRODUTORIO { (1 - p^(-s) ), p primo }.

2) A surpreendente constatação de que um problema aparentemente não tão
complicado como o último teorema de Fermat tenha uma solução tão extensa e
complicada.

3) A demonstração (enviada para a lista) de que um número irracional elevado
a um número irracional pode resultar um número racional. A saber, se p =
raiz(2) ^ raiz(2) é racional, está acabado; se p não for racional, é
irracional, e q = p ^ raiz(2) = 2 satisfaz o problema.

4) O método (não lembro de quem é, talvez Cauchy) para aproximar a soma de
séries SOMA{ s_n } onde s_1 > s_2 > s_3 > ... > 0, utilizando-se uma
integral.

5) A constatação maravilhosa de que certas constantes (como Pi e e) são
constantes em toda a matemática, isto é, aparecem em diversas áreas
(aparentemente desconexas) como geometria, análise, teoria dos números,
probabilidade, etc. dando a entender que toda a matemática tem um centro
firme (de onde saem os resultados) e uma arquitetura já planejada por alguém
bem mais inteligente que nós...

6) O método do escalonamento de matrizes descoberto por Gauss. Brilhante por
ser tão simples e ter levado tanto tempo para ser descoberto. Segundo o
Gilbert Strang: "Ele é tão simples que mesmo qualquer um de nós poderíamos
tê-lo descoberto...".

7) O fato de que podemos definir num espaço vetorial uma função com
propriedades simples (=o produto interno) e dele derivarmos muitos e muitos
resultados interessantes. (isto realmente me surpreendeu, quando comecei a
estudar álgebra linear, pareceu mágica a existência e as conseqüências do
produto interno)

8) A menos de isomorfismo, o conjunto dos Reais é o único corpo ordenado
completo.

9) O primeiro teorema da inconsistência de Gödel.

E para finalizar:

10) O princípio da indução finita.

Abração,
Duda.

From: "Claudio Buffara" <[EMAIL PROTECTED]>
> Caros colegas da lista:
>
> Gostaria de contar com sua participacao numa enquete sobre "beleza
> matematica".
>
> O que eu precisao eh que cada um de voces me envie uma lista contendo algo
> como 5 a 10 problemas/teoremas que voces consideram os mais bonitos e
cujas
> solucoes/demonstracoes sao as mais elegantes e/ou inusitadas e/ou
> engenhosas. Nao precisa incluir a solucao/demonstracao, apenas o
enunciado.
> No entanto, se voce tiver em mente uma solucao/demonstracao especifica
> (entre varias existentes) nao deixe de mencionar pelo menos o metodo
> utilizado.
>
> A unica restricao eh que estes resultados devem ser de um nivel acessivel
a
> um aluno normal de 2o. grau (ou seja, o Ultimo Teorema de Fermat e o
Porisma
> de Poncelet estao fora, mas o caso n = 4 do UTF e a versao para triangulos
> do Porisma poderiam ser incluidos).
>
> Importante: os resultados devem ser acessiveis a um aluno normal de 2o.
> grau, mas nao necessariamente fazer parte do curriculo normal do 2o. grau.
>
> Tambem nao precisa responder hoje ou amanha ou mesmo na semana que vem.
Acho
> que vale a pena pensar por um tempo e consultar a literatura - as vezes
pode
> ter um resultado belissimo do qual voce simplesmente se esqueceu por nao
> encontra-lo ha muito tempo. As Eurekas sao uma otima referencia. O "Proofs
> from the Book" tambem, apesar de nem tudo lah ter nivel de 2o. grau.
>
> Se houver um numero suficiente de respostas, eu me comprometo a publicar
uma
> compilacao dos problemas e teoremas mais votados.
>
> Desde jah a gradeco o interesse de quem quiser participar.
>
> Um abraco,
> Claudio.
>
>
> =
> Instruções para entrar na lista, sair da lista e usar a lista em
> http://www.mat.puc-rio.br/~nicolau/olimp/obm-l.html
> =
>
>

=
Instruções para entrar na lista, sair da lista e usar a lista em
http://www.mat.puc-rio.br/~nicolau/olimp/obm-l.html
=


Re: [obm-l] ENQUETE - BELEZA MATEMATICA

2003-08-14 Por tôpico Nicolau C. Saldanha
On Mon, Aug 11, 2003 at 01:16:11AM -0300, Claudio Buffara wrote:
> Estou extremamente decepcionado com as listas de problemas supostamente
> bonitos que foram enviadas pra lista ateh o presente momento. Imaginem soh -
> teorema do valor intermediario, secoes conicas, poliedros regulares,
> conjuntos enumeraveis. Onde voces estao com a cabeca? Isso tudo eh
> matematica do tempo da carochinha. Infinitude dos primos? Isso eh tao velho
> que ja devia ter sido revogado!

O Noga Alon conta que fizeram esta pergunta para ele uma vez que ele
começou explicando a prova de Euclides de que há infinitos primos
em um programa de televisão, eu acho:

And today, are there still infinitely many primes?

E sem sair do clima, deem uma olhada em
http://qsilver.queensu.ca/~phil158d/intro/montyh3.htm

Eu deveria ter visto isso antes de escrever o meu artigo da Eureka!

[]s, N.
=
Instruções para entrar na lista, sair da lista e usar a lista em
http://www.mat.puc-rio.br/~nicolau/olimp/obm-l.html
=


RE: [obm-l] ENQUETE - BELEZA MATEMATICA

2003-08-14 Por tôpico Johann Peter Gustav Lejeune Dirichlet
Pra quem curte beleza matematica,veja o livro
Proofs from THE BOOK.E so o melhor compilado da
perfeiçao!!!
Quer uma ai?

A demo do Erdös sobre o postulado de Bertrand.
Ou essa,tambem do Erdös:mostre que em uma
sequencia de mn+1 termos ha uma subsequencia
monotona de m termos ou de n+1 termos.

Eu nao acho essa demo de Medias muito
elegante.Mas a do log e mais legal...Vejam a
Eureka! 5.

Na Eureka!6 ha um exercicio de demonstrar que se
tg(x*pi)=4/5 entao x e irracional.
Tambem tem o teorema de Dehn-Hilbert,que mostra
que um cubo e um tetraedro nao sao
equidecomponiveis.E a versao 2-dimensoes
disso

Ah,essa demo da propriedade do baricentro:pegue
uma reta paralela a um dos lados,prolongue tudo e
use semelhança sem medir as consequencias.
Problemas de geometria de IMO tambem estao na
lista!Veja so o tres da ultima IMO!

A demo do teorema de Ptolomeu,e a demo de
prostaferese a partir de Ptolomeu.

A prova de Euclides de que ha infinitos primos
nao e a que se encontra por ai mas e equivalente:
"2 e primo.
Dado o conjunto p1,p2,p3,p4,...,pn de
primos,considere o numero 1+p1*p2*...*pn.
Se este cara for primo,produzimos mais um
primo.Caso contrario basta fatorarmos e ver que
mos primos que aparecem sao novos.E fim!"
Outra da infinitude dos primos:pegue o conjunto
dos numeros de Fermat,e mostre que nas fatoraçoes
os caras sao diferentes.

A prova de que uma raiz enesima de um inteiro ou
e inteira ou e irracional e bem curta se voce
usar o fato de que as raizes de x^n-a sao
multiplas de algum fator primo de a.

 --- Artur Costa Steiner <[EMAIL PROTECTED]>
escreveu: > Resolvi escrever imediatamente
aqueles que me
> vieram a cabeça, pois
> provavelmente são os que mais me tocaram. Não
> olhei ainda as outras opiniões
> da lista, para não ser influenciado.
> 
> 1) A prova de que toda sequencia de numero
> reais contem uma subsequencia
> monotonica.
> 
> 2)A famosa e linda prova de Euclides de que o
> conjunto dos numeros primos eh
> infinito.
> 
> 3) A prova de Cantor, baseada em expansoes
> decimais, de que o conjunto dos
> reais naum eh numeravel. 
> 
> 4)A elegante prova da desigualdade das medias
> aritmetica e geometrica
> baseada na propriedade da funcao exponencial
> segundo a qual e^x >= 1+ x para
> todo real x (acho que eh acessivel ao nivel
> medio).
> 
> 5)A prova de que, se n e p sao inteiros
> positivos e a= n^(1/p) nao eh
> inteiro, entao a eh irracional
> 
> 6) A simples e muito engenhosa prova de Cantor
> de que nenhum conjunto eh
> equivalente ao conjunto de suas partes, a  qual
> tem como corolario a
> conclusao de que o conjunto das partes de N (os
> naturais) nao eh numeravel.
> 
> 7) A surpreendentemente simples prova de que os
> racionais sao numeraveis
> 
> 8) A prova de que entre dois reais distintos
> hah uma infinidade de racionais
> e de irracionais.
> 
> 8)O lindo teorema de Dandelin, das conicas
> 
> 9) A prova de que as medianas de um triangulo
> encontram-se em um mesmo
> ponto, o baricentro, o qual, sobre cada
> mediana, estah a 2/3 do vertice eh a
> 1/3 da base.
> 
> 10) E este, muito simples, caiu no vestibular
> interno do antigo curso Vetor,
> em 1969: Em um triangulo ABC, o circulo
> inscrito c tangencia AB e AC nos
> pontos M e N. A partir de um ponto O sobre o
> arco MN, de c, distinto de M e
> de N, traca-se a  tangente a c, que intersecta
> Ab e AC nos pontos P e Q.
> Mostre que o perimetro do triangulo APQ
> independe da escolha do ponto O.   
> 
> Indo soh um pouquinho alem do nivel medio
> (desculpe-me, Claudio, se estou
> extrapolando!),  cito ainda a prova de alguns
> teoremas que acho lindos de
> morrer:
> 
> Se a funcao real f eh monotonica em um
> intervalo I, entao o conjunto dos
> pontos de descontinuidade de f em I eh
> numeravel.
> 
> No conjunto dos reais, derivadas apresentam a
> propriedade do valor
> intermediario
> 
> Se A eh compacto e B eh um subconjunto infinto
> de A, entao B tem um ponto de
> acumulacao em A.  Se f eh continua em um
> conjunto compacto, entao f eh
> uniformemente continua neste conjunto. 
> 
> Subconjuntos perfeitos de espacos metricos
> compactos nao sao numeraveis.
> 
> Aproveito a oportunidade para perguntar: Existe
> alguma conclusao da
> matematica que vc considere contraria aa
> intuicao? Eu, por exemplo, acho um
> tanto contra intuitivo que o fato de f ser
> diferenciavel  em R e apresentar
> limite no infinito nao implique que f'
> apresente limite zero no infinito.
> Algumas pessoas acham contra intuitivo que a
> serie harmonica seja
> divergente.
> Artur
> 

> ATTACHMENT part 2 application/ms-tnef
name=winmail.dat
 

___
Conheça o novo Cadê? - Mais rápido, mais fácil e mais preciso.
Toda a web, 42 milhões de páginas brasileiras e nova busca por imagens!
http://www.cade.com.br
=
Instruções para entrar na lista, sair da lista e usar a lista em
http://www.mat.puc-rio.br/~nicolau/olimp/obm-l.h

Re: [obm-l] Re: [obm-l] DÚVIDA... ajudarr

2003-08-14 Por tôpico Claudio Buffara
on 14.08.03 19:26, Henrique Patrício Sant'Anna Branco at
[EMAIL PROTECTED] wrote:

> Não tenho muita certeza das minha resoluções, mas já que ninguém respondeu,
> vai aí alguma tentativa...
> 
>> O número natural N = 180p, onde p é um número primo, possui 27 divisores
>> naturais. O valor de p é:
>> a) 2 b) 3 c)5 d) 7 e) 11
> 
> Creio que aqui seja N = 180^p, uma vez que 180p é divisível por {2 2 3 3 5
> p}.
> De qualquer forma, não sei resolver essa questão, mas creio que seja fácil
> usar congruências (as quais não domino). Eu mesmo gostaria de uma ajuda
> nesse problema.
> 
Oi, Henrique:

Na verdade eh multiplicacao mesmo: 180p = 2^2 * 3^2 * 5^1 * p^1

Se p > 5, entap num.div = (2+1)*(2+1)*(1+1)*(1+1) = 36 ==> p eh 2, 3 ou 5.
(a formula pro numero de divisores positivos de um inteiro pode ser achada
por combinatoria)

Se p = 2 ou 3, entao num.div = (3+1)*(2+1)*(1+1) = 24 ==> p = 5
Checando: num.div = (2+1)*(2+1)*(2+1) = 27 ==> OK

Alternativa (c).

 
>> 2) Num triângulo retângulo, a hipotenusa é o triplo de um dos catetos.
>> Considerando   o ângulo oposto ao menor lado, podemos afirmar que tg   +
> sec
>> é igual a  ?
>

Chame os lados de: 3a, a, b  (3a = hipotenusa)

Pitagoras ==> 9a^2 = a^2 + b^2 ==> b^2 = 8a^2 ==> b = 2*raiz(2)*a > a

Logo, o angulo de medida x eh oposto ao lado de medida a ==>

tg(x) + sec(x) = a/(2*raiz(2)*a) + 3a/(2*raiz(2)*a) = raiz(2)
 
> Essa é meio difícil de explicar... Tente fazer o desenho de um triângulo
> retângulo, use trigonometria e o fato de que sec(x) = 1/cos(x).
> Assim, sec(x) + tan(x) = 4B/A.
> 
>> 4) Um Aspirante ganhou, em uma competição na Escola Naval, quatro livros
>> diferentes de Matemática, três livros diferentes de Física e dois livros
>> diferentes de Português.  Querendo manter juntos aqueles da mesma
>> disciplina, concluiu que poderia enfileirá-los numa prateleira de sua
>> estante, de diversos modos.  A quantidade de modos com que poderá fazê-lo
> é
>> 
>> (A)48
>> (B 72
>> (C192
>> (D864
>> (E)1728
> 
> Ordenando cada grupo, temos para os de Matemática 24 = 4*3*2 maneiras de
> dispô-los entre si, para os de Física 6 = 3*2 maneiras e para os de
> Português, 2 maneiras.
> Agora, consideremos cada conjunto de livros como um único, facilita.
> Portanto, se tivermos 3 grupos diferentes, podemos ordená-los de 3*2 = 6
> formas.
> Portanto, o total de modos que esse estudante pode organizar os livros na
> estante é 24*6*2*6 = 1728, letra E.
> 
> Abraços,
> Henrique.
> 
> =
> Instruções para entrar na lista, sair da lista e usar a lista em
> http://www.mat.puc-rio.br/~nicolau/olimp/obm-l.html
> =
> 

=
Instruções para entrar na lista, sair da lista e usar a lista em
http://www.mat.puc-rio.br/~nicolau/olimp/obm-l.html
=


Re: [obm-l] probleminha

2003-08-14 Por tôpico Fábio Dias Moreira
-BEGIN PGP SIGNED MESSAGE-
Hash: SHA1

Em Tuesday 12 August 2003 10:51, elton francisco ferreira escreveu:
> José se deslocou entre as cidades A e B tres vezes
> pelo mesmo caminho, utilizando, em cada uma das vezes,
> um meio de transporte diferente. Na primeira ves foi
> de carro, com uma velocidade média de 60 Km/h. Na
> segunda vez doi de bicicleta, com velocidade média de
> 30 Km/h, e na terceira vez foi de moto, com velocidade
> média de 40 km/h. Sabendo que a soma dos tempos gastos
> nos tres deslocamentos doi igual a 45 h, o tempo gasto
> em cada um dos deslocamentos foi, respectivamente:
> [...]

Seja L o comprimento do percurso. Então 45 = L/60 + L/30 + L/40 <=>
45 = 9L/120 <=> L = 600 km. Logo os temos gastos foram 600/60 = 10 h, 600/30 = 
20 h e 600/40 = 15h.

[]s,

- -- 
Fábio "ctg \pi" Dias Moreira
-BEGIN PGP SIGNATURE-
Version: GnuPG v1.2.2 (GNU/Linux)

iD8DBQE/OPrWalOQFrvzGQoRAiGJAJ9Y17r96cfZ6TAUfdiwVTLCL5nkDACgi5Dq
/YgDroffjxl7XbYqfJUdPaU=
=qrQZ
-END PGP SIGNATURE-

=
Instruções para entrar na lista, sair da lista e usar a lista em
http://www.mat.puc-rio.br/~nicolau/olimp/obm-l.html
=


Re: [obm-l] Descida de fermat...

2003-08-14 Por tôpico Johann Peter Gustav Lejeune Dirichlet
 --- [EMAIL PROTECTED] escreveu: > Prove que
a equação x^4+4y^4=z^2, não possui
> soluções inteiras não nulas...
>Obrigado, 
>Crom
>  

Acho que isso e um corolario do teorema de fermat
no caso n=4.Talvez na Eureka 7 tenha algo sobre.

___
Conheça o novo Cadê? - Mais rápido, mais fácil e mais preciso.
Toda a web, 42 milhões de páginas brasileiras e nova busca por imagens!
http://www.cade.com.br
=
Instruções para entrar na lista, sair da lista e usar a lista em
http://www.mat.puc-rio.br/~nicolau/olimp/obm-l.html
=


Re: [obm-l] equação trigonometrica

2003-08-14 Por tôpico A. C. Morgado




Eleve ao quadrado e voce obtera   modulo(2senxcosx)=0. Isso dah x = k
pi/2. Conferindo, OK.

Nelson wrote:

  Olá a todos,
  Mais uma vez, peço ajuda de vocês para resolver uma questão de
equação trigonometrica
   
  Obtenha a solução da equação abaixo:
  |senx| + |cosx| = 1
   
  Desde já, agradeço.
  Nelson
  
  
  Conheça
o novo Cadê? - Mais rápido, mais fácil e mais preciso.
Toda a web, 42 milhões de páginas brasileiras e nova busca por imagens!





[obm-l] equação trigonometrica

2003-08-14 Por tôpico Nelson
Olá a todos,
Mais uma vez, peço ajuda de vocês para resolver uma questão de equação trigonometrica
 
Obtenha a solução da equação abaixo:
|senx| + |cosx| = 1
 
Desde já, agradeço.
NelsonConheça o novo Cadê? - Mais rápido, mais fácil e mais preciso.
Toda a web, 42 milhões de páginas brasileiras e nova busca por imagens!

Re: [obm-l] Re:

2003-08-14 Por tôpico Johann Peter Gustav Lejeune Dirichlet
Como nao consegui demonstrar isto em tempo
finito,alguem poderia demonstrar pra mim? 

--- Eduardo Wagner <[EMAIL PROTECTED]> escreveu: > 
> 
> --
> >From: 
> >Date: Mon, Aug 11, 2003, 5:06 PM
> >
> 
> > Na minha opiniao o Porisma de poncelet e que
> e
> > contra-intuitivo:como e que e que uma coisa
> tao
> > bonita pode ter uma demonstraçao tao feia???
> >
> > Dois problemas que nao resolvi mas acho
> legais
> > neste ponto de vista:
> > 1)Existe uma funçao continua apenas nos
> > racionais?
> 
> Nao.
> 
> > 2)Existe uma funçao continua apenas nos
> > irracionais?
> 
> Sim. Se x = p/q (irredutivel com p e q
> inteiros, q > 0),
> seja f(x) = 1/q se x eh racional e f(x) = 0 se
> x eh irracional.
> >
> >
> >
> >
> >  --- Claudio Buffara
> > <[EMAIL PROTECTED]> escreveu: > on
> > 10.08.03 00:50, Artur Costa Steiner at
> >> [EMAIL PROTECTED] wrote:
> >>
> >> > Aproveito a oportunidade para perguntar:
> >> Existe alguma conclusao da
> >> > matematica que vc considere contraria aa
> >> intuicao? Eu, por exemplo, acho um
> >> > tanto contra intuitivo que o fato de f ser
> >> diferenciavel  em R e apresentar
> >> > limite no infinito nao implique que f'
> >> apresente limite zero no infinito.
> >> > Algumas pessoas acham contra intuitivo que
> a
> >> serie harmonica seja
> >> > divergente.
> >> > Artur
> >> >
> >> Oi, Artur:
> >>
> >> Gostaria de ver que exemplos outras pessoas
> da
> >> lista vao dar, mas assim de
> >> bate-pronto eu diria que acho
> contra-intuitivo:
> >>
> >> 1) que existam funcoes continuas em toda a
> reta
> >> mas sem derivada em nenhum
> >> ponto;
> >>
> >> 2) o fato de, sendo a irracional, o conjunto
> {
> >> m + na ; m, n inteiros } ser
> >> denso em R;
> >>
> >> 3) que Pi tenha alguma relacao com a soma
> dos
> >> inversos dos quadrados dos
> >> naturais;
> >>
> >> 4) que um problema tao simples como o de 3
> >> corpos sujeitos a atracao
> >> gravitacional mutua possa ter uma solucao
> >> caotica;
> >>
> >> 5) que um conjunto nao enumeravel possa ter
> >> medida nula;
> >>
> >> 6) que exista uma bijecao entre R e R^2;
> >>
> >> 7) a maioria dos resultados quase-milagrosos
> de
> >> analise complexa;
> >>
> >> 8) que R possa ser bem-ordenado e que isso
> seja
> >> consequencia de um negocio
> >> tao intuitivo como o axioma da escolha.
> >>
> >> 9) que o porisma de Poncelet nao possa ser
> >> provado apenas por geometria
> >> Euclidiana.
> >>
> >> Mas acho que todos esses sao pinto se
> >> comparados ao
> >>
> >> 10) paradoxo de Banach-Tarski - voce pode
> >> decompor uma esfera do tamanho de
> >> uma ervilha em no maximo 5 pedacos e
> re-montar
> >> esses pedacos de modo a
> >> formar uma esfera do tamanho do Sol
> >>
> >> E com essa, vou dormir...
> >>
> >> Um abraco,
> >> Claudio.
> >>
> >>
> >
>
=
> >> Instruções para entrar na lista, sair da
> lista
> >> e usar a lista em
> >>
> >
>
http://www.mat.puc-rio.br/~nicolau/olimp/obm-l.html
> >>
> >
>
=
> >
> >
>
___
> > Conheça o novo Cadê? - Mais rápido, mais
> fácil e mais preciso.
> > Toda a web, 42 milhões de páginas brasileiras
> e nova busca por imagens!
> > http://www.cade.com.br
> >
>
=
> > Instruções para entrar na lista, sair da
> lista e usar a lista em
> >
>
http://www.mat.puc-rio.br/~nicolau/olimp/obm-l.html
> >
>
=
>
=
> Instruções para entrar na lista, sair da lista
> e usar a lista em
>
http://www.mat.puc-rio.br/~nicolau/olimp/obm-l.html
>
= 

___
Conheça o novo Cadê? - Mais rápido, mais fácil e mais preciso.
Toda a web, 42 milhões de páginas brasileiras e nova busca por imagens!
http://www.cade.com.br
=
Instruções para entrar na lista, sair da lista e usar a lista em
http://www.mat.puc-rio.br/~nicolau/olimp/obm-l.html
=


Re: [obm-l] Ajuda

2003-08-14 Por tôpico Claudio Buffara
Oi, Thyago:

Vou te confessar uma coisa: usando a identidade 1 - cis(a) =
-2isen(a/2)cis(a/2) e mais esse problema do IME, que alias eh uma
propriedade classica (e, como voce mostrou, util!) das raizes n-esimas da
unidade, voce chegou a uma solucao mais curta e elegante do que a que eu
tinha em mente. Parabens!

A minha ideia era separar os casos n par e n impar e fatorar x^n - 1 de duas
maneiras diferentes:
Primeiro: 
x^(2m) - 1 = (x^2 - 1)*(x^(2m-2) + x^(2m-4) + ... + x^4 + x^2 + 1)
x^(2m+1) - 1 = (x - 1)*(x^(2m) + x^(2m-1) + ... + x^2 + x + 1)

Depois:
x^(2m) - 1 = (x^2 - 1)*PRODUTO(1<=k<=m-1)(x^2 - 2xcos(kpi/m)x + 1)
x^(2m+1) - 1 = (x - 1)*PRODUTO(1<=k<=m)(x^2 - 2xcos(2kpi/(2m+1)) + 1)

E depois, fazer x = 1 e igualar as expressoes obtidas, mas a sua solucao eh
mais simples e, portanto, melhor.

O passo que faltou na sua solucao foi mostrar explicitamente que
(-i)^(m-1)*cis(pi/n)*cis(2pi/n)*...*cis((n-1)pi/n) = 1
mas isso eh bem facil (apesar de nao ser evidente).

Um abraco,
Claudio.

PS: Se essa sua solucao nao eh "pratica", entao eu nao sei o que eh. Repare:
voce tem um produto de senos de numeros em PA. Como voce propoe calcula-los?
Puramento por meio de identidades trigonometricas, sem usar complexos? Boa
sorte...

on 12.08.03 00:45, Thyago at [EMAIL PROTECTED] wrote:

> Olá Cláudio,
> 
> Obrigado pelas dicas  :-)
> 
> Mas a resolução que eu fiz não foi nada prática não.
> Eu já utilizei todas estas propriedades e não consegui chegar em nada.
> Bom, só para esclarecer um pouco mais... vou colocar o exercício que gerou
> tal questão:
> 
> 
> (IME) Sejam 1, X2, X3, ..., Xn as raízes de x^n=1. Calcule: P = (1 -
> x2)(1-x3)...(1-xn).
> 
> Fazendo uso de Briot-Rufini e fatoração de polinômios, conseguimos chegar
> facilmente na resposta P = n.
> Mas, utilizando o tratamento vetorial de números complexos com a fórmula
> 1-cis(a) = -2isen(a/2)cis(a/2) chegamos em
> 
> P = 2^(n-1) . S
> 
> Onde S = sen(pi/n) . sen(2pi/n) . sen(3pi/n) . ... . sen[(n-1)pi/n]
> 
> Daí, utilizando a resposta da primeira resolução com a resposta da segunda
> resolução temos que S = n/[2^(n-1) ]
> Dá para ver que esta demonstração para S não é nada prática.
> 
> Você citou uma "solução padrão" para este tipo de problema. Qual seria?
> 
> Aguardo resposta
> 
> Atenciosamente
> ¡Thyago!
> 
> - Original Message -
> From: Cláudio (Prática) <[EMAIL PROTECTED]>
> To: <[EMAIL PROTECTED]>
> Sent: Monday, August 11, 2003 2:19 PM
> Subject: Re: [obm-l] Ajuda
> 
> 
>> Oi, Thyago:
>> 
>> A solução "padrão" pra esse tipo de problema realmente envolve complexos e
>> polinômios.
>> 
>> Tentando resolver outros problemas similares, você vai perceber que
>> complexos e polinômios são uma forma de resolução bastante natural.
>> 
>> Os resultados básicos são os seguintes:
>> 1) Todo número complexo pode ser representado na forma R*(cos(a) +
>> i*sen(a)), onde "R" é um real não negativo e "a" é um real qualquer (mas
>> normalmente limitado ao intervalo [0, 2pi) ou então (-pi,pi]);
>> 2) e^(i*a) = cos(a) + i*sen(a): essa é a definição da função exponencial
>> complexa, que permite, por exemplo, que você transforme sequências de
> senos
>> e cossenos de números reais em PA em sequências de complexos em PG, que as
>> vezes são mais fáceis de manipular;
>> 3) Um polinômio com coeficientes reais pode ser expresso como o produto de
>> binômios da forma (x - b) e/ou trinômios da forma (x^2 - 2*R*cos(a)*x +
>> R^2), onde a e b são números reais quaisquer e R é um real positivo.
>> 
>> Um abraço,
>> Claudio.
>> 
>> 
>> - Original Message -
>> From: "dex" <[EMAIL PROTECTED]>
>> To: <[EMAIL PROTECTED]>
>> Sent: Monday, August 11, 2003 11:05 AM
>> Subject: [obm-l] Ajuda
>> 
>> 
>>> Olá pessoal
>>> 
>>> Gostaria de saber uma boa demonstração para o exercício abaixo
>>> 
>>> P = sen(pi/n) . sen(2pi/n) . sen(3pi/n) . ... . sen[(n-1)pi/n]
>>> com n Inteiro positivo
>>> 
>>> A resposta é P = n/[2^(n-1)], mas cheguei até este resultado de uma
>> maneira
>>> muito pouco prática, nada natural para uma questão de matemática (de
>>> vestibular). Consegui prová-la utilizando o resultado de uma outra
>> questão,
>>> que versava sobre polinômios e complexos. Ou seja, se eu não tivesse
> visto
>>> esta outra questão não conseguiria provar nada!
>>> 
>>> Atneciosamente
>>> ¡Thyago!
>>> 
>> 
>> =
>> Instruções para entrar na lista, sair da lista e usar a lista em
>> http://www.mat.puc-rio.br/~nicolau/olimp/obm-l.html
>> =
>> 
> 
> =
> Instruções para entrar na lista, sair da lista e usar a lista em
> http://www.mat.puc-rio.br/~nicolau/olimp/obm-l.html
> =
> 

=
Instruções para entrar na lista, sair da list

Re: [obm-l] Questões Esaex - RETA FINAL

2003-08-14 Por tôpico Fábio Dias Moreira
-BEGIN PGP SIGNED MESSAGE-
Hash: SHA1

Em Tuesday 12 August 2003 14:01, João escreveu:
> [...]
> 2) QUESTÃO DESGRAÇADA!
>
> Num determinado país, em um ano de transição, a sua moeda foi desvalorizada
> frente ao dólar em 10% ao mês nos primeiros 6 meses e nos 6 meses seguintes
> em 6% ao mês. Neste ano a desvalorização média mensal foi de:  
>   GABARITO: 7,98% am
> [...]

Seja I a desvalorização média. Então

(1 - 0,1)^6*(1 - 0,06)^6 = (1 - I)^12

1 - I = sqrt(0,9 * 0,94)

I = 8,02% a.m.

> [...]
> 6) Expandir a função f(x) = 1 / (1 + x2) em potências de x-2, determinando
> o intervalo de convergência da série obtida.
> [...]

Note que (1 - x^2 + x^4 - x^6 + x^8 - ...) = 1 / (1 + x^2), pois é uma soma de 
PG. Não está claro para mim onde entra o x^-2, mas note que se a série 
converge, então a PG converge, logo -1 < -x^2 < 1 <==> -1 < x < 1.

> [...]
> 8) Mostre que a soma de TODAS as raízes da equação Zn – 1 = 0, no conjunto
> dos números complexos é zero.
> [...]

Isso é falso para n = 1. Supondo n >= 2 natural, basta usar que a soma das 
raízes é -0/1 = 0 por Girard.

> [...]
> 14) A área do triângulo cujos vértices são, o centro da esfera 3x2+3y2+3z2
> – 6x+12y+3 = 0, a origem do espaço e o ponto (1,0,0) é ( COMO ACHO O CENTRO
> DA ESFERA???)
> [...]

Complete os quadrados (ou seja, escreva todas as expressões da forma x^2 + 2ax 
como (x+a)^2 - a^2)

[]s,

- -- 
Fábio "ctg \pi" Dias Moreira
-BEGIN PGP SIGNATURE-
Version: GnuPG v1.2.2 (GNU/Linux)

iD8DBQE/OZwhalOQFrvzGQoRAgZGAJ4vuQqB6W/9bCiGZXZFPJiwXCqJ3wCfWWS0
EBLsxai9XYYVIYjaTL4Vp54=
=PlNt
-END PGP SIGNATURE-

=
Instruções para entrar na lista, sair da lista e usar a lista em
http://www.mat.puc-rio.br/~nicolau/olimp/obm-l.html
=


Re: [obm-l] Re: [obm-l] DÚVIDA... ajudarr

2003-08-14 Por tôpico A. C. Morgado






Henrique Patrício Sant'Anna Branco wrote:

  Não tenho muita certeza das minha resoluções, mas já que ninguém respondeu,
vai aí alguma tentativa...

  
  
Num triângulo retângulo, a hipotenusa é o triplo de um dos catetos.
Considerando B  o ângulo oposto ao menor lado, podemos afirmar que 
tgB + secB  é igual a ...


  
  Tomando o comprimento do menor cateto como unidade, a hipotenusa vale 3, o menor cateto vale 1 e o outro cateto vale sqrt(26) (Pitagoras!)

tanB + secB = 1/sqrt(26) + 3/sqrt(26) = 4/sqrt(26) = 2*sqrt(26)/13








Re: [obm-l] Re: [obm-l] DÚVIDA... ajudarr

2003-08-14 Por tôpico A. C. Morgado






Henrique Patrício Sant'Anna Branco wrote:

  Não tenho muita certeza das minha resoluções, mas já que ninguém respondeu,
vai aí alguma tentativa...

  
  
O número natural N = 180p, onde p é um número primo, possui 27 divisores
naturais. O valor de p é:
a) 2 b) 3 c)5 d) 7 e) 11

  
  A decomposiçao em fatores primos de N eh N = 2.2.3.3.5.p

Se p for um primo diferente de 2, de 3 e de 5, o numero de divisores de
N serah 3.3.2.2=36, pois um divisor de N pode ter nenhum, um ou dois
fatores iguais a 2 (3 possibilidades), nenhum, um ou dois fatores
iguais a 3 (3 possibilidades), nenhum ou um fator igual a 5 (2
possibilidades), nenhum ou um fator igual a p (2 possibilidades) e nao
pode ter nenhum fator primo diferente desses.
Logo, p vale 2 ou 3 ou 5. Raciocinando do mesmo modo, se p=2 ha 24
divisores, se p=3 ha 24 divisores e se p=5 ha 27 divisores. Logo, p=5.

  
  
  
  





[obm-l] Fwd: Fabrício sem Avião (Jornal O Povo de hoje)

2003-08-14 Por tôpico Olimpiada Brasileira de Matematica


Date: Tue, 12 Aug 2003 08:43:34
-0300 (ART)
From: Paulo Rodrigues <[EMAIL PROTECTED]>
Subject: Fabrício sem Avião (Jornal O Povo de hoje)
To: [EMAIL PROTECTED]
X-Spam-Status: No, hits=2.0 required=5.0
tests=BIG_FONT,HTML_FONT_COLOR_GRAY,_javascript_,SPAM_PHRASE_00_01,
 
SPAM_REDIRECTOR,SUPERLONG_LINE
version=2.43
X-Spam-Level: **

Orgulho cearense nas olimpíadas

Fábio Campos

[12 Agosto 02h44min]


É com imenso orgulho que assistimos o desempenho de estudantes cearenses
em competições científicas internacionais. A edição de ontem do O
POVO mostrou o caso de Haroldo Heitor Ribeiro Filho, 17, que
ganhou a medalha de prata na Olimpíada Internacional de Química. Para
conquistar o segundo lugar, Haroldo teve que ser melhor que outros 271
estudantes de todo planeta que brigaram por um lugar no pódio. Antes,
cada um deles ainda teve que disputar duras seletivas em seus países de
origem. Além de Haroldo, outros três estudantes cearenses conseguiram se
classificar para Antenas 2003. Ano passado, Yuri Morais, outro cearense,
já havia trazido medalha da mesma competição, que teve vez na Holanda.
Nenhum deles é de escola pública. São os melhores alunos dos melhores
colégios de Fortaleza. Depois da aprovação nas seletivas, é preciso
levantar o financiamento para passagens e estadias. É aí que entra a
capacidade de articulação dos colégios privados. Elas articulam, e bem,
mas quem pagou a conta, e com prazer, é o contribuinte através da Funcap
e CNPq. Coisa de R$ 8 mil por ''atleta''. 


O MÉRITO DE FABRÍCIO SEM AVIÃO 
Fazer este tipo de articulação virou um excelente negócio para os
colégios da Aldeota. É a chance de estampar nos jornais o aluno campeão.
E sem custos. Quando conseguem classificar um aluno numa dessas disputas,
os colégios privados sabem que ganham prestígio, respeito e, no fim das
contas, muito mais matrículas num dos ramos empresariais de maior
concorrência na cidade. Os colégios fazem com mestria o que, por exemplo,
a Universidade Federal do Ceará (UFC) não consegue fazer. Estudante da
Graduação em Ciências da Computação da UFC, Fabrício Siqueira Benevides
não teve a mesma fortuna dos colegas ''atletas'' do segundo grau. Ele foi
o ÚNICO universitário cearense e nordestino a passar nas
seletivas que davam o passaporte para compor a equipe verde a amarela nas
Olimpíadas Universitárias Internacionais de Matemática. Só que a disputa
já aconteceu entre os dias 25 e 30 de julho, mas Fabrício não foi. Não
foi, creiam, porque não conseguiu as prosaicas passagens aéreas para
Bucarest, na Romênia. 


COMO CRUCIFICAR UM JOVEM CIENTISTA 
Fabrício é tido como um brilhante aluno da UFC. Já é experiente em
Olimpíadas. Ano passado, ganhou a medalha de prata nas olimpíadas
universitárias nacionais. Foi muito mais fácil conquistar a medalha e
ultrapassar as acirradas seletivas para Bucarest que conseguir um reles
financiamento para pegar um avião. Em maio, quando foi oficialmente
selecionado, de posse de uma carta da Presidente da Sociedade Brasileira
de Matemática, Suely Druck, Fabrício procurou a Coordenação da Graduação
na UFC. Começou aí uma terrível via-crucis. Sem sucesso, o jeito foi
procurar a Reitoria. Não passou da ante-sala do novo reitor. A Secretaria
de Ciência e Tecnologia do Ceará fez ouvidos de mercador. Detalhe: a
Funcap, que é vinculada à Secitece, foi quem ajudou os estudantes das
escolas privadas a viajarem para Antenas. O calvário continuou na
pró-reitoria de Planejamento da UFC. Por fim, resolveu apelar para a
Sociedade Brasileira de Matemática e junto à organização da Olimpiada
Brasileira de Matemática. Fabrício não conseguiu sair de Fortaleza. Na
época, as passagens custavam U$ 1.600,00. 


A UFC QUE NÃO É DOS ESPERTALHÕES 
Quem teve a oportunidade de ler a edição de julho do Jornal da Ciência,
produzido pela SBPC, viu estampado o orgulho da Universidade Federal de
Santa Catarina em enviar um seu aluno que se classificou para Bucarest.
Aqui, a UFC saiu de fininho. Agiu burocraticamente e fez de conta que não
tinha nada a ver com o caso. Perdeu a rara oportunidade de chamar a
atenção da sociedade para o que há de bom por lá. Afinal, a UFC não é
feita apenas de uma elite que usou de lamentáveis subterfúgios para
compor a lista dos grandes marajás da aposentadoria nacional. Tem muita
coisa boa na UFC, o problema é que ela não sabe mostrar. Pior, esconde.
Pior ainda, contribui dolorosamente para que um de seus melhores alunos
tenha fartos motivos para acreditar que não vale a pena mergulhar nos
estudos. Fabrício tem hoje todos os motivos para acreditar que precisa
carregar influência politiqueira a tiracolo se quiser convencer as
instituições públicas ligadas à ciência a cumprirem o papel que define
suas existências. Coisa de terra de mercadores. Se Fabrício fosse aluno
de uma das universidades privadas hoje garbosamente instaladas em
Fortaleza certamente teria ido a Bucarest. Certamente teria seu rosto
estampado num belo anúncio publicitário. Alguém duvida? 



Conhe

[obm-l] Fórmulas

2003-08-14 Por tôpico amurpe
Oi pessoal,

No jogo da mega sena os numeros do tipo 01 02...o9 são 
considerados dezenas e as dezenas variam de 01 a 80.

Discutindo com meus colegas, levantamos algumas dúvidas 
sobre a quantidade de dezenas quando não consideramos 1, 
2 e 3 dígitos.Conseguimos fazer usando excel , mas 
quando tentamos arrumar fórmulas aí que as duvidas 
apareceram. Consegui a muito custo arrumar algumas 
fórmulas que deram resultado em vários casos, mas não 
tenho lá muita certeza.

Uma delas é a seguinte. 

1) Quantas dezenas existem sem o dígito zero e limitando 
a dezena ( menor ou igual), por exemplo:

Sem considerar o Zero quantas dezenas existem menores ou 
iguais a 54 ? fazendo um montão de tentativas chegamos a 
fórmula:

Q=(D-1)*9+u, onde Q=quantidade das dezenas procuradas.
D=5 e u=4.
Q=(5-1)*9+4, daí . Q=40 dezenas.

2)Sem considerar um dígito diferente de zero e 
imaginando que as dezenas sejam menores ou iguais a 54. 
Chegamos a fórmula.

Q=(D-1)*9 +u-1 , D=5, u=4 dái : Q=4*9+4-1 , Q=39.

3) Não considerando dois dígitos e imaginando limitação 
nas dezenas( menor ou igual),

Chegamos a :

i)Q=(D-2)*8+(u-1)( quando o dígito que não 
consideraremos é diferente de zero)

ii)Q=(D-2)*8+(u-1)( quando os dígitoa não são diferentes 
de zero).

fizemos muitas experiências , mas não temos certeza .

Gostaríamos de uma ajuda.pois como saber se isso tá 
certo mesmo.Se estiver certo será muito legal pois é 
sinal de que estamos aprendendo a deduzir fórmulas 
através da observação.

A razão da duvida é que quando passamos para tres 
dígitos a cois foi complicando muito.

Tudo foi fruto de muita curiosidade.

Aguardo um retorno de vocês.

Um abraço.

Amurpe


 

 
__
Acabe com aquelas janelinhas que pulam na sua tela.
AntiPop-up UOL - É grátis!
http://antipopup.uol.com.br/


=
Instruções para entrar na lista, sair da lista e usar a lista em
http://www.mat.puc-rio.br/~nicolau/olimp/obm-l.html
=


[obm-l] Pagina Catarinense

2003-08-14 Por tôpico Olimpiada Brasileira de Matematica

Caros(as) amigos(as) da lista: 

Pagina Eletronica da Olimpiada Catarinense de Matematica.

www.pet.mtm.ufsc.br/indexorm.html


Abracos, Nelly 



Re: [obm-l] Soma de Senos

2003-08-14 Por tôpico Johann Peter Gustav Lejeune Dirichlet
Talvez uma induçaozinha ajude...
Mas o jeito complexo e o mais facil...
 --- Claudio Buffara
<[EMAIL PROTECTED]> escreveu: > Oi,
Thyago:
> 
> Uma solucao 100% trigonometrica pra essa soma
> de senos voce encontra no
> livro do Luis Lopes - Manual de Trigonometria,
> ou entao, voce pode usar
> sen(x) = (e^(ix) - e^(-ix))/(2i) e transformar
> a soma em duas PGs complexas.
> 
> Os dois jeitos sao um pouco bracais.
> 
> Um abraco,
> Claudio.
> 
> on 12.08.03 21:07, Thyago at [EMAIL PROTECTED]
> wrote:
> 
> > Olá Claudio e companheiros da lista
> > 
> > Bom, sabe que estou me convencendo mesmo que
> esta solução é prática :-)
> > 
> > O que eu estava querendo inicialmente é uma
> solução que nem a da questão
> > abaixo, veja só:
> > 
> > S = sen(a) + sen(2a) + sen(3a) + ... +
> sen(na)
> > 
> > Em que a solução consiste em multiplicar
> ambos os lados da igualdade pelo
> > seno da metade da razão da PA, e após efetuar
> a prostaférese e sair
> > cortando. Sem muitas delongas!
> > ...
> > 
> > Já ouvi dizer que a resolução que procuro
> existe, e está escrita em um tal
> > livro russo chamado "Lidski, problemas de
> PA", ou algo do gênero... mas
> > nunca tive o privilégio de ter algum contato
> com essa obra. Alguém já ouviu
> > falar?
> > 
> > Atenciosamente
> > ¡Thyago!
> > 
> > 
> > 
> > - Original Message -
> > From: Claudio Buffara
> <[EMAIL PROTECTED]>
> > To: <[EMAIL PROTECTED]>
> > Sent: Tuesday, August 12, 2003 9:58 AM
> > Subject: Re: [obm-l] Ajuda
> > 
> > 
> >> Oi, Thyago:
> >> 
> >> Vou te confessar uma coisa: usando a
> identidade 1 - cis(a) =
> >> -2isen(a/2)cis(a/2) e mais esse problema do
> IME, que alias eh uma
> >> propriedade classica (e, como voce mostrou,
> util!) das raizes n-esimas da
> >> unidade, voce chegou a uma solucao mais
> curta e elegante do que a que eu
> >> tinha em mente. Parabens!
> >> 
> >> A minha ideia era separar os casos n par e n
> impar e fatorar x^n - 1 de
> > duas
> >> maneiras diferentes:
> >> Primeiro:
> >> x^(2m) - 1 = (x^2 - 1)*(x^(2m-2) + x^(2m-4)
> + ... + x^4 + x^2 + 1)
> >> x^(2m+1) - 1 = (x - 1)*(x^(2m) + x^(2m-1) +
> ... + x^2 + x + 1)
> >> 
> >> Depois:
> >> x^(2m) - 1 = (x^2 -
> 1)*PRODUTO(1<=k<=m-1)(x^2 - 2xcos(kpi/m)x + 1)
> >> x^(2m+1) - 1 = (x - 1)*PRODUTO(1<=k<=m)(x^2
> - 2xcos(2kpi/(2m+1)) + 1)
> >> 
> >> E depois, fazer x = 1 e igualar as
> expressoes obtidas, mas a sua solucao
> > eh
> >> mais simples e, portanto, melhor.
> >> 
> >> O passo que faltou na sua solucao foi
> mostrar explicitamente que
> >>
>
(-i)^(m-1)*cis(pi/n)*cis(2pi/n)*...*cis((n-1)pi/n)
> = 1
> >> mas isso eh bem facil (apesar de nao ser
> evidente).
> >> 
> >> Um abraco,
> >> Claudio.
> >> 
> >> PS: Se essa sua solucao nao eh "pratica",
> entao eu nao sei o que eh.
> > Repare:
> >> voce tem um produto de senos de numeros em
> PA. Como voce propoe
> > calcula-los?
> >> Puramento por meio de identidades
> trigonometricas, sem usar complexos? Boa
> >> sorte...
> >> 
> >> on 12.08.03 00:45, Thyago at [EMAIL PROTECTED]
> wrote:
> >> 
> >>> Olá Cláudio,
> >>> 
> >>> Obrigado pelas dicas  :-)
> >>> 
> >>> Mas a resolução que eu fiz não foi nada
> prática não.
> >>> Eu já utilizei todas estas propriedades e
> não consegui chegar em nada.
> >>> Bom, só para esclarecer um pouco mais...
> vou colocar o exercício que
> > gerou
> >>> tal questão:
> >>> 
> >>> 
> >>> (IME) Sejam 1, X2, X3, ..., Xn as raízes de
> x^n=1. Calcule: P = (1 -
> >>> x2)(1-x3)...(1-xn).
> >>> 
> >>> Fazendo uso de Briot-Rufini e fatoração de
> polinômios, conseguimos
> > chegar
> >>> facilmente na resposta P = n.
> >>> Mas, utilizando o tratamento vetorial de
> números complexos com a fórmula
> >>> 1-cis(a) = -2isen(a/2)cis(a/2) chegamos em
> >>> 
> >>> P = 2^(n-1) . S
> >>> 
> >>> Onde S = sen(pi/n) . sen(2pi/n) .
> sen(3pi/n) . ... . sen[(n-1)pi/n]
> >>> 
> >>> Daí, utilizando a resposta da primeira
> resolução com a resposta da
> > segunda
> >>> resolução temos que S = n/[2^(n-1) ]
> >>> Dá para ver que esta demonstração para S
> não é nada prática.
> >>> 
> >>> Você citou uma "solução padrão" para este
> tipo de problema. Qual seria?
> >>> 
> >>> Aguardo resposta
> >>> 
> >>> Atenciosamente
> >>> ¡Thyago!
> >>> 
> >>> - Original Message -
> >>> From: Cláudio (Prática)
> <[EMAIL PROTECTED]>
> >>> To: <[EMAIL PROTECTED]>
> >>> Sent: Monday, August 11, 2003 2:19 PM
> >>> Subject: Re: [obm-l] Ajuda
> >>> 
> >>> 
>  Oi, Thyago:
>  
>  A solução "padrão" pra esse tipo de
> problema realmente envolve
> > complexos e
>  polinômios.
>  
>  Tentando resolver outros problemas
> similares, você vai perceber que
>  complexos e polinômios são uma forma de
> resolução bastante natural.
>  
>  Os resultados básicos são os seguintes:
>  1) Todo número complexo pode ser
> representado na forma R*(cos(a) +
>  i*sen(a)), onde "R" é um real não negativo
> e "a" é um real qualquer
> > (mas
>  normalmente limitado ao intervalo [0, 2pi)
> ou então (-pi,pi]);
>  2) e^(

[obm-l] MMC

2003-08-14 Por tôpico Pedro Costa



 
  Quantos pares de inteiros 
positivos A e B existem cujo mínimo múltiplo comum é 126000?
   ( Considere o par (A,B) como sendo o 
mesmo que (B,A)--
Esta mensagem foi verificada pelo sistema de antivírus e 
 acredita-se estar livre de perigo.



[obm-l] ENQUETE - BELEZA MATEMATICA

2003-08-14 Por tôpico Claudio Buffara
Caros colegas:

Foi com uma mistura de surpresa e horror que eu constatei que alguns
participantes da lista interpretaram literalmente o que eu escrevi em minha
ultima mensagem sobre o assunto em epigrafe, do qual alias sou patrocinador.
O que me entristece eh que um deles ja havia me causado uma impressao muito
positiva por sua disposicao de tirar duvidas e de mostrar um pouco da beleza
que existe na matematica.

Sendo assim, o que me resta fazer eh esclarecer que aquela mensagem tinha
dois objetivos:
1. Fazer rir;
2. Protestar contra o excesso de mensagens sem nexo, "pegadinhas", problemas
irrelevantes e mal-formulados e, mais geralmente, spam, que certos
participantes insistem em enviar para a lista.

Espero ter obtido um modico de sucesso em cada um deles.

E para que nao haja duvidas:
NAO! EU NAO ACHO QUE O TEOREMA SOBRE A INFINITUDE DOS PRIMOS DEVA SER
REVOGADO! E TAMBEM NAO ACHO QUE TEOREMAS PROVADOS NO TEMPO DA CAROCHINHA OU
ANTES DISSO DEVAM SER NEGLIGENCIADOS - MUITO PELO CONTRARIO, ESTES SAO EM
GERAL OS MAIS BONITOS!

No mais, ainda estou pensando no conteudo da minha lista de verdade.
 
Este triste episodio me fez lembrar de um ex-presidente do nosso pais de
quem se dizia que ria 3 vezes de cada piada: a primeira, quando ouvia; a
segunda, quando explicavam; e a terceira, quando entendia.

Alias, dizem que esse mesmo governante em certa ocasiao envolveu-se em
acalorada discussao com seus ministros sobre o ponto de ebulicao da agua.
Ele insistia que a agua fervia a 90 graus. Por mais de 1 hora, os ministros
tentaram de todas as formas convence-lo que nao, que a agua fervia a 100
graus. Fracassaram. Entretanto, alguns dias depois, em nova reuniao, o
presidente dirigiu-se aos ministros e disse: "Senhores: eu lhes devo
desculpas. Os senhores estao corretissimos. A agua de fato ferve a 100
graus. Eu me confundi. O que ferve a 90 graus eh o angulo reto."

Por fim, quero falar um pouco da mensagem do Nicolau. O episodio ocorrido
com o Noga Alon mostra que a vida real pode ser pelo menos tao absurda
quanto a ficcao. Alem disso, noto, com satisfacao, que a discussao gerada em
torno do problema das tres portas resultou em pelo menos dois participantes
da lista se convencendo de que a atitude correta eh trocar de porta. Eu
chamo isso de progresso. Um passo a mais para longe do obscurantismo!


Um grande abraco a todos,
Claudio.

=
Instruções para entrar na lista, sair da lista e usar a lista em
http://www.mat.puc-rio.br/~nicolau/olimp/obm-l.html
=


[obm-l] Re: [obm-l] Re: [obm-l] DÚVIDA... ajudarr

2003-08-14 Por tôpico Henrique Patrício Sant'Anna Branco
> > 2) Num triângulo retângulo, a hipotenusa é o triplo de um dos catetos.
> > Considerando   o ângulo oposto ao menor lado, podemos afirmar que tg   +
> sec
> >   é igual a  ?
>
> Essa é meio difícil de explicar... Tente fazer o desenho de um triângulo
> retângulo, use trigonometria e o fato de que sec(x) = 1/cos(x).
> Assim, sec(x) + tan(x) = 4B/A.

Esqueci de dizer o que são A e B... A é o cateto adjacente a tal ângulo e B
é o cateto oposto (o menorzinho).

=
Instruções para entrar na lista, sair da lista e usar a lista em
http://www.mat.puc-rio.br/~nicolau/olimp/obm-l.html
=


Re: [obm-l] Problema das 3 portas

2003-08-14 Por tôpico Bernardo Vieira Emerick
Oi Duda,

Já fiz um pedido de desculpas na lista há um bom tempo, mas parece-me que 
ele ainda não "chegou". Reconheci meu erro e, volto a afirmar, a resposta do 
Claudio está correta. A última mensagem minha que você deve ter visto foi em 
resposta ao Claudio. Eu tinha lido com muita pressa, sem me ater ao 
argumento dele. Quando vi que errei, corrigi-me prontamente.
Um abraço,
Bernardo


From: "Eduardo Casagrande Stabel" <[EMAIL PROTECTED]>
Reply-To: [EMAIL PROTECTED]
To: <[EMAIL PROTECTED]>
Subject: Re: [obm-l] Problema das 3 portas
Date: Tue, 12 Aug 2003 15:57:32 -0300
Oi Bernardo.

Por favor, leia a última mensagem enviado por Camilo Marcantonio Junior,
onde ele explica corretamente o problema. Há muitas pessoas que,  mesmo
depois de ler os argumentos que justificam que é melhor TROCAR DE PORTA, 
não
se convencem e continuam a insistir que tanto faz trocar ou não trocar de
porta. Posso lhe assegurar que a resposta correta (todos as pessoas sérias
dessa lista, grandes matemáticos: Nicolau, Gugu, Morgado, Luis Lopes, 
Shine,
Camilo, Paulo Santa Rita, etc. concordarão que o melhor é trocar de porta) 
é
esta.

Se você, ainda sim, não conseguir compreender o argumento, lhe sugiro para
fazer o seguinte experimento. Se você souber programar em computador, faça
um programa que escolha aleatoriamente uma dentre três opções (a premiada) 
e
lhe pede para decidir uma delas (1, 2 ou 3). Depois ele mostra que um dos
números que você escolheu não contém o prêmio. Por fim, ele diz se você
ganha permanecendo na mesma porta ou se trocando de porta. E ele faz uma
contagem. Repita este jogo, umas 100 vezes e você perceberá que em
aproximadamente 67 casos você teria ganho TROCANDO de porta e em
aproximadamente 33 casos você ganharia PERMANECENDO com a mesma porta. Isto
tem de lhe convencer.

Se você não souber programar, sugiro que pegue três copos (não
transparentes) e uma bolinha de papel que é o prêmio. Peça para alguém ter 
a
função do apresentador do programa, e vá você mesmo fazendo a contagem que
lhe sugeri. Repita umas 100 vezes o jogo, e constate a proporção
(aproximada) de 2/3 para 1/3.

Mas faça mesmo essa experiência, antes de enviar uma outra mensagem à 
lista,
ok?

Abraço,
Duda.
From: "Bernardo Vieira Emerick" <[EMAIL PROTECTED]>
> Claudio,
> Eu insisto que tanto faz trocar de porta. Pensemos no problema em duas
> etapas. Na primeira você escolhe entre três portas. Atrás de uma está o
> prêmio. A probabilidade de você ganhar será de 1/3, certo? Na segunda,
você
> tem que escolher entre duas portas. O prêmio está em uma delas. A sua
> probabilidade de ganhar será, portanto, 1/2 para as duas portas. Pouco
> importa o que você escolheu na primeira etapa. É como se fosse outro 
jogo,
> só que se tenha eliminada uma das opções erradas.
>
>
> >From: Claudio Buffara <[EMAIL PROTECTED]>
> >Reply-To: [EMAIL PROTECTED]
> >To: <[EMAIL PROTECTED]>
> >Subject: [obm-l] Problema das 3 portas
> >Date: Tue, 12 Aug 2003 00:43:58 -0300
> >
> >Oi, Henrique:
> >
> >Eu insisto que a estrategia otima eh trocar de porta.
> >
> >Veja o meu raciocinio:
> >
> >Chame as 3 portas de A, B e C.
> >Suponha s.p.d.g. que inicialmente voce escolhe a porta A.
> >
> >Temos 3 casos a considerar:
> >1) O premio estah atras de A:
> >Nesse caso, o apresentador abre B ou abre C (qualquer uma das duas
estarah
> >vazia)
> >Se voce trocar, voce estarah saindo da porta vencedora e indo para uma
das
> >perdedoras (a que ele nao abriu) - voce perde se trocar.
> >
> >2) O premio estah atras de B:
> >Nesse caso, o apresentador abre a porta C.
> >Se voce trocar, voce estarah saindo de A e indo para B - a porta
vencedora.
> >Ou seja, voce ganha se trocar.
> >
> >3) O premio estah atras de C:
> >Nesse caso, o apresentador abre a porta B.
> >Se voce trocar, voce estarah saindo de A e indo para C - a porta
vencedora.
> >Ou seja, voce ganha se trocar.
> >
> >Assim, ao decidir trocar voce perde em um caso e ganha em 2. Supondo 
que
a
> >probabilidade do premio estar atras de uma dada porta eh 1/3, a sua
> >probabilidade de ganhar ao trocar eh igual a 2/3 > 1/2. Logo, voce deve
> >trocar de porta.
> >
> >Com 1 milhao de portas, a decisao eh ainda mais obvia, pois se voce nao
> >trocar, o que voce estarah dizendo eh que voce escolheu a porta certa 
de
> >primeira, um evento que pra voce tem uma probabilidade de 1 em 10^6.
> >
> >Suponha que voce tenha escolhido inicialmente a porta no. 1, a qual 
tem,
> >pra
> >voce, probabilidade de 1/10^6 de conter o premio.
> >Isso quer dizer que, pra voce, a probabilidade do premio estar atras de
uma
> >das outras 999.999 portas eh de 999.999/10^6.
> >
> >Quando o apresentador abre 999.998 portas dentre as 999.999 que voce 
nao
> >escolheu, ele estah colapsando a probabilidade de cada porta aberta 
para
0,
> >e concentrando a probabilidade total de 999.999/10^6 numa unica porta,
que
> >permanece fechada (estas probabilidades sao sempre do seu ponto de 
vista.
> >Do
> >ponto de vista do apresentador, que sabe qual a porta premiad

Re: [obm-l] Re: [obm-l] ENQUETE - BELEZA MATEMÁTICA

2003-08-14 Por tôpico Claudio Buffara
Title: Re: [obm-l] Re: [obm-l] ENQUETE - BELEZA MATEMÁTICA



Eu vou dar mais um tempinho pra ver se alguem mais quer enviar sua lista e ai publico os enunciados, talvez ateh separados por assunto, como no Proofs from the Book. Jah as demonstracoes sao outra historia - e voluntarios serao muito bem-vindos. Eu me disponho a provar que ha infinitos primos (ou pelo menos mais de 200, o que jah eh um belo resultado parcial). Que tal?

Um abraco,
Claudio.

on 13.08.03 15:58, Domingos Jr. at [EMAIL PROTECTED] wrote:

E então, quando veremos todos esses belos teoremas/problemas listados e demonstrados/resolvidos?
Seria muito legal ter tudo isso reunido em formato eletrônico...
 
[ ]'s
 
- Original Message - 
From: Cláudio (Prática)   
To: [EMAIL PROTECTED] 
Sent: Wednesday, August 13, 2003 3:05 PM
Subject: [obm-l] ENQUETE - BELEZA MATEMÁTICA
[EMAIL PROTECTED];s+ [EMAIL PROTECTED],Ak9Yt{T~tkTtO'T~UI/'E4YDwl Ea2c/G xYR +c%w= [EMAIL PROTECTED],ucz]*sYXF]fcuXGuc)Y2 ]ffu]2*uc/uc+]cW]2T]l+]f1ucvuX]WFu+]X*u*]W/]WcY+A]c*YWvY!Y2,ucbucwY*F]fcu*{YWb]2+]2]2TY]WZYcYWcY*2]*u,Y*+]X*W]fZ]WF]2 uXcu*]WZY*W]c+Y],ucz]*sYXF]fcuXGuc)Y2 ]ffu]2*uc/uc+]cW]2T]l+]f1ucvuX]WFu+]X*u*]W/]WcY+A]c+YW,Y*z:[EMAIL PROTECTED]@[EMAIL PROTECTED]@ 







Re: [obm-l] geometria

2003-08-14 Por tôpico Fábio Dias Moreira
-BEGIN PGP SIGNED MESSAGE-
Hash: SHA1

Em Wednesday 13 August 2003 09:36, elton francisco ferreira escreveu:
> A a´rea de um triangulo de perímetro 54m circunscrito
> a um círcuo de 25pim^2, em m^2 e´?

Seja I o incentro do triângulo ABC e M, N, P as projeções de I sobre os lados 
a, b e c. Então ([XYZ] quer dizer a área do polígono XYZ)

[ABC] = [ABI] + [BCI] + [CAI] = (AB * PI + BC * MI + CA * NI)/2 = (AB + BC + 
CA)/2 * r = p * r

onde p é o semiperímetro de ABC e r é o seu inraio. Logo, para qualquer 
triângulo ABC, [ABC] = p * r.

Note que isso também vale para tetraedros -- se 6s é a área superficial do 
tetraedro, e r é o raio da esfera inscrita, então [ABCD] = s * r.

[]s,

- -- 
Fábio "ctg \pi" Dias Moreira
-BEGIN PGP SIGNATURE-
Version: GnuPG v1.2.2 (GNU/Linux)

iD8DBQE/OlWealOQFrvzGQoRAig0AJ4oGeU3EUEdpRkM3vvTzehGo0UvxACfZmtH
JkXabnn5p58DWsHCcYSARFo=
=rtRo
-END PGP SIGNATURE-

=
Instruções para entrar na lista, sair da lista e usar a lista em
http://www.mat.puc-rio.br/~nicolau/olimp/obm-l.html
=


[obm-l] OBRIGADO Questões Esaex - RETA FINAL

2003-08-14 Por tôpico João
 2) QUESTÃO DESGRAÇADA!

Eu também achei um valor parecido a 8%, mas lá da 7,98% sendo que tem o 8%
nas alternativas tb!
Mas a sua conta está diferente da minha!
Porque vc multiplicou as taxas ao invés de somar os dois períodos e dividir
por 2?

> Seja I a desvalorização média. Então
>
> (1 - 0,1)^6*(1 - 0,06)^6 = (1 - I)^12
>
> 1 - I = sqrt(0,9 * 0,94)
>
> I = 8,02% a.m.

Nessa questão eles usam uma série de Taylor e essa resposta não bate. ( NA
VERDADE, É DIFÌCIL SABER O Q ELES QUEREM EM ALGUMAS QUESTÕES! )

> > 6) Expandir a função f(x) = 1 / (1 + x2) em potências de x-2,
determinando
> > o intervalo de convergência da série obtida.
> > [...]
>
> Note que (1 - x^2 + x^4 - x^6 + x^8 - ...) = 1 / (1 + x^2), pois é uma
soma de
> PG. Não está claro para mim onde entra o x^-2, mas note que se a série
> converge, então a PG converge, logo -1 < -x^2 < 1 <==> -1 < x < 1.
>
NESSA QUESTÃO ELES USAM FÓRMULA DE MOIVRE e a eq é Z^n - 1 = 0
> > 8) Mostre que a soma de TODAS as raízes da equação Zn – 1 = 0, no
conjunto
> > dos números complexos é zero.
> > [...]
>
> Isso é falso para n = 1. Supondo n >= 2 natural, basta usar que a soma das
> raízes é -0/1 = 0 por Girard.
>
> > [...]

> > 14) A área do triângulo cujos vértices são, o centro da esfera
3x2+3y2+3z2
> > – 6x+12y+3 = 0, a origem do espaço e o ponto (1,0,0) é ( COMO ACHO O
CENTRO
> > DA ESFERA???)
> > [...]
>
> Complete os quadrados (ou seja, escreva todas as expressões da forma x^2 +
2ax
> como (x+a)^2 - a^2)
Vou "tentar" a da esfera e te digo o que encontrei!

MUITO OBRIGADO MESMO!

=
Instruções para entrar na lista, sair da lista e usar a lista em
http://www.mat.puc-rio.br/~nicolau/olimp/obm-l.html
=


Re: [obm-l] ENQUETE - BELEZA MATEMATICA

2003-08-14 Por tôpico Frederico Reis Marques de Brito
Prezado Cláudio e demais colegas da lista,
Quero desculpar-me por ter interpretado mal sua mensagem. Em realidade 
cometo um erro frequente de ler as mensagens na ordem em que elas estão na 
minha caixa, desta forma leio as mais recentes primeiro. Desta forma li a 
resposta de um membro da lista sobre sua mensagem original, mas que continha 
apenas o primeiro trecho de sua mensagem, o que me levou a uma interpretação 
incorreta e me deixou meio perplexo, minha impressão sobre o Cláudio foi 
sempre bastante positiva. Com este embróglio, prometo ler os e-mails na 
ordem correta. Aproveito para solidariezar-me com o Cláudio e outros que tb 
já não aguentam mais ficar lendo pseudo-paradoxos horripilantes e 
quebra-cabeças supostamente lógicos...

Um abraço a todos.

Frederico.


From: Claudio Buffara <[EMAIL PROTECTED]>
Reply-To: [EMAIL PROTECTED]
To: Lista OBM <[EMAIL PROTECTED]>
Subject: [obm-l] ENQUETE - BELEZA MATEMATICA
Date: Wed, 13 Aug 2003 09:44:57 -0300
Caros colegas:

Foi com uma mistura de surpresa e horror que eu constatei que alguns
participantes da lista interpretaram literalmente o que eu escrevi em minha
ultima mensagem sobre o assunto em epigrafe, do qual alias sou 
patrocinador.
O que me entristece eh que um deles ja havia me causado uma impressao muito
positiva por sua disposicao de tirar duvidas e de mostrar um pouco da 
beleza
que existe na matematica.

Sendo assim, o que me resta fazer eh esclarecer que aquela mensagem tinha
dois objetivos:
1. Fazer rir;
2. Protestar contra o excesso de mensagens sem nexo, "pegadinhas", 
problemas
irrelevantes e mal-formulados e, mais geralmente, spam, que certos
participantes insistem em enviar para a lista.

Espero ter obtido um modico de sucesso em cada um deles.

E para que nao haja duvidas:
NAO! EU NAO ACHO QUE O TEOREMA SOBRE A INFINITUDE DOS PRIMOS DEVA SER
REVOGADO! E TAMBEM NAO ACHO QUE TEOREMAS PROVADOS NO TEMPO DA CAROCHINHA OU
ANTES DISSO DEVAM SER NEGLIGENCIADOS - MUITO PELO CONTRARIO, ESTES SAO EM
GERAL OS MAIS BONITOS!
No mais, ainda estou pensando no conteudo da minha lista de verdade.

Este triste episodio me fez lembrar de um ex-presidente do nosso pais de
quem se dizia que ria 3 vezes de cada piada: a primeira, quando ouvia; a
segunda, quando explicavam; e a terceira, quando entendia.
Alias, dizem que esse mesmo governante em certa ocasiao envolveu-se em
acalorada discussao com seus ministros sobre o ponto de ebulicao da agua.
Ele insistia que a agua fervia a 90 graus. Por mais de 1 hora, os ministros
tentaram de todas as formas convence-lo que nao, que a agua fervia a 100
graus. Fracassaram. Entretanto, alguns dias depois, em nova reuniao, o
presidente dirigiu-se aos ministros e disse: "Senhores: eu lhes devo
desculpas. Os senhores estao corretissimos. A agua de fato ferve a 100
graus. Eu me confundi. O que ferve a 90 graus eh o angulo reto."
Por fim, quero falar um pouco da mensagem do Nicolau. O episodio ocorrido
com o Noga Alon mostra que a vida real pode ser pelo menos tao absurda
quanto a ficcao. Alem disso, noto, com satisfacao, que a discussao gerada 
em
torno do problema das tres portas resultou em pelo menos dois participantes
da lista se convencendo de que a atitude correta eh trocar de porta. Eu
chamo isso de progresso. Um passo a mais para longe do obscurantismo!

Um grande abraco a todos,
Claudio.
=
Instruções para entrar na lista, sair da lista e usar a lista em
http://www.mat.puc-rio.br/~nicolau/olimp/obm-l.html
=
_
MSN Messenger: converse com os seus amigos online.  
http://messenger.msn.com.br

=
Instruções para entrar na lista, sair da lista e usar a lista em
http://www.mat.puc-rio.br/~nicolau/olimp/obm-l.html
=


Re: [obm-l] Questões Esaex - RETA FINAL

2003-08-14 Por tôpico Fábio Dias Moreira
-BEGIN PGP SIGNED MESSAGE-
Hash: SHA1

Em Tuesday 12 August 2003 14:01, João escreveu:
> [...]
> 1) Seja:
>
> Z1 = 2eiπ/6Z2 = (1 + i/1 – i)15ln Z3 = 1 +
> i(π/3 + 2kπ)
>
>
>
> Então o valor de (Z1 +  Z2)/e  . Z3  é:
> [...]

z1 = 2 cis 30
z2 = [(1+i)^2]^7/[(1-i)^2]^7 * (1+i/1-i) = (2i/-2i)^7 * (1+i/1-i) = i+1/i-1 = 
2i/2 = i

ln z3 = 1 + i(60 + 360k) <==> z3 = e^(1+i(60+360k)) = e * cis 60

Logo

(z1 + z2)*z3/e = (2 cis 30 + i)*e*cis 60/e = (sqrt(3) + i + i)(1 + 
i*sqrt(3))/2 = (5i - sqrt(3))/2

[]s,

- -- 
Fábio "ctg \pi" Dias Moreira
-BEGIN PGP SIGNATURE-
Version: GnuPG v1.2.2 (GNU/Linux)

iD8DBQE/OZ54alOQFrvzGQoRAv/5AJ9Ro0/i0PW+IEkNeOcLM36r3iVuvQCdHDd9
Zt3OEAdJ9tpy3aluZwkSyfM=
=1W0Z
-END PGP SIGNATURE-

=
Instruções para entrar na lista, sair da lista e usar a lista em
http://www.mat.puc-rio.br/~nicolau/olimp/obm-l.html
=


Re: [obm-l] ENQUETE - BELEZA MATEMATICA

2003-08-14 Por tôpico Johann Peter Gustav Lejeune Dirichlet
Uma das mensagens do Shine expressa bem isso.

From: "Frederico Reis Marques de Brito" <[EMAIL PROTECTED]>
Reply-To: [EMAIL PROTECTED]
To: [EMAIL PROTECTED]
Subject: Re: [obm-l] ENQUETE - BELEZA MATEMATICA
Date: Tue, 12 Aug 2003 16:30:00 -0300
Cláudio:

" A Matemática é a única ciência em que uma geração não poe abaixo o que a 
anterior construiu. "

As idéias matemáticas belas são permanentes e  a sua enquete em nada 
remetia a que enviássemos apenas teoremas "modernosos", ademais a 
genialidade dos "Matemáticos da Carochinha"  parece não ter mais espaço nos 
últimos anos ...

Perplexo, com seu comentário,

Frederico.


From: Johann Peter Gustav Lejeune Dirichlet 
<[EMAIL PROTECTED]>
Reply-To: [EMAIL PROTECTED]
To: [EMAIL PROTECTED]
Subject: Re: [obm-l] ENQUETE - BELEZA MATEMATICA
Date: Tue, 12 Aug 2003 15:15:20 -0300 (ART)

Ah e,e??Veja o primeiro capitulo do
Proofs from THE BOOK.Ai ce vai ver como topologia
e ultrapassado
> Claudio Buffara wrote:
> > Estou extremamente decepcionado com as listas
> de problemas supostamente
> > bonitos que foram enviadas pra lista ateh o
> presente momento. Imaginem soh -
> > teorema do valor intermediario, secoes
> conicas, poliedros regulares,
> > conjuntos enumeraveis. Onde voces estao com a
> cabeca? Isso tudo eh
> > matematica do tempo da carochinha. Infinitude
> dos primos? Isso eh tao velho
> > que ja devia ter sido revogado!
>
> O Noga Alon conta que fizeram esta pergunta
> para ele uma vez que ele
> começou explicando a prova de Euclides de que
> há infinitos primos
> em um programa de televisão, eu acho:
>
> And today, are there still infinitely many
> primes?
>
> E sem sair do clima, deem uma olhada em
>
http://qsilver.queensu.ca/~phil158d/intro/montyh3.htm
>
> Eu deveria ter visto isso antes de escrever o
> meu artigo da Eureka!
>
> []s, N.
>
=
> Instruções para entrar na lista, sair da lista
> e usar a lista em
>
http://www.mat.puc-rio.br/~nicolau/olimp/obm-l.html
>
=
___
Conheça o novo Cadê? - Mais rápido, mais fácil e mais preciso.
Toda a web, 42 milhões de páginas brasileiras e nova busca por imagens!
http://www.cade.com.br
=
Instruções para entrar na lista, sair da lista e usar a lista em
http://www.mat.puc-rio.br/~nicolau/olimp/obm-l.html
=
_
MSN Hotmail, o maior webmail do Brasil.  http://www.hotmail.com
=
Instruções para entrar na lista, sair da lista e usar a lista em
http://www.mat.puc-rio.br/~nicolau/olimp/obm-l.html
=
_
MSN Messenger: converse com os seus amigos online.  
http://messenger.msn.com.br

=
Instruções para entrar na lista, sair da lista e usar a lista em
http://www.mat.puc-rio.br/~nicolau/olimp/obm-l.html
=


[obm-l] Re: [obm-l] ENQUETE - BELEZA MATEMÁTICA

2003-08-14 Por tôpico Henrique Patrício Sant'Anna Branco
Cláudio,

A classica prova de Euclides é aquela que diz:
Sejam p1, p2, ..., pm todos os primos. Entao consideremos o número N = p1 *
p2 * ... * pm + 1. Esse número não seria divisível por nenhum primo e,
portanto, contradiz o Teorema Fundamental da Aritmetica?

Abraços,
Henrique.

- Original Message - 
From: "Claudio Buffara" <[EMAIL PROTECTED]>
To: <[EMAIL PROTECTED]>
Sent: Wednesday, August 13, 2003 6:08 PM
Subject: Re: [obm-l] Re: [obm-l] ENQUETE - BELEZA MATEMÁTICA


Eu vou dar mais um tempinho pra ver se alguem mais quer enviar sua lista e
ai publico os enunciados, talvez ateh separados por assunto, como no Proofs
from the Book. Jah as demonstracoes sao outra historia - e voluntarios serao
muito bem-vindos. Eu me disponho a provar que ha infinitos primos (ou pelo
menos mais de 200, o que jah eh um belo resultado parcial). Que tal?

=
Instruções para entrar na lista, sair da lista e usar a lista em
http://www.mat.puc-rio.br/~nicolau/olimp/obm-l.html
=


[obm-l] Re: [obm-l] DÚVIDA... ajudarr

2003-08-14 Por tôpico Henrique Patrício Sant'Anna Branco
Não tenho muita certeza das minha resoluções, mas já que ninguém respondeu,
vai aí alguma tentativa...

> O número natural N = 180p, onde p é um número primo, possui 27 divisores
> naturais. O valor de p é:
> a) 2 b) 3 c)5 d) 7 e) 11

Creio que aqui seja N = 180^p, uma vez que 180p é divisível por {2 2 3 3 5
p}.
De qualquer forma, não sei resolver essa questão, mas creio que seja fácil
usar congruências (as quais não domino). Eu mesmo gostaria de uma ajuda
nesse problema.

> 2) Num triângulo retângulo, a hipotenusa é o triplo de um dos catetos.
> Considerando   o ângulo oposto ao menor lado, podemos afirmar que tg   +
sec
>   é igual a  ?

Essa é meio difícil de explicar... Tente fazer o desenho de um triângulo
retângulo, use trigonometria e o fato de que sec(x) = 1/cos(x).
Assim, sec(x) + tan(x) = 4B/A.

> 4) Um Aspirante ganhou, em uma competição na Escola Naval, quatro livros
> diferentes de Matemática, três livros diferentes de Física e dois livros
> diferentes de Português.  Querendo manter juntos aqueles da mesma
> disciplina, concluiu que poderia enfileirá-los numa prateleira de sua
> estante, de diversos modos.  A quantidade de modos com que poderá fazê-lo
é
>
> (A)48
> (B 72
> (C192
> (D864
> (E)1728

Ordenando cada grupo, temos para os de Matemática 24 = 4*3*2 maneiras de
dispô-los entre si, para os de Física 6 = 3*2 maneiras e para os de
Português, 2 maneiras.
Agora, consideremos cada conjunto de livros como um único, facilita.
Portanto, se tivermos 3 grupos diferentes, podemos ordená-los de 3*2 = 6
formas.
Portanto, o total de modos que esse estudante pode organizar os livros na
estante é 24*6*2*6 = 1728, letra E.

Abraços,
Henrique.

=
Instruções para entrar na lista, sair da lista e usar a lista em
http://www.mat.puc-rio.br/~nicolau/olimp/obm-l.html
=


Re: [obm-l] Re: [obm-l] ENQUETE - BELEZA MATEMÁTICA

2003-08-14 Por tôpico Claudio Buffara
on 13.08.03 20:28, Henrique Patrício Sant'Anna Branco at
[EMAIL PROTECTED] wrote:

> Cláudio,
> 
> A classica prova de Euclides é aquela que diz:
> Sejam p1, p2, ..., pm todos os primos. Entao consideremos o número N = p1 *
> p2 * ... * pm + 1. Esse número não seria divisível por nenhum primo e,
> portanto, contradiz o Teorema Fundamental da Aritmetica?
> 
> Abraços,
> Henrique.
>
Eh isso ai mesmo.

=
Instruções para entrar na lista, sair da lista e usar a lista em
http://www.mat.puc-rio.br/~nicolau/olimp/obm-l.html
=


[obm-l] Re: [obm-l] OUTRAS Questões Esaex

2003-08-14 Por tôpico Henrique Patrício Sant'Anna Branco
>4) NÃO CONSIGO FATORAR A RAIZ CÚBICA!
>O valor do limite quando x → 0 de


4√(x+1) + 3√(x+1) + √(x+1) – 3


√(x+1) – 1

Com x = 0, vamos chegar a uma indeterminação do tipo 0/0 e, assim, podemos
usar L'Hopital.
Temos derivando f(x) = (x+1)^(1/4)+(x+1)^(1/3)+(x+1)^(1/2)-3 e g(x) =
(x+1)^(1/2)-1, temos
f'(x) = 1/(4*(x+1)^(3/4))+1/(3*(x+1)^(2/3))+1/(2*(x+1)^(1/2)) e g'(x) =
1/(2*(x+1)^(1/2)). Colocando f'(x)/g'(x) e fazendo as simplicações
necessárias, temos:
f'(x)/g'(x) = (3+4*(x+1)^(1/12)+6*(x+1)^(1/4))/(x+1)^(1/4). Agora fazendo x
= 0, chegamos a 13/6.
Portanto, limite de ((x+1)^(1/4)+(x+1)^(1/3)+(x+1)^(1/2)-3)/((x+1)^(1/2)-1)
com x -> 0 é 13/6.

Abraços,
Henrique.

=
Instruções para entrar na lista, sair da lista e usar a lista em
http://www.mat.puc-rio.br/~nicolau/olimp/obm-l.html
=


[obm-l] Re: [obm-l] ENQUETE - BELEZA MATEM ÁTICA

2003-08-14 Por tôpico Henrique Patrício Sant'Anna Branco
> Por falar nisso, esta prova aparentemente induz um dos erros mais comuns.
> As pessoas incorretamente entendem que foi provado que 2*3*...*p + 1 é
primo.
> Isto é falso mas o primeiro contraexemplo demora o suficiente para
aparecer
> para convencer os mais afoitos de que sim, estes números são primos:
(...)

Nicolau,
Foi muito válida sua observação. No caso, a primeira vez que vi essa
demonstração foi no seu "livro virtual", sobre primos de Mersenne e "outros
primos muito grandes".
Lá você cita isso também, com esses contra-exemplos. Como você cita lá: "Não
existe nenhuma fórmula simples conhecida que gere sempre números primos".

Aproveitando a mensagem, não quero causar nenhum tipo de polêmica, mas acho
que os participantes da lista deviam tomar mais cuidado com certas sutilezas
em seus e-mails. Por exemplo, tentem diminuir a linha de "Assunto". Como
ilustração, a mensagem "ENQUETE - BELEZA MATEMÁTICA", depois de sucessivos
"replys", virou "Re: [obm-l] Re:
[obm-l]_Re:_[obm-l]_ENQUETE_-_BELEZA_MATEMÁTICA", com esses tantos Re:
[obm-l] repetidos.
Tentem também dar uma chance aos expoentes fracionários ao invés de usar os
próprios sinais de fraçao. Raiz cubica de 2 pode ser escrita como 2^(1/3).
Para facilitar a visualização.
São apenas sugestões para tornar a leitura dos e-mails mais simples.

Abraços,
Henrique.

=
Instruções para entrar na lista, sair da lista e usar a lista em
http://www.mat.puc-rio.br/~nicolau/olimp/obm-l.html
=


Re: [obm-l] DÚVIDA

2003-08-14 Por tôpico okakamo kokobongo
Não é mais uma pegadinha!? 

mc = tc/f=1,2

mc1=tc/.8f=1,2/.8=4/3

Abracos,
  Okakamo Kokobongo

Valeu Morgado!
 --- [EMAIL PROTECTED] escreveu: > Olá! Pessoal,
Gostaria da ajuda de vocês no
> probleminha abaixo:
> 
> Uma revista afirma que a média de carros por família
> assinante é de 1,2. Se 20% 
> das famílias assinantes não têm carro, qual é o
> número médio de carros, por 
> família, das famílias que têm um ou mais carros?
> 
> 
> Obrigado pela atenção!
> 
> 
> 
> 
> WebMail UNIFOR - http://www.unifor.br
>
=
> Instruções para entrar na lista, sair da lista e
> usar a lista em
> http://www.mat.puc-rio.br/~nicolau/olimp/obm-l.html
>
= 

___
Yahoo! Mail
O melhor e-mail gratuito da internet: 6MB de espaço, antivírus, acesso POP3, filtro 
contra spam. 
http://br.mail.yahoo.com/
=
Instruções para entrar na lista, sair da lista e usar a lista em
http://www.mat.puc-rio.br/~nicolau/olimp/obm-l.html
=


Re: [obm-l] ENQUETE - BELEZA MATEMATICA

2003-08-14 Por tôpico Paulo Jose Rodrigues
Não venho acompanhando a discussão sobre o problema das 
portas (Conhecido com Dilema de Monty Hall), mas gostaria de 
lembrar que Paul Erdös, um dos grandes matemáticos do último 
século, ao escutar de Vázsonyi o problema e a resposta (mudar 
de porta), discordou e demorou a aceitar a solução. 

Na verdade, só se convenceu quando Graham lhe disse: "A chave 
para o problema de Monty Hall era saber desde o início que o 
apresentador dava sempre a hipótese de mudar de porta. Isso é 
uma parte das regras do jogo que é preciso interiorizar".

Vejam o livro O Homem que Só Gostava de Números, da Gradiva.

 
Paulo

 
---
Acabe com aquelas janelinhas que pulam na sua tela.
AntiPop-up UOL - É grátis! 
http://antipopup.uol.com.br

=
Instruções para entrar na lista, sair da lista e usar a lista em
http://www.mat.puc-rio.br/~nicolau/olimp/obm-l.html
=


Re: [obm-l] Fwd: Fabrício sem avião 2

2003-08-14 Por tôpico joao dias

At 22:18 12/8/2003 -0700, you wrote:

De
outro professor da UFC: "Na verdade o pior de tudo foi que a
UFC e a Secitece não disseram não ao Fabrício, mas ficaram protelando, e
quando resolveram já era tarde demais... Se o NÃO tivesse sido avisado à
tempo, a Sociedade Brasileira de Matemática, através da Olimpíada
Brasileira de Matemática, provavelmente teria dado a passagem. Vale
salientar que esta é a primeira vez que o Brasil participou da IMC
(International Mathematics Competition)". 
    Olimpíada de Maio
1997    medalha de bronze
1998    medalha de bronze
Olimpíada do Cone Sul
1998   medalha de prata
1999   medalha de prata
Olimpíada Iberoamericana
1999   medalha de bronze
2000   medalha de ouro
Olimpíada Internacional de Matemática
1999   medalha de bronze
2000   medalha de bronze
Olimpíada Brasileira nível 3
1997  medalha de prata
1998  medalha de ouro
1999  medalha de prata
2000  medalha de prata
Olimpíada Brasileira nível universitário
2001  medalha de prata
2002  medalha de prata
Histórico invejável de um olímpico que desde 1997  até os dias de
hoje  "medalhou" em todas as olimpíadas que
participou.
Aprovado na seletiva para a IMC.
O Brasil perdeu a participação deste  brilhante aluno, independente
de qualquer resultado que por ele fosse obtido .
Não acredito que haja um único contribuinte do dinheiro público que não
considerasse justa a aplicação do seu imposto na sua viagem.
Não é mais possível corrigir a tremenda injustiça com o aluno
talentoso,resta a esperança de que este fato jamais se repita.
[]s,
joaodias.


[obm-l] Re: [obm-l] Re: [obm-l] ENQUETE - BELEZA MATEM ÁTICA

2003-08-14 Por tôpico Nicolau C. Saldanha
On Wed, Aug 13, 2003 at 09:39:49PM -0200, Claudio Buffara wrote:
> on 13.08.03 20:28, Henrique Patrício Sant'Anna Branco at
> [EMAIL PROTECTED] wrote:
> 
> > Cláudio,
> > 
> > A classica prova de Euclides é aquela que diz:
> > Sejam p1, p2, ..., pm todos os primos. Entao consideremos o número N = p1 *
> > p2 * ... * pm + 1. Esse número não seria divisível por nenhum primo e,
> > portanto, contradiz o Teorema Fundamental da Aritmetica?
> > 
> > Abraços,
> > Henrique.
> >
> Eh isso ai mesmo.

Por falar nisso, esta prova aparentemente induz um dos erros mais comuns.
As pessoas incorretamente entendem que foi provado que 2*3*...*p + 1 é primo.
Isto é falso mas o primeiro contraexemplo demora o suficiente para aparecer
para convencer os mais afoitos de que sim, estes números são primos:
2*3*5*7*11*13+1 = 30031 = 59*509
2*3*5*7*11*13*17+1 = 510511 = 19*97*277
2*3*5*7*11*13*17*19+1 = 9699691 = 347*27953
2*3*5*7*11*13*17*19*23+1 = 223092871 = 317*703763
Também dá errado se trocarmos +1 por -1 no final:
2*3*5*7-1 = 209 = 11*19
2*3*5*7*11*13*17-1 = 510509 = 61*8369
2*3*5*7*11*13*17*19-1 = 9699689 = 53*197*929
2*3*5*7*11*13*17*19*23-1 = 223092869 = 37*131*46027

Desculpem, eu sei que ninguém perguntou, mas eu já ouvi este erro
vezes demais.

[]s, N.


=
Instruções para entrar na lista, sair da lista e usar a lista em
http://www.mat.puc-rio.br/~nicolau/olimp/obm-l.html
=


Re: [obm-l] Questões Esaex - RETA FINAL

2003-08-14 Por tôpico Fábio Dias Moreira
-BEGIN PGP SIGNED MESSAGE-
Hash: SHA1

Em Tuesday 12 August 2003 23:12, Fábio Dias Moreira escreveu:
> [...]
> z2 = [(1+i)^2]^7/[(1-i)^2]^7 * (1+i/1-i) = (2i/-2i)^7 * (1+i/1-i) = i+1/i-1
> = 2i/2 = i
> [...]

Correção: i+1/i-1 = 2i/-2 = -i, e não i. Assim a resposta fica

(sqrt(3) + i - i)/e * e * cis 60 = sqrt(3) * cis 60 = sqrt(3)/2 + 3i/2.

[]s,

- -- 
Fábio "ctg \pi" Dias Moreira
-BEGIN PGP SIGNATURE-
Version: GnuPG v1.2.2 (GNU/Linux)

iD8DBQE/OwYcalOQFrvzGQoRAqd4AKDlUux/xcRLMdi0xj6xmk7diaH5/QCg10HE
PHSlVuIww0UgMdS/HBAyBAQ=
=asVr
-END PGP SIGNATURE-

=
Instruções para entrar na lista, sair da lista e usar a lista em
http://www.mat.puc-rio.br/~nicolau/olimp/obm-l.html
=


Re: [obm-l] problemas

2003-08-14 Por tôpico Aleandre Augusto da Rocha
A opcao correta (se eu tiver acertado logicamente) esta  marcada com #.  O
desenvolvimento esta abaixo de cada problema.

-Auggy

- Original Message -
From: "elton francisco ferreira" <[EMAIL PROTECTED]>
To: <[EMAIL PROTECTED]>
Sent: Thursday, August 14, 2003 10:09 AM
Subject: [obm-l] problemas


>
> O número de anagramas formados com as letras da
> palavra ROMA de modo que não apareça vogais ou
> consoantes juntas é igual a:
>
> 4!
> 4
> 8 #
> 2

So existem 2 formatos possiveis, VCVC ou CVCV

Em cada caso temos 2 vogais que podem ocupar 2 espacos e 2 consoantes que
podem ocupar 2 espacos, ou seja 2*2 = 4 cominacoes possiveis... 4 por caso =
8 total

>
> um atirador deu 49 tiros, pagando 10 de multa por tiro
> fora do alvo e recebendo 0,25 de premio pr tiro
> acertado no alvo. Se nada recebeu e nada pagou, então
> a multa foi de:
>
> 1,40
> 3,50 #
> 5,00
> 8,75

Aki eh onde o Ilmo Sr Prof Morgado implica.  O enunciado esta errado.  Mas
imaginando que a multa na verdade era de 0,10 entao fica assim (A = tiros
acertados, F= tiros fora do alvo)

0,25*A - 0,10*F = 0
A + F   = 49

resolvendo o sistema A=14 e F=35, logo 0,10*F = 3,50

>
> Se de um retangulo de perímetro 4 e dimensões ``x`` e
> ``y`` , X então a área remanescente em função de ``x`` é:
>
> 1 - 2x
> x - 2x^2
> 2x - 2x^2 #
> 2x - 4x^2

2x + 2y = 4 ==> y = 2 - x

x(y - x) = x(2 - x - x) = 2x -2x^2

>
> Uma das raízes da equação 2x^3 + x^2 - 7x - 6 = 0  é
> x1= 2. Pode-se afirmar que:
>
> As outras raízes são imaginário puro
> As outras raízes são -3 e -2
> Só uma das outras raízes é real
> As outras raízes estão entre -2 e 0 #

Como 2 e raiz, basta fatorar
2x^3 + x^2 - 7x - 6 = (x-2)(2x^2 + 5x + 3)
resolvendo a equacao de 2o grau x2=-1 e x3=-1.5

>
>
> ___
> Conheça o novo Cadê? - Mais rápido, mais fácil e mais preciso.
> Toda a web, 42 milhões de páginas brasileiras e nova busca por imagens!
> http://www.cade.com.br
> =
> Instruções para entrar na lista, sair da lista e usar a lista em
> http://www.mat.puc-rio.br/~nicolau/olimp/obm-l.html
> =
>


=
Instruções para entrar na lista, sair da lista e usar a lista em
http://www.mat.puc-rio.br/~nicolau/olimp/obm-l.html
=


[obm-l] Enquete da Beleza

2003-08-14 Por tôpico Johann Peter Gustav Lejeune Dirichlet
Oi turmaVejam o teorema da Corda Quebrada de
Arquimedes.
Considere um arco AC de ponto medio D.No arco CD
que nao contem A marque um ponto B.E e a projeçao
ortogonal de D em AB.Prove que AE=EB+BC.

Outro otimo,do Erdös e Szekeres,da eureka
6,artigo do Gugu:
Para qualquer n natural existe um numero f(n) tal
que entre f(n) pontos do plano em posiçao geral
ha n pontos formando um poligono convexo.

___
Conheça o novo Cadê? - Mais rápido, mais fácil e mais preciso.
Toda a web, 42 milhões de páginas brasileiras e nova busca por imagens!
http://www.cade.com.br
=
Instruções para entrar na lista, sair da lista e usar a lista em
http://www.mat.puc-rio.br/~nicolau/olimp/obm-l.html
=


[obm-l] Re: [obm-l] Re: [obm-l]_Re:_[obm-l]_ENQUETE_-_BELEZA_MATEMÁTICA

2003-08-14 Por tôpico Nicolau C. Saldanha
On Thu, Aug 14, 2003 at 02:54:19PM -0300, Johann Peter Gustav Lejeune Dirichlet wrote:
> A demonstraçao que 3eu escrevi evitara este
> mal-entendido.
> Alias o Tengan me disse que este e um problema em
> aberto muito chato e de que ninguem conseguiu uma
> ideia muito esperançosa...

Desculpe, mas qual exatamente é o problema em aberto?
Talvez decidir se 2*3*5*...*p + 1 é primos infinitas vezes?

[]s, N.
=
Instruções para entrar na lista, sair da lista e usar a lista em
http://www.mat.puc-rio.br/~nicolau/olimp/obm-l.html
=


Re: [obm-l] [univ] Teoria dos Grupos

2003-08-14 Por tôpico Nicolau C. Saldanha
On Thu, Aug 14, 2003 at 02:39:47PM -0300, Eduardo Casagrande Stabel wrote:
> Olá pessoal!
> 
> [Agradeço ao Nicolau pela solução enviada... teorema de Baire era o mais
> natural...]
> 
> Uma questão de álgebra que não estou conseguindo resolver, do livro de
> introdução a álgebra do Hernstein.
> 
> QUESTÃO. Um grupo abeliano finito possui dois subgrupos, um de ordem N e
> outro de ordem M. Mostre que ele trambém possui um subgrupo de ordem MMC{ M,
> N }.
> 
> Eu consigui resolver a questão no caso particular em que o grupo é cíclico.
> No caso geral, eu pensei em usar o produto de subgrupos MN, mas a ordem pode
> ser maior que MMC{ M, N }. O interessante é que unindo esta questão ao
> teorema de Silow para grupos abelianos, acho que se demonstra a existência
> de subgrupos de qualquer ordem divisora da ordem do grupo original. Este
> resultado não é forte demais?

Não. Se G é um grupo abeliano finito e h é um divisor de |G| então existe
um subgrupo H de G com |H| = h. O que aliás resolve trivialmente a sua
questão: se N e M são divisores de |G| então mmc{M,N} é divisor de |G|.
O fato que eu enunciei acima é corolário do teorema de classificação
de grupos abelianos finitos. E é completamente falso para grupos não abelianos.

[]s, N.
=
Instruções para entrar na lista, sair da lista e usar a lista em
http://www.mat.puc-rio.br/~nicolau/olimp/obm-l.html
=


[obm-l] Re: [obm-l] Questões Esaex - RETA FINAL

2003-08-14 Por tôpico Ricardo Knop



 Entendi a questão desta maneira:
 
 Suponha a paridade do real com o 
dólar:
 
  R$1 -> US$1
  Com as desvalorizações da moeda,a taxa de 
câmbio naturalmente aumenta:
  Então devemos ter:
  (1.10^6).(1.06^6) = 2.512993...
  Isto significa que após as 
desvalorizações precisamos aproximadamente de R$2.52 para comprar  um  
dólar.
  Elevando 2.512993...  a (1/12),acha-se 
1.0798147
  Isto significa que uma desvalorização 
constante de aproximadamente 7,98% a.m da moeda,durante um  
ano,equivale a desvalorização de 10% nos primeiros 6 meses e de 6% nos 
6 meses restantes.
 
[]s
 
Ricardo

  - Original Message - 
  From: 
  João 
  To: [EMAIL PROTECTED] 
  Sent: Tuesday, August 12, 2003 2:01 
  PM
  Subject: [obm-l] Questões Esaex - RETA 
  FINAL
  
   
  
  2) QUESTÃO 
  DESGRAÇADA!
  Num 
  determinado país, em um ano de transição, a sua moeda foi desvalorizada frente 
  ao dólar em 10% ao mês nos primeiros 6 meses e nos 6 meses seguintes em 6% ao 
  mês. Neste ano a desvalorização média mensal foi de:      
     
  GABARITO: 7,98% am
   


[obm-l] Re:[obm-l] Re: [obm-l] Questões Esaex - RETA FINAL 4 e 5

2003-08-14 Por tôpico amurpe
> 
Os simbolos estão com problema, por favor reenvie a msg.

Um abraço.Amurpe



>   1) Seja:
> 
>   Z1 = 2eiπ/6Z2 = (1 + i/1 – i)
15ln Z3 = 1 + i(π/3 + 2kπ)
> 
>
> 
>   Então o valor de (Z1 +  Z2)/e  . Z3  é:
> 
>   R: minha resposta deu diferente da do fabio ... como 
ninguem falou nada ate agora ! a minha deve ta errada!
> 
>
> 
> 
> 
>   4) NÃO CONSIGO FATORAR A RAIZ CÚBICA!
> 
>   O valor do limite quando x → 0 de
> 
>
  4√(x+1) + 3√(x+1) + √(x+1) – 3
> 
>
√(x+1) – 1
> 
>
> 
> 
> 
>   Resposta: 
> 
>   Fazendo  (x+1) = y^12 , como x->0  ,   y->1.
> 
>(y^3 + y^4 + y^6 - 3)/(y^6 - 1) ,  [(y^3 -
 1) + (y^4 -1) + (y^6 - 1)]/[(y^3 + 1)(y^3 -1)]
> 
>eleminando o fator (y-
1), nao existira mais  a indeterminacao ! 
> 
>
> 
>
> 
> 
> 
>   5) Determinar lim   ln (1 +  _2_  )2x – 1/3  
> 
>  n→∞x+3

> 
> 
> 
>   Resposta:
> 
>   n->∞
> 
>   ln[lim (1 + 2/(x+3))^(2x - 1/3)], lim (1 + 2/(x+3))^
(x+3)*[(2x -1/3)/(x+3)  Elevando a (x+3)/(x+3)
> 
>   temos o limite fundamental { n->∞ }  (1 + k/x)
^x = e^k
> 
>   (e^2)^lim(2x -1/3)/(x+3) => (e^2)
^2 , logo ln e^4  eh 4.
> 
>
> 
> 
> 
>
> 
> 

 
__
Acabe com aquelas janelinhas que pulam na sua tela.
AntiPop-up UOL - É grátis!
http://antipopup.uol.com.br/


=
Instruções para entrar na lista, sair da lista e usar a lista em
http://www.mat.puc-rio.br/~nicolau/olimp/obm-l.html
=


Re: [obm-l] Produto de Cossenos

2003-08-14 Por tôpico Johann Peter Gustav Lejeune Dirichlet
Acho que o do Lidsky que ele fala e o problema da
IMO,
cos pi/7-cos 2*pi/7+cos 3*pi/7=?

 --- Claudio Buffara
<[EMAIL PROTECTED]> escreveu: >
Thyago:
> 
> Para um produto de senos de numeros em PA, eu
> acho que a sua solucao eh a
> melhor.
> 
> No entanto, se o produto for de cossenos de
> numeros em PG da razao 2, ai a
> coisa muda de figura...
> 
> P = cos(a)cos(2a)cos(4a)...cos(2^na) ==>
> 
> sen(a)P =
> sen(a)cos(a)cos(2a)cos(4a)...cos(2^na) =
> 
> = (1/2)sen(2a)cos(2a)cos(4a)...cos(2^na) =
> 
> = (1/4)sen(4a)cos(4a)...cos(2^na) =
> 
> = (1/8)sen(8a)cos(8a)...cos(2^na) =
> 
> ...
> 
> = (1/2^n)sen(2^na)cos(2^na) =
> 
> = (1/2^(n+1))sen(2^(n+1)a)
> 
> Logo: P = sen(2^(n+1)a)/(2^(n+1)sen(a))
> 
> 
> Serah que era esse o problema do Lidski que
> voce procurava?
> 
> 
> Um abraco,
> Claudio.
> 
> on 12.08.03 21:07, Thyago at [EMAIL PROTECTED]
> wrote:
> 
> > Olá Claudio e companheiros da lista
> > 
> > Bom, sabe que estou me convencendo mesmo que
> esta solução é prática :-)
> > 
> > O que eu estava querendo inicialmente é uma
> solução que nem a da questão
> > abaixo, veja só:
> > 
> > S = sen(a) + sen(2a) + sen(3a) + ... +
> sen(na)
> > 
> > Em que a solução consiste em multiplicar
> ambos os lados da igualdade pelo
> > seno da metade da razão da PA, e após efetuar
> a prostaférese e sair
> > cortando. Sem muitas delongas!
> > ...
> > 
> > Já ouvi dizer que a resolução que procuro
> existe, e está escrita em um tal
> > livro russo chamado "Lidski, problemas de
> PA", ou algo do gênero... mas
> > nunca tive o privilégio de ter algum contato
> com essa obra. Alguém já ouviu
> > falar?
> > 
> > Atenciosamente
> > ¡Thyago!
> > 
> > 
> > 
> > - Original Message -
> > From: Claudio Buffara
> <[EMAIL PROTECTED]>
> > To: <[EMAIL PROTECTED]>
> > Sent: Tuesday, August 12, 2003 9:58 AM
> > Subject: Re: [obm-l] Ajuda
> > 
> > 
> >> Oi, Thyago:
> >> 
> >> Vou te confessar uma coisa: usando a
> identidade 1 - cis(a) =
> >> -2isen(a/2)cis(a/2) e mais esse problema do
> IME, que alias eh uma
> >> propriedade classica (e, como voce mostrou,
> util!) das raizes n-esimas da
> >> unidade, voce chegou a uma solucao mais
> curta e elegante do que a que eu
> >> tinha em mente. Parabens!
> >> 
> >> A minha ideia era separar os casos n par e n
> impar e fatorar x^n - 1 de
> > duas
> >> maneiras diferentes:
> >> Primeiro:
> >> x^(2m) - 1 = (x^2 - 1)*(x^(2m-2) + x^(2m-4)
> + ... + x^4 + x^2 + 1)
> >> x^(2m+1) - 1 = (x - 1)*(x^(2m) + x^(2m-1) +
> ... + x^2 + x + 1)
> >> 
> >> Depois:
> >> x^(2m) - 1 = (x^2 -
> 1)*PRODUTO(1<=k<=m-1)(x^2 - 2xcos(kpi/m)x + 1)
> >> x^(2m+1) - 1 = (x - 1)*PRODUTO(1<=k<=m)(x^2
> - 2xcos(2kpi/(2m+1)) + 1)
> >> 
> >> E depois, fazer x = 1 e igualar as
> expressoes obtidas, mas a sua solucao
> > eh
> >> mais simples e, portanto, melhor.
> >> 
> >> O passo que faltou na sua solucao foi
> mostrar explicitamente que
> >>
>
(-i)^(m-1)*cis(pi/n)*cis(2pi/n)*...*cis((n-1)pi/n)
> = 1
> >> mas isso eh bem facil (apesar de nao ser
> evidente).
> >> 
> >> Um abraco,
> >> Claudio.
> >> 
> >> PS: Se essa sua solucao nao eh "pratica",
> entao eu nao sei o que eh.
> > Repare:
> >> voce tem um produto de senos de numeros em
> PA. Como voce propoe
> > calcula-los?
> >> Puramento por meio de identidades
> trigonometricas, sem usar complexos? Boa
> >> sorte...
> >> 
> >> on 12.08.03 00:45, Thyago at [EMAIL PROTECTED]
> wrote:
> >> 
> >>> Olá Cláudio,
> >>> 
> >>> Obrigado pelas dicas  :-)
> >>> 
> >>> Mas a resolução que eu fiz não foi nada
> prática não.
> >>> Eu já utilizei todas estas propriedades e
> não consegui chegar em nada.
> >>> Bom, só para esclarecer um pouco mais...
> vou colocar o exercício que
> > gerou
> >>> tal questão:
> >>> 
> >>> 
> >>> (IME) Sejam 1, X2, X3, ..., Xn as raízes de
> x^n=1. Calcule: P = (1 -
> >>> x2)(1-x3)...(1-xn).
> >>> 
> >>> Fazendo uso de Briot-Rufini e fatoração de
> polinômios, conseguimos
> > chegar
> >>> facilmente na resposta P = n.
> >>> Mas, utilizando o tratamento vetorial de
> números complexos com a fórmula
> >>> 1-cis(a) = -2isen(a/2)cis(a/2) chegamos em
> >>> 
> >>> P = 2^(n-1) . S
> >>> 
> >>> Onde S = sen(pi/n) . sen(2pi/n) .
> sen(3pi/n) . ... . sen[(n-1)pi/n]
> >>> 
> >>> Daí, utilizando a resposta da primeira
> resolução com a resposta da
> > segunda
> >>> resolução temos que S = n/[2^(n-1) ]
> >>> Dá para ver que esta demonstração para S
> não é nada prática.
> >>> 
> >>> Você citou uma "solução padrão" para este
> tipo de problema. Qual seria?
> >>> 
> >>> Aguardo resposta
> >>> 
> >>> Atenciosamente
> >>> ¡Thyago!
> >>> 
> >>> - Original Message -
> >>> From: Cláudio (Prática)
> <[EMAIL PROTECTED]>
> >>> To: <[EMAIL PROTECTED]>
> >>> Sent: Monday, August 11, 2003 2:19 PM
> >>> Subject: Re: [obm-l] Ajuda
> >>> 
> >>> 
>  Oi, Thyago:
>  
>  A solução "padrão" pra esse tipo de
> problema realmente envolve
> > complexos e
>  polinômios.
>  
>  Tentando resolver outros problemas
> similares, você vai perceber que
>  complexos 

Re: [obm-l] ENQUETE - BELEZA MATEMATICA

2003-08-14 Por tôpico Helder Suzuki
Temos duas escolhas, portanto temos 4 casos:
1- escolhemos a porta premiada, e em seguida trocamos
2- escolhemos a porta premiada e não trocamos
3- não escolhemos a porta premiada e trocamos
4- não escolhemos a porta premiada e não trocamos

vamos analisar os casos em que sempre trocamos de
porta:
(i) se escolhemos a porta premiada, perdemos.
p = 1/3
(ii) se escolhemos uma porta não-premiada, ganhamos.
p = 2/3

vamos analisar os casos em que não trocamos de porta:
(i) se escolhemos a porta premiada, ganhamos:
p = 1/3
(ii) se escolhemos uma porta não-premiada, perdemos:
p = 2/3

veja que se trocamos de porta, temos uma chance de 2/3
~= 66% de ganharmos
e se não trocarmos de porta, temos uma chance de ~33%
de ganharmos.
então trocando de porta, aumentamos a chance de
ganhar.

[]'s,
HTS
 --- Bernardo Vieira Emerick
<[EMAIL PROTECTED]> escreveu: > Que piada!!!
Marylin vos Savant, tida como a pessoa
> com o maior QI do mundo 
> (concordo com o Domingos Jr.: bulsshit!) confundiu
> tudo. O problema era 
> assim: num jogo, a pessoa escolha uma entre três
> portas. O apresentador, 
> então, abra uma das portas. Como ele sabe qual é a
> porta que contém o 
> prêmio, ele abre uma que não o contém - já que o
> jogo dar-se-ia por 
> encerrado. A pergunta é: o jogador deveria trocar de
> porta?
> Segundo Marylin, sim!, porque a probabilidade da
> opção que ele teria 
> continuaria 1/3, enquanto a outra aumentaria para
> 2/3!!! Qual a razão disso? 
> A probabilidade da porta que ele escolheu não
> poderia subir subitamente para 
> 1/2, como sugerem os matemáticos. Ora, como então a
> outra porta pode??? Isso 
> ela não explica.
> Ela aparentemente desconhece o conceito primeiro de
> probabilidade, que é a 
> chance de se acertar, e por isso está atrelada ao
> número de possibilidades 
> possíveis e o número de possibilidades "requeridas"
> para se acertar o 
> resultado. Então, a probabilidade será dada - como é
> de conhecimento geral, 
> exceto possivelmente de Marylin - pela divisão do
> número de possibilidades 
> "requeridas" pelo número total de possibilidades.
> Parece-me que ela acredita 
> que a única forma de se aumentar a probabilidade é
> aumentando o número de 
> possibilidades "requeridas". Isso justificaria o
> "we've learned nothing to 
> allow us to revise the chances on the shell under
> your finger" que ela diz. 
> O que mudou, e que ela incrivelmente não percebeu, é
> o número total de 
> possibilidades. Simplificando para ela, o
> numerozinho de baixo diminuiu, 
> então o número do outro lado do sinal de igualdade
> aumentou, já que o 
> numerozinho de cima da fração permaneceu constante.
> Será que assim ela 
> entenderia???

___
Conheça o novo Cadê? - Mais rápido, mais fácil e mais preciso.
Toda a web, 42 milhões de páginas brasileiras e nova busca por imagens!
http://www.cade.com.br
=
Instruções para entrar na lista, sair da lista e usar a lista em
http://www.mat.puc-rio.br/~nicolau/olimp/obm-l.html
=


Re: [obm-l] Re: [obm-l] Cochilo na aula de algebra

2003-08-14 Por tôpico Henrique Patrício Sant'Anna Branco
> Todas as raízes são iguais a 2. De fato, se as raízes são x_1, x_2,...,
> x_10, então pelas relações de Girard temos:
>   x_1 + x_2 + ... +  x_10= 20
>   x_1.x_2...x_10= 1024
>  Como as raízes são reais positivas, podemos usar MA >= MG:
> (x_1 + x_2 + ... +  x_10)/10 >= (x_1.x_2...x_10)^(1/10) =>
> 20/10 >= (1024)^(1/10) => 2 >= 2
>  Como ocorre a igualdade, devemos ter que todos os x_i´s são iguais, logo
> 10.x_1=20 => x_1= x_2=...= x_10= 2.

Yuri,
Tá certo, a solução é muito boa... Mas como você pensou em usar MA e MG? Já
conhecia o problema (ou algum parecido?)
E se, no caso, as médias fossem diferentes? Não daria pra sair daí?
Desculpe pela dúvida um tanto quanto "idiota", mas não custa perguntar...

Grato,
Henrique.

=
Instruções para entrar na lista, sair da lista e usar a lista em
http://www.mat.puc-rio.br/~nicolau/olimp/obm-l.html
=


RE: [obm-l] Matematica contra-intuitiva

2003-08-14 Por tôpico Artur Costa Steiner










Oi Faelc,

Eu nao conheco este asssunto
em profundidade, mas me parece que o paradoxo de Banach-Tarski deve ser olhado
com cuidado. O conceito de volume a que ele se refere nao eh exatamente o
conceito usual, segundo o qual o volume de um cubo de lado L eh L^3 e o de uma  esfera de raio r eh 4 PI R^3/3. O volume
do qual trata o paradoxo, segundo  o
que li, estah ligado a um conceito matematico mais avancado, conhecido por
medida de um conjunto. Isto eh estudado em Teoria de Medidas, a qual eh ligada
aa integral de Lesbegue. Nbao eh assunto que se estude em cursos de Engenharia,
por exemplo,

Artur

 

Ola pessoal, 

De todos os teoremas que vcs postaram ateh agora o que me chamou mais atencao e
eh extremamente contra-intuitivo eh o teorema abaixo. Pois uma esfera com
volume V e massa M sendo
fragmentada e depois reagrupada para depois se transformar em outra c/ volume
V*f (sendo f um fator de aumento de grandeza) e massa
M soh eh possivel se a esfera maior for oca, nao eh isso ? Mas acho que o
teorema diz que a esfera (V, M) se transformarah em outra esfera (V*f , M*f). A
passagem passagem 
V-> V*f eh extremamente aceitavel, mas acho que o paradoxo vem da passagem 
M->M*f, pois eh contra as proprias leis da Fisica. De onde o corpo pegaria
emprestado esta massa. Acho que
este emprestimo de massa o paradoxo
nao permite, nao eh isso ? Por favor me esclarecam, pois adoro paradoxos e este
me chamou a atencao. 














Re: [obm-l] ENQUETE - BELEZA MATEMATICA

2003-08-14 Por tôpico Paulo Santa Rita
Ola Carissimo Prof Morgado e demais
colegas desta lista ... OBM-L,
Este principio e devido a Erdos, que o usou pela primeira vez. No livro :

Proofs from the Book
Martin Aigner e Gunter Ziegler
Springer Verlag
os autores tambem o creditam a Alfred Renyi, mas eu nao conheco nenhuma 
artigo deste Matematico e, muito menos, claramente, alguma aplicacao 
especifica deste principio por ele. Evidentemente que o a regra e uma forma 
original de pensar, usando probabilidades num dominio onde dificilmente 
imaginariamos que ela pudesse ser util. Deve-se destacar que serve tao 
somente para provas de existencia.

O livro "Proofs from the book" mostra diversas aplicacoes deste metodo 
probabilistico e, em particular, aos numeros de Ramsey ( Alias, o Teorema de 
Ramsey e um dos resultados elementares que eu citaria, mas e um assunto 
nunca ou quase nunca abordado no Nivel Medio - injustificadamente, ao meu 
ver ).

Exemplo ( de Proofs from the book )

Seja Y um conjunto. Toda funcao f:Y -> {a,b}  e chamada uma "pintura" dos 
pontos de Y. Se y esta em Y e f(y) =a dizemos que o ponto "y" foi pintado 
com a cor "a". Claramente que existem pinturas possiveis para Y.

Considere agora um conjunto F cujos elementos sao subconjuntos de Y, isto e, 
F e uma familia de subconjunto de Y. Nos dizemos que F e "BICOLORIZAVEL" ou 
"2-COLORIZAVEL" se dentre todas as pinturas de Y com duas cores ( as funcoes 
f mencionadas acima ) EXISTE UMA tal que em todo subconjunto de Y 
pertencente a F poderemos encontrar dois elementos pintados com cores 
diferentes.

1) Claramente que se F e 2-colorizavel entao todos os subconjunto de F sao 
2-colorizavel : basta usar a mesma funcao f:Y->{a,b} que demonstrou a 
carater 2-colorizavel de F.

Agora, seria toda familia de subconjuntos de Y 2-colorizavel ? Claramente 
que Nao !

2) Com efeito, seja Y um conjunto com 2d+1 elementos. Toda pintura de f, 
f:Y->{a,b}, pode ser vista como uma distribuicao de 2 elementos ( "a" e "b" 
) nos 2d+1 "lugares" que sao os pontos de Y e, portanto, pelo principio da 
casa dos pombos, haverao ao menos "d" pontos de Y com a mesma cor ! Assim, 
se tomarmos F como o CONJUNTO DE TODOS OS SUBCONJUNTOS de Y com d elementos, 
QUALQUER QUE SEJA  a pintura f:Y -> {a,b} havera um subconjunto de Y com d 
elementos
( elemento de F ) com todos os seus pontos pintados da mesma cor ! Logo, F 
nao sera 2-colorizavel.

Os fatos 1) e 2) sao extremos. Eles tornam claro que para todo "d" existe um 
extremante ... Isto e, dado d, qualquer subconjunto de Y com d elementos e 
2-colorizavel, evidentemente. Todos os subconjutos de Y com d elementos e 
uma familia que nao e 2-colorizavel, conforme vimos acma. Entao, para cada 
d, tem sentido perguntarmos : qual e O MENOR NUMERO DE SUBCONJUNTOS DE Y COM 
d ELEMENTOS que nao e 2-colorizavel ?

TEOREMA ( FENOMENO ) : Toda familia F de subconjuntos de Y com no maximo 
2^(d-1) elementos e 2-colorizavel

Assim, o numero que estamos procurando, que por homenagem a Erdos chamaremos 
de E(d), e tal que E(d) > 2^(d-1)

DEMONSTRACAO ( EXPLICACAO ) : IMAGINE que todas as pinturas f:Y->{a,b} 
possiveis sao eventos igualmente provaveis ... Assim, a toda pintura que 
ocorrer, estaremos concomitantemente colorindo todos os elementos de cada 
subconjunto de F, evidentemente. Vamos agora escolher um elemento de F, isto 
e, um subconjunto de Y pertencente a F. Qual sera a probabilidade de os os 
seus elementos estejam pintados com uma unica cor ?

Claramente que P = (1/2)^(d-1),  pois existem apenas duas maneiras dos 
elementos do subconjunto escolhido estarem pintados com uma mesma cor. 
Aplicando este mesmo raciocinio a todos os elementos da familia e notando 
que o evento de escolher um conjuntos 1-colorizavel nao e mutuamente 
exclusivo com as outras ocorrencias e, portanto, a probabilidade e inferior 
a mera soma das probabilidades, termos :

E(d)*((1/2)^(d-1)) =< 1  => E(d) =< 2^(d-1)

Logo, pelo principio de Erdos, existira uma pintura de Y sem sub-conjunto de 
F  pintado com uma cor somente, que e o que queriamos demonstrar !

Essa e uma aplicacao trivial do Principio de Erdos. No livro "Proof from the 
Book" existe aplicacoes mais interessantes. Alias, este livro e altamente 
acessivel a qualquer estudante serio e dedicado do nivel medio, pedindo 
pouquissimos pre-requisitos "mais avancados" , todos de facil assimilacao e 
facilmente encontraveis.

E digno de nota que este principio e muito criticado por muitas pessoas, que 
nao aceitam um resultado derivado da sua utilizacao. Ele e o "Axioma da 
Escolha" do momento. Falta surgir o Godel do momento...

Digo isso porque em face das inumeras criticas que o axioma da escolha 
sofria, Godel mostrou que se a Teoria do conjuntos COM O AXIOMA DA ESCOLHA e 
inconsistente e contraditoria, ENTAO,  a teoria dos conjuntos SEM O AXIOMA 
DA ESCOLHA, e igualmente inconsistente e contraditoria, isto e, o Axioma da 
Escolha, em que pese as inumeras consequencias pouco verossimeis que COM ELE 
podemos deduzir, se muito,

[obm-l] Descida de fermat...

2003-08-14 Por tôpico DEOLIVEIRASOU
Prove que a equação x^4+4y^4=z^2, não possui soluções inteiras não nulas...
   Obrigado, 
   Crom


[obm-l] Moedas em caixas

2003-08-14 Por tôpico Claudio Buffara
Title: Moedas em caixas



Oi, Alexandre:

Eu achei esse problema das moedas em caixas mais interessante do que o do no. de solucoes da equacao, onde a matematica "legal" acaba no momento em que voce estabelece a relacao entre o no. de solucoes de uma equacao e os coeficientes de um certo polinomio associado aquela equacao. Depois eh soh braco (ou de preferencia, transistor).

O das moedas eh uma aplicacao bem legal da representacao binaria de numeros naturais.

No caso de 10 caixas e 1000 moedas, a k-esima caixa tem que ter 2^(k-1) moedas, para k = 1, 2, ..., 9.  Total das 9 primeiras caixas = 1 + 2 + 4 + ... + 128 + 256 = 511 moedas.
A 10a. caixa contera as 489 moedas restantes.

Usando as 9 primeiras caixas, voce consegue pegar qualquer numero de moedas entre 1 e 511 (inclusive) - pra ver isso basta reparar que qualquer numero tem uma (unica) representacao binaria (ou seja, como soma de potencias de 2).

Pra pegar 512 ou mais moedas, voce usa a 10. caixa e depois uma combinacao das 9 primeiras que contenha de 1 a 511 moedas. Por exemplo, 836 = 489 + 256 + 64 + 16 + 8 + 2 + 1 = 489 + 2^(9-1) + 2^(7-1) + 2^(5-1) + 2^(4-1) + 2^(2-1) + 2^(1-1) ==> pra pegar 836 moedas voce vai usar as caixas 1, 2, 4, 5, 7, 9 e 10.

O problema geral eh o seguinte:
1) Prove que com caixas contendo 1, 2, 4, ..., 2^(n-1) moedas, voce consegue pegar um numero qualquer de moedas entre 1 e 2^n - 1 (inclusive).
2) Prove que essa eh a unica forma de distribuir 2^n - 1 moedas por n caixas em que isso eh possivel.

Um abraco,
Claudio.


on 10.08.03 00:26, Alexandre Daibert at [EMAIL PROTECTED] wrote:

Não, isto não caiu em vestibular nenhum, eu cheguei nisto no meio de um problema, q eh o seguinte:
Tendo 10 caixas e 1000 moedas, colocar as caixas nas moedas de modo q qualquer quantidade de 1 a 1000 moedas possam ser pegas de modo a não abrir nenhuma caixa. Se não me engano este problema eh do homem q calculava. tem uma solução mais usual para ele q eh ir colocando 1, 2, 4, 8, ... moedas em cada caixa, e no fim as q sobrarem colocar na última caixa. 

*** De uma checada nas contas: 1+2+4+8+...+256 = 511 ==> sobram 489 moedas.
Esse 23 deve vir do fato de que 2^10 - 1 = 1023 e de que existem apenas 1000 moedas, ou seja, faltam 23 moedas pra completar uma eventual 11a. caixa, que conteria 2^9 = 512 moedas (489+23=512).

Sobram 23 moedas, mas elas não precisam ser colocadas necessariamente apenas na última caixa. Se vc pensar em cima do problema vc chega q o número de soluções do problema é o número de soluções inteiras não negativas da equação: 16a + 8b + 4c + 2d + e = 23. Claro q isto vai além do q se esperava q a pessoa fizesse no problema. Cheguei a este resultado e quis, por curiosidade, saber como calcular o número de soluções deste tipo de equação, visto q o cálculo no braço seria muito trabalhoso. Pensei q houvesse alguma solução por análise combinatória deste problema, porém mais avançada q a resolução clássica da equação a + b + c + d = 10 por exemplo. Mas pelo que eu entendi, este tipo de problema, pelo q vimos ateh aki, mesmo com problemas menores, ou vc calcula todas as soluções no braço mesmo ou joga em um computador. Não há método matemático q seja pouco trabalhoso. Mas mesmo assim gostaria de agradecer imensamente ao colega, pois as suas explicações contribuiram muito para mim.  :-) 

Se alguém quiser a minha resolução deste problema das caixas e como eu cheguei a isso, depois me dê um toque q colocarei minha resolução aki com paciência

Alexandre Daibert








Re: [obm-l] Matematica contra-intuitiva

2003-08-14 Por tôpico Johann Peter Gustav Lejeune Dirichlet
Na minha opiniao o Porisma de poncelet e que e
contra-intuitivo:como e que e que uma coisa tao
bonita pode ter uma demonstraçao tao feia???

Dois problemas que nao resolvi mas acho legais
neste ponto de vista:
1)Existe uma funçao continua apenas nos
racionais?
2)Existe uma funçao continua apenas nos
irracionais?



 --- Claudio Buffara
<[EMAIL PROTECTED]> escreveu: > on
10.08.03 00:50, Artur Costa Steiner at
> [EMAIL PROTECTED] wrote:
> 
> > Aproveito a oportunidade para perguntar:
> Existe alguma conclusao da
> > matematica que vc considere contraria aa
> intuicao? Eu, por exemplo, acho um
> > tanto contra intuitivo que o fato de f ser
> diferenciavel  em R e apresentar
> > limite no infinito nao implique que f'
> apresente limite zero no infinito.
> > Algumas pessoas acham contra intuitivo que a
> serie harmonica seja
> > divergente.
> > Artur
> > 
> Oi, Artur:
> 
> Gostaria de ver que exemplos outras pessoas da
> lista vao dar, mas assim de
> bate-pronto eu diria que acho contra-intuitivo:
> 
> 1) que existam funcoes continuas em toda a reta
> mas sem derivada em nenhum
> ponto;
> 
> 2) o fato de, sendo a irracional, o conjunto {
> m + na ; m, n inteiros } ser
> denso em R;
> 
> 3) que Pi tenha alguma relacao com a soma dos
> inversos dos quadrados dos
> naturais;
> 
> 4) que um problema tao simples como o de 3
> corpos sujeitos a atracao
> gravitacional mutua possa ter uma solucao
> caotica;
> 
> 5) que um conjunto nao enumeravel possa ter
> medida nula;
> 
> 6) que exista uma bijecao entre R e R^2;
> 
> 7) a maioria dos resultados quase-milagrosos de
> analise complexa;
> 
> 8) que R possa ser bem-ordenado e que isso seja
> consequencia de um negocio
> tao intuitivo como o axioma da escolha.
> 
> 9) que o porisma de Poncelet nao possa ser
> provado apenas por geometria
> Euclidiana.
> 
> Mas acho que todos esses sao pinto se
> comparados ao
> 
> 10) paradoxo de Banach-Tarski - voce pode
> decompor uma esfera do tamanho de
> uma ervilha em no maximo 5 pedacos e re-montar
> esses pedacos de modo a
> formar uma esfera do tamanho do Sol
> 
> E com essa, vou dormir...
> 
> Um abraco,
> Claudio.
> 
>
=
> Instruções para entrar na lista, sair da lista
> e usar a lista em
>
http://www.mat.puc-rio.br/~nicolau/olimp/obm-l.html
>
= 

___
Conheça o novo Cadê? - Mais rápido, mais fácil e mais preciso.
Toda a web, 42 milhões de páginas brasileiras e nova busca por imagens!
http://www.cade.com.br
=
Instruções para entrar na lista, sair da lista e usar a lista em
http://www.mat.puc-rio.br/~nicolau/olimp/obm-l.html
=


[obm-l] Análise Combinatória

2003-08-14 Por tôpico Rafael Barcellos
Olá pessoal, estou com dúvida nesta questão e só consegui resolver pelo diagrama da 
arvore, se alguem tiver uma outra resolução, eu agradeço.


Questão:
 Dez balões azuis e oito brancos deverão ser distribuídos em três enfeites de salão, 
sendo que um deles tenha 7 balões e os outros dois, no mínimo 5. Cada enfeite deverá 
ter 2 balões azuis e 1 branco, pelo menos. De quantas maneiras distintas pode-se fazer 
os enfeites usando simultaneamente todos os balões?





[EMAIL PROTECTED]  - o melhor site sobre a EPCAR
epcar.zzn.com - não tem o seu e-mail personalizado da EPCAR? digite: www.epcar.zzn.com 
e tenha o seu
Pegue o seu E-mail grátis em  http://epcar.zzn.com

Obtenha seu Serviço de Correio eletrônico Baseado na  Web Service em http://www.zzn.com
=
Instruções para entrar na lista, sair da lista e usar a lista em
http://www.mat.puc-rio.br/~nicolau/olimp/obm-l.html
=


RE: [obm-l] ENQUETE - BELEZA MATEMATICA

2003-08-14 Por tôpico Artur Costa Steiner
Hah algumas passagens elementares da matematica que acho lindas:


A demonstracao de que, sendo A, B e C conjuntos, entao A inter (B uniao
C) = (A inter B) uniao (A inter C) e de que A uniao (B inter C) = (A
uniao B) inter (A uniao C)

As leis de De Morgan
Como estas relacoes elementares sao importantes ma matematica, em todos
os niveis!

Indo um pouco mais longe, mas ainda de forma acessivel ao nivel medio:
A prova de que interseccoes finitas de conjuntos abertos sao abertas e
de que unioes finitas de conjuntos fechados sao fechadas, mas que tais
constatacoes podem nao valer para interseccoes e unioes infinitas.

A respeito de conjuntos, indo ainda um pouco mais longe, mas ainda ao
alcance de um aluno de nivel medio que realmente goste de matematica
(parece que soa poucos):

O teorema de Heine Borel
O teorema de Cantor Bendixon

E a nivel bem simples:

A deducao da formula de Baskara (hoje, evidente, mas uma grande sacada
para a epoca)
A demonstracao por meios puramente algebricos de que o trinomio do
segundo grau y = ax^2 + bx + c tem um maximo ou um minimo global em x=
-b/2a
A formula da soma dos n primeiros termos de uma PG, a qual permite que
um aluno de nivel medio veja, mesmo sem maiores conhecimentos de
limites, que a serie geometrica converge se |razao|<1 e diverge caso
contrario.
Artur  

=
Instruções para entrar na lista, sair da lista e usar a lista em
http://www.mat.puc-rio.br/~nicolau/olimp/obm-l.html
=


Re: [obm-l] continuidade

2003-08-14 Por tôpico Claudio Buffara
on 10.08.03 20:58, edalbuquerque at [EMAIL PROTECTED] wrote:

> Como eu provo que f(x)=1/x² é contínua?Melhor,como determinar
> o delta apropriado?
> 
> Grato por qualquer ajuda.
> 
> Eder
> 
Oi, Eder:

Devemos ter cuidado pra definir f, pois seu dominio nao contem x = 0.

Seja  a <> 0. Temos que provar que lim(x -> a) 1/x = 1/a.


Seja eps > 0.

Como a <> 0, teremos |a| > |a|/2 > 0

Tomemos delta = min( a^2*eps/2, |a|/2 )

|x - a| < delta ==>

a - delta < x < a + delta ==>

a - |a|/2 < x < a + |a|/2 ==>

se a < 0, entao 3a/2 < x < a/2
e
se a > 0, entao a/2 < x < 3a/2 ==>

de qualquer jeito, |x| > |a|/2 ==> 1/|x| < 1/(|a|/2)

Assim:
|1/x - 1/a| = |x - a|/(|a||x|) < delta/(|a||a|/2) = 2delta/a^2 <= eps


Um abraco,
Claudio.


PS: Acabei nao respondendo a sua pergunta. O delta apropriado voce acha
resolvendo o problema de tras pra frente, ou seja, fazendo:

|1/x - 1/a| = |x - a|/(|a||x|) < delta/(|a||x|) <= eps ==>

delta <= eps*|a|*|x|

A partir desse ponto, voce soh precisa achar um limitante inferior para |x|
(no caso, eu achei |a|/2).



=
Instruções para entrar na lista, sair da lista e usar a lista em
http://www.mat.puc-rio.br/~nicolau/olimp/obm-l.html
=


[obm-l] probleminha

2003-08-14 Por tôpico elton francisco ferreira
José se deslocou entre as cidades A e B tres vezes
pelo mesmo caminho, utilizando, em cada uma das vezes,
um meio de transporte diferente. Na primeira ves foi
de carro, com uma velocidade média de 60 Km/h. Na
segunda vez doi de bicicleta, com velocidade média de
30 Km/h, e na terceira vez foi de moto, com velocidade
média de 40 km/h. Sabendo que a soma dos tempos gastos
nos tres deslocamentos doi igual a 45 h, o tempo gasto
em cada um dos deslocamentos foi, respectivamente:

a) 11h;22h e 12h
b)12h; 25h e 7,5 h
c)10h; 20h e 15h
d)12h; 24h e 9h
e)10,5h;21h e 13,5

___
Conheça o novo Cadê? - Mais rápido, mais fácil e mais preciso.
Toda a web, 42 milhões de páginas brasileiras e nova busca por imagens!
http://www.cade.com.br
=
Instruções para entrar na lista, sair da lista e usar a lista em
http://www.mat.puc-rio.br/~nicolau/olimp/obm-l.html
=


Re: [obm-l] ENQUETE - BELEZA MATEMATICA

2003-08-14 Por tôpico Johann Peter Gustav Lejeune Dirichlet
As demos do Erdös e deo selberg sao bem feias
segundo o que ouvi falar,por exemplo,do
Edmilson.Mas ele me deu uma demo do TNP bem curta
e inteligivel.Depois eu envio.

a demonstraçao com geometria esferica da relaçao
de Euler e mais legal ainda!

 --- Frederico Reis Marques de Brito
<[EMAIL PROTECTED]> escreveu: > Exatamente
Cláudio, o Princípio de Dirichlet tb
> é conhecido como Princípio 
> da Casa dos Pombos ou das gavetas.
> 
> O exemplo do monge é muito bom. Coloquei-o
> certa vez numa prova de cálculo 
> I. Os alunos acharam "bacana". E quanto ao TNP
> a prova não é simples 
> realmente, mas a tentação de mostrar aos alunos
> a relação entre ln  e os nos 
> primos, destas relações absolutaamente
> inesperadas, é forte.
> 
> Aproveito para colocar mais alguns resultados e
> dizer que, a medida que leio 
> as respostas dos nossos colegas a sua enquete,
> fico cada vez mais perplexo, 
> pois raramente discordo de algum, o que me
> alegra por demonstrar que, 
> convenhamos, a Matemática é linda demais.
> 
> (6) Esse é simples e bonitinho demais: Existem
> desertos de primos tão 
> grandes qto se queira, isto é, formalmente:
> dado N natural, existe uma 
> sequência de N inteiros consecutivos compostos.
> 
> (7) A demonstração de que os números
> transcendentes são não-enumeráveis.
> 
> (8) A solução da eq: 2^x = x^2  . ( acho que se
> encontra isto em : Meu 
> Professor de Matemática... , do Elon. SBM )
> 
> (9) A demonstração de que  C  não é um corpo
> ordenado, pela simplicidade. ( 
> Aqui vale dizer que não é necessário o emprego
> dos termos técnicos, como 
> corpo, por exemplo... )
> 
> (10) A relação de Euler para poliedros, que,
> pecaminosamente havia me 
> esquecido.
> 
> Abraços,
> 
> Frederico.
> 
> >From: Claudio Buffara
> <[EMAIL PROTECTED]>
> >Reply-To: [EMAIL PROTECTED]
> >To: <[EMAIL PROTECTED]>
> >Subject: Re: [obm-l] ENQUETE - BELEZA
> MATEMATICA
> >Date: Sat, 09 Aug 2003 20:34:04 -0300
> >
> >on 09.08.03 18:39, Frederico Reis Marques de
> Brito at 
> >[EMAIL PROTECTED]
> >wrote:
> >
> > > 1) Acho que esse será praticamente unânime:
> Teorema de Euclides sobre a
> > > exist~encia de  infinitos primos.
> > >
> > > 2) Teorema de Bezout sobre MDC: O máximo
> dvisor comum de dois inteiros é 
> >uma
> > > comb. linear inteira ( em realidade a menor
> positiva ) desses números ,
> > > pelas várias aplicações deste na Teoria dos
> Números.
> > >
> > > 3) O Princípio de Dirichlet, pela potência
> .
> > >
> > > 4) O posto-linha = posto-coluna. Não sei
> mais sempre achei este 
> >resultado
> > > muito inusitado, já que uma matriz nada
> mais é que "um amontoado" de
> > > números...
> > >
> > > 5) Teorema do Valor Intermediário ( Acho
> que podemos abordar este tema 
> >no
> > > ensino médio... )
> > >
> > > Como o Morgado, pensarei um pouco mais
> antes de enviar outros 5. (  A
> > > propósito é tentador citar o Teorema dos
> Números Primos, mas acho que 
> >esse
> > > tema não seria acessível. Fica um voto de
> louvor então!)
> > >
> > > Frederico.
> > >
> >Oi, Frederico.
> >
> >O principio de Dirichlet a que voce se refere
> eh o das casas de pombos?
> >
> >O TNP eh um pouco avancado demais (mesmo com
> uma demonstracao elementar),
> >mas aquelas desigualdades de Chebichev sao
> aceitaveis, assim como o
> >postulado de Bertrand, que penso seriamente em
> botar na minha lista.
> >
> >Pra mim, o TVI (e qualquer resultado que
> dependa do axioma do supremo) foi
> >um caso mais dificil de decidir, mas como tem
> aquele probleminha do monge
> >subindo e descendo a montanha, acho que ele
> tambem eh aceitavel. E, afinal
> >de contas, tem um volume da colecao do Iezzi
> que trata de limites, 
> >derivadas
> >e integrais...
> >
> >
> >Um abraco,
> >Claudio.
> >
>
>=
> >Instruções para entrar na lista, sair da lista
> e usar a lista em
>
>http://www.mat.puc-rio.br/~nicolau/olimp/obm-l.html
>
>=
> 
>
_
> MSN Hotmail, o maior webmail do Brasil. 
> http://www.hotmail.com
> 
>
=
> Instruções para entrar na lista, sair da lista
> e usar a lista em
>
http://www.mat.puc-rio.br/~nicolau/olimp/obm-l.html
>
= 

___
Conheça o novo Cadê? - Mais rápido, mais fácil e mais preciso.
Toda a web, 42 milhões de páginas brasileiras e nova busca por imagens!
http://www.cade.com.br
=
Instruções para entrar na lista, sair da lista e usar a lista em
http://www.mat.puc-rio.br/~nicolau/olimp/obm-l.html
=


Re: [obm-l] EsSA

2003-08-14 Por tôpico Henrique Patrício Sant'Anna Branco
> > 3 – Sendo x= 19 e y= 81, então a expressão (x+y)^2 +
> > x^2 – y^2 + 2x é divisível por:
> > a)2,19 e 81
> > b)2,19 e 101
> > c)2,81 e 100
> > d)19,100 e 101
> > e)81,100 e 101
> >
> > achei a letra B
>
> (x+y)^2 + x^2 – y^2 + 2x
>
> 100^2 +((x+y)*(x-y)) + 38
>
> 1 + (100*(-62)) + 38
>
> =3838 que eh dividsivel por 101, mas nao por 2,19...2,81...100...
>
> O que fiz de errado ?

Fael,

3838 é divisível por 2 e 19 sim.
3838/2 = 1919/19 = 101/101 = 1
Aliás, segundo minha HP, são os únicos fatores primos desse número... :)
Dá uma olhada na minha solução (desenvolvendo a expressão). Acho mais
simples...

Abraços,
Henrique.

=
Instruções para entrar na lista, sair da lista e usar a lista em
http://www.mat.puc-rio.br/~nicolau/olimp/obm-l.html
=


Re: [obm-l] ENQUETE - BELEZA MATEMATICA

2003-08-14 Por tôpico Bernardo Vieira Emerick
Caros Colegas,

Gostaria de pedir desculpas a todos pelos comentários horrorosos que teci 
sobre Marilyn vos Savant. Primeiro, porque ela acertou o problema que errei, 
e mais, que reiterei meu erro. Cabe aqui um agradecimento especial ao 
Claudio que solucionou corretamente, enquanto eu e o Henrique erramos. Eu, 
por outro lado, errei mais ao tentar ridicularizar alguém que estava correta 
o tempo inteiro. Explico aqui o meu erro, que creio ser o mesmo do Henrique, 
para que outros entendam bem: o problema não permitia que a análise fosse 
feita através de dois jogos diferentes. Se assim fosse, as probabilidades 
das duas portas seriam iguais. Como bem explicou o Claudio, o problema está 
na primeira escolha, porque ela altera a probabilidade do que considerei 
como o segundo jogo. O pior que fiz, e do que agora me arrependo 
profundamente, parece ter sido influenciar a última mensagem do Dirichlet: 
"Colegas,nao acreditem em testes de QI". A ele peço desculpas, em 
especial.
Estou escrevendo esse texto para que outros não façam a mesma bobagem que 
eu. Antes de comentar algo, reflita bem sobre as possibilidades. Eu não fiz 
isso, e olha no que deu. Agora fica claro porque ela tem o QI mais alto do 
mundo (reitero: bullshit!!!), e eu não
Desculpas sinceras, e, desde já, agradeço pela compreensão da minha 
ignorância,
"SAPIENTIAM AUTEM NON VICIT MALITIA",
Bernardo


From: Johann Peter Gustav Lejeune Dirichlet 
<[EMAIL PROTECTED]>
Reply-To: [EMAIL PROTECTED]
To: [EMAIL PROTECTED]
Subject: Re: [obm-l] ENQUETE - BELEZA MATEMATICA
Date: Tue, 12 Aug 2003 14:36:30 -0300 (ART)

Colegas,nao acreditem em testes de QI

 --- Bernardo Vieira Emerick
<[EMAIL PROTECTED]> escreveu: > Que
piada!!! Marylin vos Savant, tida como a
> pessoa com o maior QI do mundo
> (concordo com o Domingos Jr.: bulsshit!)
> confundiu tudo. O problema era
> assim: num jogo, a pessoa escolha uma entre
> três portas. O apresentador,
> então, abra uma das portas. Como ele sabe qual
> é a porta que contém o
> prêmio, ele abre uma que não o contém - já que
> o jogo dar-se-ia por
> encerrado. A pergunta é: o jogador deveria
> trocar de porta?
> Segundo Marylin, sim!, porque a probabilidade
> da opção que ele teria
> continuaria 1/3, enquanto a outra aumentaria
> para 2/3!!! Qual a razão disso?
> A probabilidade da porta que ele escolheu não
> poderia subir subitamente para
> 1/2, como sugerem os matemáticos. Ora, como
> então a outra porta pode??? Isso
> ela não explica.
> Ela aparentemente desconhece o conceito
> primeiro de probabilidade, que é a
> chance de se acertar, e por isso está atrelada
> ao número de possibilidades
> possíveis e o número de possibilidades
> "requeridas" para se acertar o
> resultado. Então, a probabilidade será dada -
> como é de conhecimento geral,
> exceto possivelmente de Marylin - pela divisão
> do número de possibilidades
> "requeridas" pelo número total de
> possibilidades. Parece-me que ela acredita
> que a única forma de se aumentar a
> probabilidade é aumentando o número de
> possibilidades "requeridas". Isso justificaria
> o "we've learned nothing to
> allow us to revise the chances on the shell
> under your finger" que ela diz.
> O que mudou, e que ela incrivelmente não
> percebeu, é o número total de
> possibilidades. Simplificando para ela, o
> numerozinho de baixo diminuiu,
> então o número do outro lado do sinal de
> igualdade aumentou, já que o
> numerozinho de cima da fração permaneceu
> constante. Será que assim ela
> entenderia???
>
>
>
> >From: "Domingos Jr." <[EMAIL PROTECTED]>
> >Reply-To: [EMAIL PROTECTED]
> >To: <[EMAIL PROTECTED]>
> >Subject: Re: [obm-l] ENQUETE - BELEZA
> MATEMATICA
> >Date: Mon, 11 Aug 2003 19:03:11 -0300
> >
> >O Noga Alon conta que fizeram esta pergunta
> para ele uma vez que ele
> >começou explicando a prova de Euclides de que
> há infinitos primos
> >em um programa de televisão, eu acho:
> >
> >And today, are there still infinitely many
> primes?
> >
> >E sem sair do clima, deem uma olhada em
>
>http://qsilver.queensu.ca/~phil158d/intro/montyh3.htm
> >
> >Eu deveria ter visto isso antes de escrever o
> meu artigo da Eureka!
> >
> >--- x ---
> >Putz, essa mulher do QI mais alto do mundo
> (bullshit!) não concorda com o
> >Princípio da Indução Finita também! hehehe, o
> pior é que é sério!!!
> >
>
>=
> >Instruções para entrar na lista, sair da lista
> e usar a lista em
>
>http://www.mat.puc-rio.br/~nicolau/olimp/obm-l.html
>
>=
>
>
_
> MSN Hotmail, o maior webmail do Brasil.
> http://www.hotmail.com
>
>
=
> Instruções para entrar na lista, sair da lista
> e usar a lista em
>
http://www.mat.puc-rio.br/~nicolau/olimp/obm-l.html
>
=

[obm-l] Fwd: Fabrício sem avião 2

2003-08-14 Por tôpico Olimpiada Brasileira de Matematica


Date: Wed, 13 Aug 2003 08:40:44
-0300 (ART)
From: Paulo Rodrigues <[EMAIL PROTECTED]>
Subject: Fabrício sem avião 2
To: [EMAIL PROTECTED]
X-Spam-Status: No, hits=1.8 required=5.0
tests=HTML_00_10,HTML_FONT_BIG,HTML_FONT_COLOR_GRAY,HTML_MESSAGE,
 
HTML_TAG_EXISTS_TBODY
version=2.55
X-Spam-Level: *
X-Spam-Checker-Version: SpamAssassin 2.55
(1.174.2.19-2003-05-19-exp)

Escola privada carimba passaportes

Fábio Campos

[13 Agosto 04h22min]


Certas histórias tocam fundo a alma dos leitores. A do estudante Fabrício
Benevides, o sem-passagem, provocou a indignação de muitos. Fabrício é
aluno da graduação em ciências da Computação (UFC). Passou pelas
acirradas seletivas nacionais e se classificou para representar o Brasil
nas Olimpíadas Internacionais de Matemática, em Bucareste, Romênia. A
disputa já aconteceu, mas Fabrício não concorreu porque sua universidade
e as instituições que criamos para promover o "progresso da
ciência" negaram-lhe uma passagem aérea. A mesquinharia só faz
vítimas entre os que freqüentam os bancos da escola pública. A garotada
"caxias" que estuda nos melhores colégios da gloriosa Aldeota
carimba seu passaporte com a inestimável ajuda do contribuinte cearense.
E os melhores colégios estampam anúncios com o feito. 


OS AVIÕES APENAS SOBREVOAM O PICI 
Quando era aluno de uma escola particular, Fabrício nunca deixou de
viajar para disputar uma olimpíada. É um recorrente ganhador de medalhas.
Colocou-as no peito inclusive nas Olimpíadas Mundiais de 1999 e 2000.
Sempre com o apoio de seu colégio, tivesse ou não a sempre bem vinda
ajudinha das instituições públicas. Quando queimava as pestanas numa
escola inscrita na junta comercial, Fabrício costumava ganhar a mesma
atenção da que foi dedicada a Larissa Lima, 17 anos. Veja a notícia
divulgada no sítio
(www.sct.ce.gov.br)
da Secitece: "Ao saber que o governo do Ceará iria patrocinar a
participação de Larissa na Global Young Leaders Conference (3 a 14 de
agosto, nos EUA), o secretário da C&T, Hélio Barros, propôs que a
entrega do cheque de R$ 7.500 fosse feita pelo governador Lúcio
Alcântara. O pedido foi aceito e a solenidade ocorreu no Palácio Iracema,
dia 4 de maio". Pois é. O aluno da pública e gratuita UFC ficou a
ver aviões. A questão é: porque nossos gestores foram sensíveis a um caso
e tão impiedosos com outro? 


UMA AULA DE BOM SENSO E EQUILÍBRIO 
É de se esperar rompantes de revolta do estudante que, após a via-crucis
pelos gabinetes de instituições públicas, não conseguiu o apoio para
disputar as Olimpíadas Internacionais. O rancor seria próprio da idade e
das circunstâncias. Ledo engano. Mensagem enviada à Coluna pelo próprio
Fabrício desmente tal idéia. "Fiquei surpreso e lisonjeado ao ler
sua coluna no Jornal O POVO. Nela, você trata de maneira
bastante crítica o descaso da administração pública e a falta de
incentivo a nós estudantes. De fato, participo de Olimpíadas de
Matemática desde a 7ª Série do Ensino Fundamental. Já tive diversas
oportunidades de viajar e trazer prêmios para o Brasil, inclusive nas
Olimpíadas Mundiais de 1999 e 2000. Nesta época, sempre tive apoio total
do meu colégio. Não com grande surpresa, percebi que na UFC as coisas se
passam de maneira bastante diferente. Assim, venho por meio deste,
agradecer-lhe pela publicação". O grifo a seguir é do editor:
Espero que a mesma tenha sido lida por nossas autoridades
universitárias e que a partir desse momento elas passem a refletir melhor
nas suas tomadas de decisão, procurando valorizar muito mais a todos nós
que fazemos a Universidade". 


MARASMO DE TECNOCRATAS CORPORATIVISTAS 
O caso despertou a indignação de leitores. Um deles foi muito duro com a
UFC. Apesar de ter cometido o pecado da generalização, vale fazer
reflexão acerca de suas ácidas palavras: "O mais incrível é
verificar que os nossos professores da UFC levam um banho no que se
refere a gestão e marketing dos gestores das escolas particulares. Logo
eles que deveriam ser referencial de excelência. O que vemos são
profissionais corporativistas, pouco interessados nas demandas sociais,
agarrados aos seus empregos e sem a menor idéia de suas funções perante a
sociedade. São esses comportamentos que favorecem a idéia daqueles que
propõem a privatização de tudo. Aonde encontrar argumentos para defender
a universidade pública enterrada no marasmo do seus tecnocratas
incompetentes, que só enxergam seus contracheques e a reforma da
previdência?" 


AS "NUANCES" DO TER E DO NÃO TER 
De gente que convive com Fabrício na UFC veio o seguinte: "A
participação do Fabrício na Olímpiada não era imprescindível à sua
carreira. Ele fará carreira brilhante de qualquer forma, dada a sua
capacidade intelectual e emocional. Entretanto, ele teria orgulho, e para
tanto fez a sua parte, em representar o Brasil e o Ceará. Com certeza, a
sua participação contribuiria de maneira efetiva para o sucesso da equipe
brasileira. Para o Fabrício, na ocasião, disse algumas palavras de
consolo e encorajame

Re: [obm-l] ENQUETE - BELEZA MATEMATICA

2003-08-14 Por tôpico Helder Suzuki
 --- Henrique_Patrício_Sant'Anna_Branco
<[EMAIL PROTECTED]> escreveu: > > Por mais que
eu ache pedante e ridiculo alguem se
> vangloriar de ter o QI
> > mais alto do mundo, nesse caso acho que a Marilyn
> estah certa. Voce deve
> > trocar de porta.
> >
> > Desculpem a minha ignorancia, mas o que ha de
> errado com o argumento de 1
> > milhao de portas? Me parece que, nesse caso, a
> probabilidade de voce ter
> > escolhido a porta certa de primeira eh apenas de
> 1/1.000.000. Logo, a
> > probabilidade da outra porta ter o premio eh de
> 999.999/1.000.000. Ou nao?
> 
> Cláudio,
> 
> No problema original, temos três portas, escolhemos
> uma e o apresentador
> logo em seguida abre outra que, com certeza, não tem
> o prêmio. Inicialmente,
> havia uma chance de 1/3 de uma determinada porta
> conter o prêmio. Ao ser
> aberta uma das portas e mostrar que ela não contém o
> prêmio, sobram apenas
> duas portas: a que você escolheu e uma outra. É como
> se a probabilidade
> tivesse sido "atualizada" pelo fato do apresentador
> mostrar uma porta que
> não contém o prêmio (isso é o Teorema de Bayes se
> não me engano). Agora que
> sobraram apenas duas portas, cada uma delas tem uma
> em duas chances (1/2) de
> ter o prêmio e, portanto, não há justificativa
> (matematica) para trocar de
> porta ou não. O fato do apresentador abrir uma das
> portas muda a
> probabilidade das DUAS portas e não apenas para uma,
> como a Sra. Marilyn
> quer nos fazer crer.
> 
> Quanto ao argumento de 1 milhão de portas... Como
> você disse, a
> probabilidade de você ter escolhido a porta certa de
> primeira é de 1/10^6
> que é a mesma probabilidade de cada uma das outras
> portas individualmente.
> Lembre-se que todas as probabilidades devem somar 1
> = 10^6/10^6. O caso que
> você apontou (999.999/10^6) é a probabilidade
> combinada de todas as outras
> portas (cada uma entre as 10^6 portas têm
> probabilidade de 1/10^6) que você
> não escolheu de terem o prêmio e não de uma única
> porta das que você não
> escolheu. Se você simplesmente muda de porta, a
> probabilidade continua sendo
> a mesma... E, se ele abrir 777.777 portas sem o
> prêmio, a probabilidade de
> TODAS as portas fica em 1/222.223 e, novamente, não
> faz diferença mudar a
> porta...
> 
> Espero ter sido claro.
> Abraço,
> Henrique.

Oi Henrique!

Faz diferença mudar de porta nesses casos.
Se tem 1 milhão de portas, com apenas 1 premiada, a
chance de vc escolher ela é 10^-6
se o cara abre uma porta que não tem o prêmio, vc pode
mudar de porta e ter uma chance de (10^6-2)/(10^6-1)
de acertar a porta certa, caso vc não tenha pego ela
na primeira escolha.

[]'s,
Hélder T. Suzuki

___
Conheça o novo Cadê? - Mais rápido, mais fácil e mais preciso.
Toda a web, 42 milhões de páginas brasileiras e nova busca por imagens!
http://www.cade.com.br
=
Instruções para entrar na lista, sair da lista e usar a lista em
http://www.mat.puc-rio.br/~nicolau/olimp/obm-l.html
=


[obm-l] Re: [obm-l] Problema das Tres Portas

2003-08-14 Por tôpico bmat
Bom, quanto aos argumentos matemáticos, acho que o Cláudio já falou tudo.
Mas é impressionante como este problema é difícil, pois ele envolve separar
o que da informação do apresentador é útil e o que é inútil. Este problema
me foi exposto recentemente durante o Colóquio Brasileiro de Matemática,
numa palestra de "Probabilistic Reasoning", que citou outro problema que
o Nicolau comenta no seu artigo da Eureka!, também bastante contra-intuitivo
e genial.

Mas, voltando ao problema: Quando você escolhe a porta, qual a probabilidade
de ela estar certa? 1/3, correto?
Ok. O que acontece quando o apresentador abre uma porta? Ele diz para você:
"Olha, se não estiver na que você escolheu, está na outra!!!" Você agora
pode escolher entre duas portas, certo? Mas isso não quer dizer que as probabilidades
são iguais, pois você não ganhou NADA em relação à sua porta, já que o apresentador
não pode fazer NADA com a sua (ele não escolhe se abre a sua ou não, ele
NÃO pode abrir a sua). Mas a outra ele ESCOLHEU não abrir. Ou seja, ele
ESCOLHEU abrir a outra.

Pense assim: O prêmio pode estar na 1, na 2 ou na 3. Você escolhe a 1. O
apresentador pode abrir a 2 ou a 3, apenas. Se o prêmio estivesse numa delas,
ele FORÇOSAMENTE terá que abrir a OUTRA! OU seja, se você errou, trocando
você passa a ganhar. Por outro lado, se você acertou na escolha inicial,
tanto faz para o apresentador, ele pode abrir qualquer uma, que não tem
restrição.
Ou seja, a "informação" relevante é a seguinte: se você trocar, a probabilidade
de ganhar é a probabilidade de você ter errado. (o que talvez fique bem
claro com o exemplo de 1.000.000 de portas)

A "grande sacada" consiste em você se convencer de que, se você trocar,
você vai ganhar SEMPRE que tiver errado o primeiro palpite. Uma vez isso,
o jogo é fácil: você tem 1/3 de chances de acertar e o resto é erro! Ao
abrir as portas, o apresentador consolida o resto do jogo: ao trocar, você
troca seu estado, de certo para errado e vice-versa só que como P(errar
o primeiro palpite) > P(acertar o primeiro palpite), vale a pena trocar.

É isso... qualquer dúvida, pergunte.
Bernardo Costa

-- Mensagem original --

>Claudio,
>
>Matematicamente, tanto faz, porque a probabilidade de se ganhar é a mesma.
>
>Agora, se você for supersticioso, a coisa muda um pouco de figura. Mas
eu,
>
>pessoalmente, não mudaria.
>Abraços,
>Bernardo
>
>
>>From: Claudio Buffara <[EMAIL PROTECTED]>
>>Reply-To: [EMAIL PROTECTED]
>>To: <[EMAIL PROTECTED]>
>>Subject: [obm-l] Problema das Tres Portas
>>Date: Mon, 11 Aug 2003 22:58:22 -0300
>>
>>Bernardo:
>>
>>Pra resumir, qual eh a sua conclusao? O jogador deve ou nao deve trocar
>de
>>porta?
>>
>>Claudio.
>>
>>on 11.08.03 21:51, Bernardo Vieira Emerick at [EMAIL PROTECTED]
>>wrote:
>>
>> > Que piada!!! Marylin vos Savant, tida como a pessoa com o maior QI
do
>
>>mundo
>> > (concordo com o Domingos Jr.: bulsshit!) confundiu tudo. O problema
era
>> > assim: num jogo, a pessoa escolha uma entre três portas. O apresentador,
>> > então, abra uma das portas. Como ele sabe qual é a porta que contém
o
>> > prêmio, ele abre uma que não o contém - já que o jogo dar-se-ia por
>> > encerrado. A pergunta é: o jogador deveria trocar de porta?
>> > Segundo Marylin, sim!, porque a probabilidade da opção que ele teria
>> > continuaria 1/3, enquanto a outra aumentaria para 2/3!!! Qual a razão
>
>>disso?
>> > A probabilidade da porta que ele escolheu não poderia subir subitamente
>
>>para
>> > 1/2, como sugerem os matemáticos. Ora, como então a outra porta pode???
>
>>Isso
>> > ela não explica.
>> > Ela aparentemente desconhece o conceito primeiro de probabilidade,
que
>é 
>>a
>> > chance de se acertar, e por isso está atrelada ao número de 
>>possibilidades
>> > possíveis e o número de possibilidades "requeridas" para se acertar
o
>> > resultado. Então, a probabilidade será dada - como é de conhecimento
>
>>geral,
>> > exceto possivelmente de Marylin - pela divisão do número de 
>>possibilidades
>> > "requeridas" pelo número total de possibilidades. Parece-me que ela

>>acredita
>> > que a única forma de se aumentar a probabilidade é aumentando o número
>
>>de
>> > possibilidades "requeridas". Isso justificaria o "we've learned nothing
>
>>to
>> > allow us to revise the chances on the shell under your finger" que
ela
>
>>diz.
>> > O que mudou, e que ela incrivelmente não percebeu, é o número total
de
>> > possibilidades. Simplificando para ela, o numerozinho de baixo diminuiu,
>> > então o número do outro lado do sinal de igualdade aumentou, já que
o
>> > numerozinho de cima da fração permaneceu constante. Será que assim
ela
>> > entenderia???
>> >
>> >
>> >
>> >> From: "Domingos Jr." <[EMAIL PROTECTED]>
>> >> Reply-To: [EMAIL PROTECTED]
>> >> To: <[EMAIL PROTECTED]>
>> >> Subject: Re: [obm-l] ENQUETE - BELEZA MATEMATICA
>> >> Date: Mon, 11 Aug 2003 19:03:11 -0300
>> >>
>> >> O Noga Alon conta que fizeram esta pergunta para ele uma vez que ele
>> >> começou explicando a prova de Euclides de que h

Re: [obm-l] ENQUETE - BELEZA MATEMATICA

2003-08-14 Por tôpico Claudio Buffara
on 09.08.03 18:39, Frederico Reis Marques de Brito at [EMAIL PROTECTED]
wrote:

> 1) Acho que esse será praticamente unânime: Teorema de Euclides sobre a
> exist~encia de  infinitos primos.
> 
> 2) Teorema de Bezout sobre MDC: O máximo dvisor comum de dois inteiros é uma
> comb. linear inteira ( em realidade a menor positiva ) desses números ,
> pelas várias aplicações deste na Teoria dos Números.
> 
> 3) O Princípio de Dirichlet, pela potência .
> 
> 4) O posto-linha = posto-coluna. Não sei mais sempre achei este resultado
> muito inusitado, já que uma matriz nada mais é que "um amontoado" de
> números...
> 
> 5) Teorema do Valor Intermediário ( Acho que podemos abordar este tema no
> ensino médio... )
> 
> Como o Morgado, pensarei um pouco mais antes de enviar outros 5. (  A
> propósito é tentador citar o Teorema dos Números Primos, mas acho que esse
> tema não seria acessível. Fica um voto de louvor então!)
> 
> Frederico.
> 
Oi, Frederico.

O principio de Dirichlet a que voce se refere eh o das casas de pombos?

O TNP eh um pouco avancado demais (mesmo com uma demonstracao elementar),
mas aquelas desigualdades de Chebichev sao aceitaveis, assim como o
postulado de Bertrand, que penso seriamente em botar na minha lista.

Pra mim, o TVI (e qualquer resultado que dependa do axioma do supremo) foi
um caso mais dificil de decidir, mas como tem aquele probleminha do monge
subindo e descendo a montanha, acho que ele tambem eh aceitavel. E, afinal
de contas, tem um volume da colecao do Iezzi que trata de limites, derivadas
e integrais...


Um abraco,
Claudio.

=
Instruções para entrar na lista, sair da lista e usar a lista em
http://www.mat.puc-rio.br/~nicolau/olimp/obm-l.html
=


Re: [obm-l] ENQUETE - BELEZA MATEMATICA

2003-08-14 Por tôpico Johann Peter Gustav Lejeune Dirichlet
Um que e o meu preferido, alias o meu par teorema-problema preferido:a 
Desigualdade de Erdös-Mordell e o problema de geometria da IMO 1996.Ja 
enviei ha bastante tempo uma mensagem completa com a solução do segundo mas 
poucos tiveram tempo de ler (eu mesmo nem tive tempo de rever esta perola 
problemistica!!!).Quem achar uma desigualdade mais legal,meus parabens.Caso 
contrario continue tentando!!
Ass.;


From: "Eduardo Casagrande Stabel" <[EMAIL PROTECTED]>
Reply-To: [EMAIL PROTECTED]
To: <[EMAIL PROTECTED]>
Subject: Re: [obm-l] ENQUETE - BELEZA MATEMATICA
Date: Tue, 12 Aug 2003 16:32:27 -0300
Olá!

É minha vez de enviar meus problemas/teoremas bonitos...

1) O teorema, devido a Euler, que diz que quando s > 1 temos ZETA(s) =
SOMA{ 1/n^s, n=1...infinito } = PRODUTORIO { (1 - p^(-s) ), p primo }.
2) A surpreendente constatação de que um problema aparentemente não tão
complicado como o último teorema de Fermat tenha uma solução tão extensa e
complicada.
3) A demonstração (enviada para a lista) de que um número irracional 
elevado
a um número irracional pode resultar um número racional. A saber, se p =
raiz(2) ^ raiz(2) é racional, está acabado; se p não for racional, é
irracional, e q = p ^ raiz(2) = 2 satisfaz o problema.

4) O método (não lembro de quem é, talvez Cauchy) para aproximar a soma de
séries SOMA{ s_n } onde s_1 > s_2 > s_3 > ... > 0, utilizando-se uma
integral.
5) A constatação maravilhosa de que certas constantes (como Pi e e) são
constantes em toda a matemática, isto é, aparecem em diversas áreas
(aparentemente desconexas) como geometria, análise, teoria dos números,
probabilidade, etc. dando a entender que toda a matemática tem um centro
firme (de onde saem os resultados) e uma arquitetura já planejada por 
alguém
bem mais inteligente que nós...

6) O método do escalonamento de matrizes descoberto por Gauss. Brilhante 
por
ser tão simples e ter levado tanto tempo para ser descoberto. Segundo o
Gilbert Strang: "Ele é tão simples que mesmo qualquer um de nós poderíamos
tê-lo descoberto...".

7) O fato de que podemos definir num espaço vetorial uma função com
propriedades simples (=o produto interno) e dele derivarmos muitos e muitos
resultados interessantes. (isto realmente me surpreendeu, quando comecei a
estudar álgebra linear, pareceu mágica a existência e as conseqüências do
produto interno)
8) A menos de isomorfismo, o conjunto dos Reais é o único corpo ordenado
completo.
9) O primeiro teorema da inconsistência de Gödel.

E para finalizar:

10) O princípio da indução finita.

Abração,
Duda.
From: "Claudio Buffara" <[EMAIL PROTECTED]>
> Caros colegas da lista:
>
> Gostaria de contar com sua participacao numa enquete sobre "beleza
> matematica".
>
> O que eu precisao eh que cada um de voces me envie uma lista contendo 
algo
> como 5 a 10 problemas/teoremas que voces consideram os mais bonitos e
cujas
> solucoes/demonstracoes sao as mais elegantes e/ou inusitadas e/ou
> engenhosas. Nao precisa incluir a solucao/demonstracao, apenas o
enunciado.
> No entanto, se voce tiver em mente uma solucao/demonstracao especifica
> (entre varias existentes) nao deixe de mencionar pelo menos o metodo
> utilizado.
>
> A unica restricao eh que estes resultados devem ser de um nivel 
acessivel
a
> um aluno normal de 2o. grau (ou seja, o Ultimo Teorema de Fermat e o
Porisma
> de Poncelet estao fora, mas o caso n = 4 do UTF e a versao para 
triangulos
> do Porisma poderiam ser incluidos).
>
> Importante: os resultados devem ser acessiveis a um aluno normal de 2o.
> grau, mas nao necessariamente fazer parte do curriculo normal do 2o. 
grau.
>
> Tambem nao precisa responder hoje ou amanha ou mesmo na semana que vem.
Acho
> que vale a pena pensar por um tempo e consultar a literatura - as vezes
pode
> ter um resultado belissimo do qual voce simplesmente se esqueceu por nao
> encontra-lo ha muito tempo. As Eurekas sao uma otima referencia. O 
"Proofs
> from the Book" tambem, apesar de nem tudo lah ter nivel de 2o. grau.
>
> Se houver um numero suficiente de respostas, eu me comprometo a publicar
uma
> compilacao dos problemas e teoremas mais votados.
>
> Desde jah a gradeco o interesse de quem quiser participar.
>
> Um abraco,
> Claudio.
>
>
> 
=
> Instruções para entrar na lista, sair da lista e usar a lista em
> http://www.mat.puc-rio.br/~nicolau/olimp/obm-l.html
> 
=
>
>

=
Instruções para entrar na lista, sair da lista e usar a lista em
http://www.mat.puc-rio.br/~nicolau/olimp/obm-l.html
=
_
MSN Messenger: converse com os seus amigos online.  
http://messenger.msn.com.br

=
Instruções para en

Re: [obm-l] Ajuda

2003-08-14 Por tôpico Cláudio \(Prática\)
Oi, Thyago:

A solução "padrão" pra esse tipo de problema realmente envolve complexos e
polinômios.

Tentando resolver outros problemas similares, você vai perceber que
complexos e polinômios são uma forma de resolução bastante natural.

Os resultados básicos são os seguintes:
1) Todo número complexo pode ser representado na forma R*(cos(a) +
i*sen(a)), onde "R" é um real não negativo e "a" é um real qualquer (mas
normalmente limitado ao intervalo [0, 2pi) ou então (-pi,pi]);
2) e^(i*a) = cos(a) + i*sen(a): essa é a definição da função exponencial
complexa, que permite, por exemplo, que você transforme sequências de senos
e cossenos de números reais em PA em sequências de complexos em PG, que as
vezes são mais fáceis de manipular;
3) Um polinômio com coeficientes reais pode ser expresso como o produto de
binômios da forma (x - b) e/ou trinômios da forma (x^2 - 2*R*cos(a)*x +
R^2), onde a e b são números reais quaisquer e R é um real positivo.

Um abraço,
Claudio.


- Original Message -
From: "dex" <[EMAIL PROTECTED]>
To: <[EMAIL PROTECTED]>
Sent: Monday, August 11, 2003 11:05 AM
Subject: [obm-l] Ajuda


> Olá pessoal
>
> Gostaria de saber uma boa demonstração para o exercício abaixo
>
> P = sen(pi/n) . sen(2pi/n) . sen(3pi/n) . ... . sen[(n-1)pi/n]
> com n Inteiro positivo
>
> A resposta é P = n/[2^(n-1)], mas cheguei até este resultado de uma
maneira
> muito pouco prática, nada natural para uma questão de matemática (de
> vestibular). Consegui prová-la utilizando o resultado de uma outra
questão,
> que versava sobre polinômios e complexos. Ou seja, se eu não tivesse visto
> esta outra questão não conseguiria provar nada!
>
> Atneciosamente
> ¡Thyago!
>

=
Instruções para entrar na lista, sair da lista e usar a lista em
http://www.mat.puc-rio.br/~nicolau/olimp/obm-l.html
=


[obm-l] geometria

2003-08-14 Por tôpico elton francisco ferreira
A a´rea de um triangulo de perímetro 54m circunscrito
a um círcuo de 25pim^2, em m^2 e´?

___
Conheça o novo Cadê? - Mais rápido, mais fácil e mais preciso.
Toda a web, 42 milhões de páginas brasileiras e nova busca por imagens!
http://www.cade.com.br
=
Instruções para entrar na lista, sair da lista e usar a lista em
http://www.mat.puc-rio.br/~nicolau/olimp/obm-l.html
=


Re: [obm-l] ENQUETE - BELEZA MATEMATICA

2003-08-14 Por tôpico Paulo Santa Rita
Ola Claudio !

Muito legal essa sua enquete. Bom, so pode entrar resultados elementares 
e/ou de facil compreensao, certo ? Entao me ocorre de imediato alguns 
resultados.

PRIMEIRO ( trivial, mas mercece um quadro na parece. Devido a Bernoulli )

1^P + 2^P + 3^P + ... + (N-1)^P + N^P = [(N+B)^P  -  B^P]/(P+1)
onde B^k deve ser interpretado como o K-esimo numero de bernoulli.
Alias, foi verificando as somas das potencias P-esimas dos numeros naturais 
que Bernoulli descobriu os fantasticos numeros que hoje levam o seu nome. 
Mais adiante, quando eu estiver mais tranquilo, vou escrever sobre este 
tema.

SEGUNDO ( Isso nao e um principio, e um Salmo do Profeta. Devido a Erdos )

"Se em um conjunto de objeto, um objeto tem uma probabilidade menor que 1 de 
ter uma determinada propriedade, entao existe um objeto do conjunto com 
aquela propriedade"

Esse principio, nao obstante muito contestado e criticado por alguns, e 
poderoso e acredito que abre novas e imensas possibilidades para o 
pensamente matematico.

TERCEIRO ( trivial, mas facilita a prova de muitas coisas. A desigualdade 
Eduardo Wagner )

Em todo triangulo, o semi-perimetro nunca e menor que a soma dos produtos de 
cada lado pelo cosseno do angulo oposto"

p >= a*cosA + b*cosB + c*cosC

Com a desigualdade acima da pra derivar quase todas as desigualdades 
complicadas da Geometria Elemntar.

Um Abraco
Paulo Santa Rita
7,1425,090803
EM TEMPO. Sobre a beleza matematica :

A Divina Proporcao
Um Ensaio sobre a beleza na Matematica
H. E. Huntley
Editora UnB
O autor mostra como o numero "fi", ( 1 + raiz_quadrada(5) )/2, aparece nas 
mais diversas circunstancias e inesperadas circunstancias, sempre com um 
toque de inegavel beleza. Eu acredito que este numero contribuem pelo menos 
com um resultado :

"A UNICA progressao geometrica de termos positivos que na qual An+1 = An + 
An-1 e a sequencia :
1, fi, fi^2, fi^3, fi^4, ..."


From: Claudio Buffara <[EMAIL PROTECTED]>
Reply-To: [EMAIL PROTECTED]
To: Lista OBM <[EMAIL PROTECTED]>
CC: Claudio Buffara <[EMAIL PROTECTED]>
Subject: [obm-l] ENQUETE - BELEZA MATEMATICA
Date: Sat, 09 Aug 2003 10:24:26 -0300
Caros colegas da lista:

Gostaria de contar com sua participacao numa enquete sobre "beleza
matematica".
O que eu precisao eh que cada um de voces me envie uma lista contendo algo
como 5 a 10 problemas/teoremas que voces consideram os mais bonitos e cujas
solucoes/demonstracoes sao as mais elegantes e/ou inusitadas e/ou
engenhosas. Nao precisa incluir a solucao/demonstracao, apenas o enunciado.
No entanto, se voce tiver em mente uma solucao/demonstracao especifica
(entre varias existentes) nao deixe de mencionar pelo menos o metodo
utilizado.
A unica restricao eh que estes resultados devem ser de um nivel acessivel a
um aluno normal de 2o. grau (ou seja, o Ultimo Teorema de Fermat e o 
Porisma
de Poncelet estao fora, mas o caso n = 4 do UTF e a versao para triangulos
do Porisma poderiam ser incluidos).

Importante: os resultados devem ser acessiveis a um aluno normal de 2o.
grau, mas nao necessariamente fazer parte do curriculo normal do 2o. grau.
Tambem nao precisa responder hoje ou amanha ou mesmo na semana que vem. 
Acho
que vale a pena pensar por um tempo e consultar a literatura - as vezes 
pode
ter um resultado belissimo do qual voce simplesmente se esqueceu por nao
encontra-lo ha muito tempo. As Eurekas sao uma otima referencia. O "Proofs
from the Book" tambem, apesar de nem tudo lah ter nivel de 2o. grau.

Se houver um numero suficiente de respostas, eu me comprometo a publicar 
uma
compilacao dos problemas e teoremas mais votados.

Desde jah a gradeco o interesse de quem quiser participar.

Um abraco,
Claudio.
=
Instruções para entrar na lista, sair da lista e usar a lista em
http://www.mat.puc-rio.br/~nicolau/olimp/obm-l.html
=
_
MSN Hotmail, o maior webmail do Brasil.  http://www.hotmail.com
=
Instruções para entrar na lista, sair da lista e usar a lista em
http://www.mat.puc-rio.br/~nicolau/olimp/obm-l.html
=


Re: [obm-l] Problema das 3 portas

2003-08-14 Por tôpico Henrique Patrício Sant'Anna Branco
> Oi, Henrique:
> Eu insisto que a estrategia otima eh trocar de porta.

Cláudio,

Realmente, me enganei. Mas esse problema é, de fato, bem sutil...
Inicialmente nos leva a crer, sem sombra de dúvidas, que as probabilidades
ficam em 1/2, de qualquer forma.
Lendo as suas argumentações, bem como as do Nicolau, percebe-se claramente o
erro do meu raciocínio.

Peço desculpas se, eventualmente, confundi outros com minhas dúvidas.

Abraços,
Henrique.

=
Instruções para entrar na lista, sair da lista e usar a lista em
http://www.mat.puc-rio.br/~nicolau/olimp/obm-l.html
=


Re: [obm-l] ENQUETE - BELEZA MATEMATICA

2003-08-14 Por tôpico Frederico Reis Marques de Brito
1) Acho que esse será praticamente unânime: Teorema de Euclides sobre a 
exist~encia de  infinitos primos.

2) Teorema de Bezout sobre MDC: O máximo dvisor comum de dois inteiros é uma 
comb. linear inteira ( em realidade a menor positiva ) desses números , 
pelas várias aplicações deste na Teoria dos Números.

3) O Princípio de Dirichlet, pela potência .

4) O posto-linha = posto-coluna. Não sei mais sempre achei este resultado 
muito inusitado, já que uma matriz nada mais é que "um amontoado" de 
números...

5) Teorema do Valor Intermediário ( Acho que podemos abordar este tema no 
ensino médio... )

Como o Morgado, pensarei um pouco mais antes de enviar outros 5. (  A 
propósito é tentador citar o Teorema dos Números Primos, mas acho que esse 
tema não seria acessível. Fica um voto de louvor então!)

Frederico.

From: Claudio Buffara <[EMAIL PROTECTED]>
Reply-To: [EMAIL PROTECTED]
To: Lista OBM <[EMAIL PROTECTED]>
CC: Claudio Buffara <[EMAIL PROTECTED]>
Subject: [obm-l] ENQUETE - BELEZA MATEMATICA
Date: Sat, 09 Aug 2003 10:24:26 -0300
Caros colegas da lista:

Gostaria de contar com sua participacao numa enquete sobre "beleza
matematica".
O que eu precisao eh que cada um de voces me envie uma lista contendo algo
como 5 a 10 problemas/teoremas que voces consideram os mais bonitos e cujas
solucoes/demonstracoes sao as mais elegantes e/ou inusitadas e/ou
engenhosas. Nao precisa incluir a solucao/demonstracao, apenas o enunciado.
No entanto, se voce tiver em mente uma solucao/demonstracao especifica
(entre varias existentes) nao deixe de mencionar pelo menos o metodo
utilizado.
A unica restricao eh que estes resultados devem ser de um nivel acessivel a
um aluno normal de 2o. grau (ou seja, o Ultimo Teorema de Fermat e o 
Porisma
de Poncelet estao fora, mas o caso n = 4 do UTF e a versao para triangulos
do Porisma poderiam ser incluidos).

Importante: os resultados devem ser acessiveis a um aluno normal de 2o.
grau, mas nao necessariamente fazer parte do curriculo normal do 2o. grau.
Tambem nao precisa responder hoje ou amanha ou mesmo na semana que vem. 
Acho
que vale a pena pensar por um tempo e consultar a literatura - as vezes 
pode
ter um resultado belissimo do qual voce simplesmente se esqueceu por nao
encontra-lo ha muito tempo. As Eurekas sao uma otima referencia. O "Proofs
from the Book" tambem, apesar de nem tudo lah ter nivel de 2o. grau.

Se houver um numero suficiente de respostas, eu me comprometo a publicar 
uma
compilacao dos problemas e teoremas mais votados.

Desde jah a gradeco o interesse de quem quiser participar.

Um abraco,
Claudio.
=
Instruções para entrar na lista, sair da lista e usar a lista em
http://www.mat.puc-rio.br/~nicolau/olimp/obm-l.html
=
_
MSN Hotmail, o maior webmail do Brasil.  http://www.hotmail.com
=
Instruções para entrar na lista, sair da lista e usar a lista em
http://www.mat.puc-rio.br/~nicolau/olimp/obm-l.html
=


Re: [obm-l] Geomeria Plana

2003-08-14 Por tôpico André Martin Timpanaro
Como DF=DE+EF, A=(75/5).2=30

André T.


From: elton francisco ferreira <[EMAIL PROTECTED]>
Reply-To: [EMAIL PROTECTED]
To: [EMAIL PROTECTED]
Subject: [obm-l] Geomeria Plana
Date: Mon, 11 Aug 2003 15:35:59 -0300 (ART)
Um triângulo ABC tem área 75m^2. os pontos D,E,F e G
dividem o lado AC em 5 partes congruentes:
AD=DE=EF=FG=GC. Desse modo, a área do triangulo BDF é:
20
30
40
50
55
___
Conheça o novo Cadê? - Mais rápido, mais fácil e mais preciso.
Toda a web, 42 milhões de páginas brasileiras e nova busca por imagens!
http://www.cade.com.br
=
Instruções para entrar na lista, sair da lista e usar a lista em
http://www.mat.puc-rio.br/~nicolau/olimp/obm-l.html
=
_
MSN Messenger: converse com os seus amigos online.  
http://messenger.msn.com.br

=
Instruções para entrar na lista, sair da lista e usar a lista em
http://www.mat.puc-rio.br/~nicolau/olimp/obm-l.html
=


Re: [obm-l] EsSA

2003-08-14 Por tôpico Bernardo Vieira Emerick
Existe uma solução mais rápida. Basta subtrairmos 900 por 888, que nos dará 
12. Como sairam dois trabalhadores, dividimos 20 por dois - já que as 
parcelas são iguais -, o que nos dá 6. Note-se que essa resposta só serve 
porque as parcelas de todos os trabalhadores permanecerá constante.


From: "Henrique Patrício Sant'Anna Branco" <[EMAIL PROTECTED]>
Reply-To: [EMAIL PROTECTED]
To: <[EMAIL PROTECTED]>
Subject: Re: [obm-l] EsSA
Date: Sat, 9 Aug 2003 15:09:15 -0300
> Numa fábrica, trabalhadores reuniram-se para
> presentear um amigo que iria casar. O presente
> escolhido foi a quantia de 900,00, que seria dividida
> igualmente entre eles. Por razões particulares, dois
> daqueles trabalhadores tiraram seus nomes da lista e,
> por isso, decidiu-se diminuir a quantia para 888,00,
> de modo que na nova divisão coubesse a cada
> participante a mesma cota de antes da saída dos dois
> colegas. Com isso, coube a cada um dos participantes a
> quantia de :
Vamos chamar de x o número de trabalhadores no início da divisão. Assim,
(x-2) é o número final de participantes. Como as cotas tem que ser iguais,
temos:
900/x = 888/(x-2) ==> 12x - 1800 = 0
Resolvendo, achamos x = 150. Agora, 900/150 = 6.
Segunda opção.
Abraços,
Henrique.
=
Instruções para entrar na lista, sair da lista e usar a lista em
http://www.mat.puc-rio.br/~nicolau/olimp/obm-l.html
=
_
MSN Hotmail, o maior webmail do Brasil.  http://www.hotmail.com
=
Instruções para entrar na lista, sair da lista e usar a lista em
http://www.mat.puc-rio.br/~nicolau/olimp/obm-l.html
=


Re: [obm-l] ENQUETE - BELEZA MATEMATICA

2003-08-14 Por tôpico Johann Peter Gustav Lejeune Dirichlet
Ah e,e??Veja o primeiro capitulo do 
Proofs from THE BOOK.Ai ce vai ver como topologia
e ultrapassado

> Claudio Buffara wrote:
> > Estou extremamente decepcionado com as listas
> de problemas supostamente
> > bonitos que foram enviadas pra lista ateh o
> presente momento. Imaginem soh -
> > teorema do valor intermediario, secoes
> conicas, poliedros regulares,
> > conjuntos enumeraveis. Onde voces estao com a
> cabeca? Isso tudo eh
> > matematica do tempo da carochinha. Infinitude
> dos primos? Isso eh tao velho
> > que ja devia ter sido revogado!
> 
> O Noga Alon conta que fizeram esta pergunta
> para ele uma vez que ele
> começou explicando a prova de Euclides de que
> há infinitos primos
> em um programa de televisão, eu acho:
> 
> And today, are there still infinitely many
> primes?
> 
> E sem sair do clima, deem uma olhada em
>
http://qsilver.queensu.ca/~phil158d/intro/montyh3.htm
> 
> Eu deveria ter visto isso antes de escrever o
> meu artigo da Eureka!
> 
> []s, N.
>
=
> Instruções para entrar na lista, sair da lista
> e usar a lista em
>
http://www.mat.puc-rio.br/~nicolau/olimp/obm-l.html
>
= 

___
Conheça o novo Cadê? - Mais rápido, mais fácil e mais preciso.
Toda a web, 42 milhões de páginas brasileiras e nova busca por imagens!
http://www.cade.com.br
=
Instruções para entrar na lista, sair da lista e usar a lista em
http://www.mat.puc-rio.br/~nicolau/olimp/obm-l.html
=


Re: [obm-l] Re:

2003-08-14 Por tôpico Eduardo Casagrande Stabel
Olá Dirichlet,

eu também pensei sobre o problema: demonstrar que não existe uma função nos
reais contínua nos racionais e somente neles. Sequer tenho alguma estratégia
ou alguma idéia de como atacar o problema. Será que alguém pode dar uma
sugestão? O único "progresso" que fiz - que nem sei se está certo - é intuir
que os racionais não são um conjunto tão especial neste enunciado, eu
suspeito que podemos substituir por enumeraveis densos nos reais.

Quem quiser fazer comentários, sinta-se à vontade.

Abraço,
Duda.

From: "Johann Peter Gustav Lejeune Dirichlet"
<[EMAIL PROTECTED]>
> Como nao consegui demonstrar isto em tempo
> finito,alguem poderia demonstrar pra mim?


=
Instruções para entrar na lista, sair da lista e usar a lista em
http://www.mat.puc-rio.br/~nicolau/olimp/obm-l.html
=


Re: [obm-l] ENQUETE - BELEZA MATEMATICA

2003-08-14 Por tôpico Bernardo Vieira Emerick
Caro Claudio,

Em termos de probabilidade não há razão para se mudar de opção, ou dee 
porta. Digamos que haja um milhão de portas. A probabilidade de se acertar 
em todas elas serão iguais entre si e iguai a 1/1.000.000.
Digamos que o apresentador abra 999.998 portas, ou seja, que só sobrem duas. 
Está certo: a probabilidade de se acertar aumentou; mas ela aumenta para 
ambas as portas. O que modificou foi o número total de possibilidades. 
Antes, tinha-se que escolher uma porta entre um milhão; depois, uma porta 
entre duas. No entanto, a probabilidade de se ganhar em ambas as portas é de 
1/2. Só seria necessário mudar a porta se após a abertura das outras portas 
a probabilidade de se acertar de uma porta ficasse maior do que a da outra.


From: Claudio Buffara <[EMAIL PROTECTED]>
Reply-To: [EMAIL PROTECTED]
To: <[EMAIL PROTECTED]>
Subject: Re: [obm-l] ENQUETE - BELEZA MATEMATICA
Date: Mon, 11 Aug 2003 19:53:06 -0300
on 11.08.03 19:03, Domingos Jr. at [EMAIL PROTECTED] wrote:

> O Noga Alon conta que fizeram esta pergunta para ele uma vez que ele
> começou explicando a prova de Euclides de que há infinitos primos
> em um programa de televisão, eu acho:
>
> And today, are there still infinitely many primes?
>
> E sem sair do clima, deem uma olhada em
> http://qsilver.queensu.ca/~phil158d/intro/montyh3.htm
>
> Eu deveria ter visto isso antes de escrever o meu artigo da Eureka!
>
> --- x ---
> Putz, essa mulher do QI mais alto do mundo (bullshit!) não concorda com 
o
> Princípio da Indução Finita também! hehehe, o pior é que é sério!!!
>
Oi, Domingos e Nicolau:

Por mais que eu ache pedante e ridiculo alguem se vangloriar de ter o QI
mais alto do mundo, nesse caso acho que a Marilyn estah certa. Voce deve
trocar de porta.
Desculpem a minha ignorancia, mas o que ha de errado com o argumento de 1
milhao de portas? Me parece que, nesse caso, a probabilidade de voce ter
escolhido a porta certa de primeira eh apenas de 1/1.000.000. Logo, a
probabilidade da outra porta ter o premio eh de 999.999/1.000.000. Ou nao?
Um abraco,
Claudio.
=
Instruções para entrar na lista, sair da lista e usar a lista em
http://www.mat.puc-rio.br/~nicolau/olimp/obm-l.html
=
_
MSN Messenger: converse com os seus amigos online.  
http://messenger.msn.com.br

=
Instruções para entrar na lista, sair da lista e usar a lista em
http://www.mat.puc-rio.br/~nicolau/olimp/obm-l.html
=


[obm-l] problemas

2003-08-14 Por tôpico elton francisco ferreira
O número de anagramas formados com as letras da
palavra ROMA de modo que não apareça vogais ou
consoantes juntas é igual a:

4!
4
8
2

um atirador deu 49 tiros, pagando 10 de multa por tiro
fora do alvo e recebendo 0,25 de premio pr tiro
acertado no alvo. Se nada recebeu e nada pagou, então
a multa foi de:

1,40
3,50
5,00
8,75

Se de um retangulo de perímetro 4 e dimensões ``x`` e
``y`` , Xhttp://www.cade.com.br
=
Instruções para entrar na lista, sair da lista e usar a lista em
http://www.mat.puc-rio.br/~nicolau/olimp/obm-l.html
=


  1   2   3   >